Unz评论•另类媒体选择$
美国主流媒体大都排除了有趣,重要和有争议的观点
 博客浏览詹姆斯·汤普森档案馆
男子提前4分?

书签 全部切换总目录添加到图书馆从图书馆中删除 • B
显示评论下一个新评论下一个新回复了解更多
回复同意/不同意/等等 更多... This Commenter This Thread Hide Thread Display All Comments
同意不同意谢谢LOL轮唱
这些按钮可将您的公开协议,异议,感谢,LOL或巨魔与所选注释一起注册。 仅对最近使用“记住我的信息”复选框保存姓名和电子邮件的频繁评论者可用,并且在任何八个小时的时间内也只能使用三次。
忽略评论者 关注评论者
搜寻文字 区分大小写  确切的词  包括评论
列表 书签

人类季刊第 58 卷第 1 期

新闻中出现了性别差异。 一位男性谷歌员工在他的工作场所实践中查阅了一些关于该主题的文献,并被解雇了。 一本质疑睾丸激素在性别差异中的作用的书,以及更普遍的先天生物性别差异的真实性,获得了皇家学会科学图书奖,尽管该领域的皇家学会研究员并未对其进行审查。 更普遍的是,关于 STEM 学科、技术工作和公司董事会缺乏女性的新闻经常出现,这些讨论往往归咎于阻碍女性进步的厌女症的玻璃天花板。 与此同时,由于宣传较少,理查德林恩教授根据最近的研究重新审视了他 1994 年的论文,并邀请评论家将他的发现分开。

正如编辑 Gerhard Meisenberg 评论的那样:

在本期《人类季刊》中,理查德·林恩 (Richard Lynn) 对他的发展理论进行了数据丰富的总结,随后是该领域学者的 10 条评论和对评论的回复。 许多评论家为性别差异之谜添加了一些经验证据,而其他人则提出了对发展理论的理论替代或改进。 总之,目标文章和评论对认知性别差异的研究现状以及该领域不同研究人员使用的理论方法提供了相当具有代表性的概述。

以下是论文:

智力的性别差异:发展理论。 理查德·林恩
男性和女性资产负债表。 詹姆斯·R·弗林
计数不是测量:评论理查德林恩的智力性别差异发展理论。 罗伯托·科洛姆
智力性别差异研究中的常见悖论。 赫尔穆斯·尼堡
认知性别差异:进化与历史。 大卫·贝克尔和海纳·林德曼
职业成就中的男性大脑、睾酮和性别差异。 爱德华·达顿
智力的性别差异:发育起源是的,詹森效应不。格哈德·迈森伯格
自我估计智力、竞争力和冒险的性别差异。 阿德里安·弗纳姆
智力的性别差异:遗传学观点。 大卫·皮弗
推定与偏见:配额可以解决一些问题,但会造成更多问题。
盖·麦迪逊
认知要求高的游戏中的性别差异:扑克、西洋双陆棋和麻将。
Heitor BF 费尔南德斯
职业围棋选手表现的性别差异。 王明瑞
智力的性别差异:回复评论。 理查德·林恩

订阅人类季刊在这里: http://www.mankindquarterly.org/subscribe

林恩教授从以下观察开始:

一个悖论是男性的平均大脑大小比女性大,大脑大小与智力呈正相关,但许多专家断言,智力没有性别差异。 本文提出了智力性别差异的发展理论作为解决这个问题的方法。 这表明男孩和女孩在 15 岁之前的智商大致相同,但从 16 岁开始,男性的平均智商高于女性,成年后优势增加到大约 4 个智商点。

林恩继续表明,该领域的大多数专家都断言智力没有性别差异,或者存在的这种差异相互抵消。 然后他继续考虑明显的异常,因为大脑的大小与智力有关,而男性的大脑比女性大,他们应该更聪明。

Pakkenberg 和 Gundersen (1997) 报告说,男性的神经元平均比女性多 16 亿,相差 2008%。 Pelvig 等人提供的进一步数据表明,男性比女性拥有更多的神经元。 (XNUMX)。

林恩接着解释了他是如何预测男性智商更高的:

为了计算可以从较大的男性大脑尺寸预测的较高成年男性 IQ 的幅度,我采用了 Ankney 的数字,该数字表示以 0.78d 的标准偏差单位和 Willerman 等人(1991 年)表示的男性-女性大脑尺寸差异) 估计大脑大小和智力之间的相关性为 0.35。 这些数字将使成年男性的平均智商更高,为 0.78 乘以 0.35 = .27d = 4.0 智商点。 在我 1994 年的论文中,我提供的数据显示,成年男性在语言能力上的优势为 1.7 IQ 点,在语言和非语言推理能力上为 2.1 IQ 点,在空间上为 7.5 IQ 点,这表明成年男性在 IQ 上的平均优势为 3.8 点和因此非常接近 4.0 IQ 点的预测优势。 我在 Lynn (1998, 1999) 中发表了关于这种男性优势的进一步数据。 Meisenberg (2009) 给出的男性优势在表 1 中给出,白人为 0.42d,黑人为 0.30d,与这些结果相当一致。

艾森克接受了我的论点,即男性的智商比女性高 4 分,并计算出这一优势加上男性标准差为男性 15 和女性为 14 的较大男性方差将产生每 55 人中 5 名男性和 10,000 名女性的 IQ 为160及以上,比例为10:1。 Nyborg (2015, p. 51) 最近提出了同样的观点,他提供的数据表明,在美国 3.9 岁白人中,男性的智商优势为 17 分,并计算出这一优势使男性与女性的比例为 5:1智商为 145(大约为每 300 名男性中的一个)。

你可能还记得我玩过这些数字,显示了男性/女性比例,这是由于对男性/女性智力差异和男性/女性标准偏差差异的不同假设造成的。

https://www.unz.com/jthompson/womens-brains

回到林恩。

表 1 显示了随着男性大脑变得更大,他们的智力优势如何变得更大。

林恩发展男性优势

事实上,即使是儿童版的韦克斯勒(6-16岁)也有男性优势,能力四项指标性别差异明显,女生表现出处理速度优势。 正如林恩讽刺地观察到的那样,每个人都错了,韦克斯勒没有性别差异,这是一项广泛的能力测试,面对面进行,因此能够监控参与度和努力程度。 此外,他错误地认为性别差异要到 16 岁才会出现。它们在此之前就已经存在。

Lynn 开发 wisc 差异

在成人韦克斯勒身上,超过 33 项研究表明男性优势明显,相当于 3.8 智商点。

林恩发展表5

成人性别能力差异的更详细性质见表6

Lynn开发表6标准化

请注意,即使在测试的标准化样本中也存在这些差异(以星号显示),出版商不愿公开这一发现,而 Lynn 仅通过第一个向他们索要数据的人获得。 这意味着 Wechsler 已经发现了性别差异,然后将其标准化,因此当临床医生在手动表格中查找结果时,隐藏了固有的性别差异。 在我看来,这是一个丑闻。

事实上,长期以来,测试人员一直致力于尽量减少男性的优势,省略他们得分更高的任务,例如空间感知和形状的心理旋转,以及机械知识(后者智商相差 10)。

林恩总结:

在所有 3.6 个样本中,男性在 WAIS Full Scale IQ 上的 33 IQ 中位数优势是对 Halpern (2000, p. 91)、Anderson (2004, p. 829) 和 Haier 等人的断言的否定。 (2004, p.1) WAIS Full Scale IQ 没有性别差异。 这也是对 Halpern (2012, p. 115) 的说法的否定,即在美国 WAIS IV 的标准化样本中“整体 IQ 分数不显示性别差异”。

谈到高成就的性别差异,林恩指出:

霍华德报告说,2012 年有 1324 名男子和 26 名女子国际象棋特级大师,在 1975 年至 2014 年间,在所有国际棋手的前 10 名和前 50 名的表现中,男性的优势大约是一个标准差。 他得出的结论是,男性能力较高是顶级国际象棋选手中男性人数较多的最合理解释:“男性在视觉空间能力方面的平均得分更高,而智商较高的男性得分更高”,而在国际象棋中男性占主导地位的是“可能部分是天生的”(第 219-20 页)。 他是对的,高智商的男性人数更多是他们在国际象棋顶级选手中人数更多的部分原因。

这篇论文还有更多内容,你可以自己阅读,但我的感觉是,林恩已经为男性在智力方面的优势提出了强有力的理由,现在批评者必须让他参与其中的详细结果。

他们这样做了吗? 这已经足够了。 稍后我会谈谈特刊中对他的论文的回复,也提请注意非回复,关于不吠叫的福尔摩斯原则,不吠叫的狗也值得关注。

注:封面为18世纪大师小笠原一斋雕刻的围棋手根附。 您或您的附属图书馆的订阅: http://www.mankindquarterly.org/subscribe

作为介绍性优惠,请在此处免费阅读整个版本:

https://drive.google.com/open?id=0B3c4TxciNeJZdkR6V3NJYWcwdE0

 
• 类别: 科学 •标签: 性别, IQ 
隐藏393条评论发表评论
忽略评论者...跟随Endorsed Only
修剪评论?
    []
  1. dearieme 说:

    “忽略[男性]得分较高的任务,例如空间感知和形状的心理旋转以及机械知识”。鉴于男性对机械事物的兴趣明显更大,我对男性拥有更好的机械知识并不感到惊讶。是否还有其他话题女性可能得分更高,但在测试中并未常规测量?例如,与婴儿打交道。

    • 回复: @res
    , @Serg Derbst
  2. res 说:
    @dearieme

    我首先想到的是处理社交网络的方法和能力。以下是这个一般领域的一些论文,但我并没有真正深入研究这些材料: http://journals.plos.org/plosone/article?id=10.1371/journal.pone.0078663
    https://www.nature.com/articles/srep01214

    尽管在我看来存在偏见,但海德的荟萃分析集(在《达莫尔事件》中进行了很多讨论)对于各种特征的性别差异来说是一个很好的起始参考: https://www.ncbi.nlm.nih.gov/pubmed/16173891

  3. 同意我们应该对最广泛的智力任务进行抽样。我认为,海德消除了分歧,尽管在许多领域分歧很小,而且从来都不是争论的焦点。我还没有研究过对情绪状态的感知是否存在真正的差异,即“情商”组合中潜在的主动部分,其中被动部分可能是人格差异,而不是智力差异。

  4. res 说:

    感谢汤普森博士的有趣帖子!还要感谢您和《人类季刊》提供的慷慨介绍。有这么多要读...

    上述结果存在一些较大差异。表 5 中的这一对引起了我的注意:
    国家/地区测试 N FS VP 参考
    美国 W-Bell 235 0.59 0.63 0.35 Strange & Palmer,1953 年
    美国 W-贝尔 153 0.20 0.52 -0.35 诺曼,1953

    考虑到相同的国家、相同的考试和年份,这种表现智商的差异是惊人的。有什么想法吗?

    我想仔细检查一件事。林恩在他的论文中谈到了在观察大脑大小时控制身体大小。但如果我理解正确的话,他并没有在上面的表 1 中进行控制。有什么办法可以看到数据的受控情况吗?论文正文提到,身体尺寸的相对变化是同时发生的。

    有谁知道关于控制身体尺寸的技术的很好的讨论?在某些时候,我会尝试追查林恩的参考文献——“在控制身体尺寸时得到确认
    Ankney (1992) 和 Rushton (1992)”——但是考虑到有多少种可能的控制(例如使用什么指数),最好进行平衡的讨论。

    我快速浏览了这些参考文献。链接: http://www.sciencedirect.com/science/article/pii/016028969290013H
    http://www.sciencedirect.com/science/article/pii/016028969290017L
    我只找到了后者的PDF。第 6 页讨论了校正(例如指数),表 3 和图 1 提供了相当一致的(我猜测样本量较小导致“蒙古人种女性”的变异)校正后的差异。然而,我没有看到任何关于“这是我们推荐的纠正尺寸差异的方法”的内容。

    • 回复: @James Thompson
    , @Santoculto
  5. @res

    对表 5 进行快速评论。那里的样本量可能太小。 WAIS 是更健康的尺寸,并且 d 更小。
    “控制身体尺寸”。为什么?,这是一个大问题。物种之间可能是明智的,但性别大小差异并不那么大,所以在某些方面我会说:忘记校正,只需测量大脑大小即可。鲸鱼则不同。

    • 回复: @res
    , @EH
  6. res 说:
    @James Thompson

    “控制身体尺寸”。为什么?,这是一个大问题。物种之间可能是明智的,但性别大小差异并不那么大,所以在某些方面我会说:忘记校正,只需测量大脑大小即可。鲸鱼则不同。

    对此有一些话要说,但我不喜欢在林恩所主张的方向上引入偏见。让我尝试估计不同体重的影响大小,并将其与原始性别差异的大小进行比较。

    Rushton 1992 的一些相关摘录:

    性别非常显着,F(1,6313) = 6,634.18,p < .001;平均而言,男性的颅容量比女性大(加权 Ms = 1,462、1,266 cma;未加权 Ms = 1,471、1,282 cm3)。
    ...
    平均而言,身体状况良好的女性(假设军人也是如此)其体重的约 20% 为脂肪,而男性的脂肪仅占体重的 10%。因此,为了解释大部分非神经支配组织的差异,我将每位女性的体重减去 20%,将每位男性的体重减去 10%。新值称为“调整权重”。体重和哺乳动物脑重量之间异速生长关系的指数不是 1.0(Jerison,1982),范围从相似物种比较中的 20 到不同物种比较中的 67(Pagel & Harvey,1989)。
    ...
    在调整身高和体重以及种族、等级或性别的影响后,蒙古人种平均为 1,416 立方厘米,高加索人为 1,380 立方厘米,尼格罗人为 1,359 立方厘米;军官平均 3 立方厘米,士兵平均 1,393 立方厘米;男性平均为 1,375 厘米,女性为 3 厘米 1,442。表 1,332 中显示的所有调整都没有改变结果的总体模式(参见表 3,第 2 页)。

    因此,我们正在研究控制后的性别大脑大小比率差异为 0.924,而表 1(Lynn,上面,我认为来自 Rushton 加权 1266/1462)比率为 0.866

    这表明,在初始(原始)13.4% 的差异中,5.8% 被各种控制措施(不仅仅是尺寸)消除了。因此,控制其他变量带来的变化占原始差异的 43%。我认为这是值得担心的。

    让我们从另一个角度来看这个问题。假设相似物种权重系数为 0.20,并使用 Rushton 的男性 10%、女性 20% 权重理念。使用美国平均 M/F 高度 5'9" 和 5'3.5": https://en.wikipedia.org/wiki/List_of_average_human_height_worldwide
    并查看此处中框的中间: http://www.healthchecksystems.com/heightweightchart.htm
    我们得到的 M/F 重量为 154/129.5 磅
    给我们一个比率 (129.5*0.8)^0.2 / (154*0.9)^0.2 = 0.9435

    恕我直言,这个数字与上面估计的 5.65% 惊人地接近(低 5.8%)(这是我唯一的计算)。尽管这些数字并不完全具有可比性,但我认为它们表明了相似的效应大小。

    鉴于此,我认为有必要纠正群体差异中男性/女性身体大小的差异(当考虑可能更大的个体差异时更是如此)的大脑大小。

    PS Ankney 1992 看起来像是 Rushton 1992 中的重要参考文献,我在上面犯了一个错误,PDF 可以在 libgen 上找到: http://www.sciencedirect.com/science/article/pii/016028969290013H
    有一些关于男性和女性大脑大小与身体大小(例如体重、表面积)的良好图。

    PPS 0.2 指数数来自 https://www.ncbi.nlm.nih.gov/pubmed/2740904
    PDF 也在 libgen 上。我认为拉什顿从第二页底部得到了 0.2 指数。

    • 回复: @James Thompson
  7. AP 说:

    詹森声称,男女智商不存在显着差异(与种族差异不同),因为尽管女性大脑平均较小,但它们的组织效率也比男性大脑更高(神经元密集),在某种程度上消除了智商差异。智力。他写道,每个性别群体的大脑大小和智力之间都存在很强的相关性(大脑较大的女性往往比大脑较小的女性更聪明;男性也是如此),但不同群体之间没有这种相关性。他还推测,男性大脑体积的一些增加可能涉及与智力没有直接关系的领域,例如空间能力(能够判断一个人在哪里),并且神经元效率可能存在差异。

    • 回复: @James Thompson
  8. EH 说:

    离题但普遍感兴趣的:

    人们早就知道,正态分布与尾部智力的实际分布不符,低估了高分者的真实数量。真实的分布仍然是一个有争议的问题——我很多年前读到它可能是皮尔逊 IV 型分布(对此我一无所知),但更常见的是对数正态分布被认为接近真实的智力曲线。虽然标准电子表格程序中包含对数正态分布,但不能只插入 15 标准差和 100 均值,这对大多数人来说不太方便。

    以下是转换:
    100 的等效对数正态平均值 is: LN(100)-0.5*LN((15/100)^2+1) = 4.5940
    (LN 是自​​然对数函数,不是 log-normal 的缩写)

    等效对数正态标准差为 15 is: (LN((15/100)^2+1))^0.5 = 0.14917

    (我似乎记得读过,16 而不是 15 的 sd 实际上具有对数正态 IQ 的一些偶然属性,但不记得细节了。)

    作为使用此方法的示例,以下是 Excel/OpenOffice 公式,用于根据已知的对数正态分数 A1 计算等效稀有性正态分布 IQ 分数:
    =NORMINV(1-(1-(LOGNORMDIST(A1;LN(100)-0.5*LN((15/100)^2+1);(LN((15/100)^2+1))^0.5;1)));100;15)

    通过研究转换后的对数正态智商分数表,我发现了一些令人难忘的规律:
    *对数正态和正态分布 IQ 分数等于 2.5 – 76 之间的 +/- 130 分以内,以及 1 – 82 之间的 +/- 122 分以内 (在对数正态尺度上)。

    *智商高于 116 时,对数正态(真实)智力得分始终高于人群中相同稀有度的正常智商得分.

    *对数正态(真实)分数与正常 IQ 分数之间的差异为 到一个很好的近似:
    智商 2 时 125 分 在正常范围内,
    5 于 135,
    10 于 145,
    15
    (16) 在155
    20在160.
    -这给出了一个容易记住的经验法则。

    举个例子:正态分布 4 西格玛任何东西,例如 160 的智商,大约是三万分之一。事实上,根据智力的对数正态分布,我们预计会发现大约 1 人中就有 30,000 人的得分高出 20 分,即 180 分。正态分布将预测分数为 1 的人数的千分之一,即 30,000M 中 180 人(或 1 分 20M 中)。

    当正常智商分数较高时,等稀度对数正态分数与正常分数之间的差距继续扩大:
    25 时 166(1 万分之一),
    30 于 172(1 百万分之一),
    35 于 176(1 百万分之一),
    40 at 181 (1 in 30M) )

    • 回复: @EH
    , @AnotherDad
    , @Harold
  9. EH 说:
    @EH

    后续: 点击此处 前 Mega Society(智商百万分之一)官员和 NASA 工程师 Bob Seitz 讨论了对数正态分布,并将其预测与 Terman 著名研究的观察结果进行了比较。

    “我相信这就是我的想法,应该使用一个特定的对数正态标准差,但它是 15,”Seitz 说。

    • 回复: @res
    , @James Thompson
  10. @res

    感谢这些。我记得一连串的修正,并且我同意它对大脑大小的真正性别差异给出了最​​保守的估计。不过,我仍然对其合理性存有疑虑,但我无法进一步推进我的论点。

  11. EH 说:
    @James Thompson

    我认为对身体尺寸的校正不应该是体重,甚至是体积,而应该是体表面积(它应该与神经末梢数量更密切相关),可能还可以对男女之间神经末梢表面密度的任何差异进行额外校正。女性。同样,相关的大脑大小/身体大小比例应该是:皮质面积/皮肤面积,或更准确地说是皮质神经元/身体神经末梢。

    • 回复: @res
  12. Reg Cæsar 说:

    读者文摘 早在 1970 世纪 XNUMX 年代,就有报道称,一项调查/研究表明,男女双方都对女性更高、更年长、更富有以及许多其他与刻板印象相反的特征的婚姻感到非常满意。

    唯一的例外是,男女对于女性是更聪明的伴侣的婚姻都抱有深深的悲观态度。

    • 回复: @curt dunkel
  13. res 说:
    @EH

    Ankney (1992)(上面链接)着眼于大脑质量与身高和体表面积的关系。图中有明显的趋势,但所有相关性都约为 0.2,所以没有那么大。图中的一件好事是他指出了男性和女性的平均值,这有助于估计性别差异与平均身体差异的比较。

    他发现体型矫正的效果比我上面估计的要小:

    因此,对于平均身高的个体来说,校正身体尺寸仅使男性和女性脑质量的绝对差异减少约25%,即从约135克减少到约100克。

    以下是他的总体结论:

    总而言之,如果人类的智力确实与大脑大小存在因果关系,那么我们就面临着一个难题,即女性的大脑比男性相对较小,但两性在一般智力上显然没有差异。我为此提出了四种解决方案:
    1. 一些与体型差异相关的未知影响。
    2. 智商测试偏向女性。
    3、女性的大脑比男性更有效率。
    4. 相对大脑大小的性别差异与男性擅长的智力能力有关。
    也许天真地,我忽视了前两个,而作为一个“适应主义者”,我对第三个不屑一顾。因此我赞成第四种。不管具体的解释如何,对男性和女性大脑差异方式的进一步研究可以提供对我们所说的智力的起源的理解。

  14. res 说:
    @EH

    一个问题是特曼研究使用了比率智商。我认为非正常观察是智商偏差的部分推动力。我似乎记得有人看到智商偏差存在非正态性,但手头没有参考资料。

    https://en.wikipedia.org/wiki/IQ_classification 给出了哪些测试使用比率/偏差智商的良好列表。

    PS 我看到他们在您的链接中讨论了比率和偏差智商,但我没有看到关于当前偏差智商的真实分布的明确结论。

    • 回复: @EH
  15. @Reg Cæsar

    雷格,您有更多信息吗?我对这种可能性很好奇。谢谢。

  16. @AP

    林恩在他的论文中考虑并拒绝了神经元密度论点。没有证据证明这一点。然而,额外的男性大脑可能解释了更好的空间能力。

  17. @EH

    谢谢。我觉得这个话题很切题,也很有趣。无论如何,实际分布常常偏离正态分布,要么是因为 20 世纪初格拉斯哥样本中的不利生活条件/疾病,要么是 1960 世纪 XNUMX 年代怀特岛研究中的理所当然。衡量高智商是有问题的,因为标准方法变得紧张。人们倾向于转向其他聪明人给出的排名。例如,费米关于冯诺依曼的评论。我认为所有这些都与“玻璃天花板”类型的论点有关,也与使用 La Griffe du Lion 的技术来计算群体平均智力(从中抽取杰出成就者)有关。

  18. EH 说:
    @res

    根据定义,偏差智商是正态分布的,但在尾部,它们不是等间隔量表,量表上的标记实际上间隔更宽,每个偏差智商点在 160 左右代表智力差异比接近 100 的智商点更大。

    年龄比例尺度对我来说不仅令人困惑,而且有点可疑,因为:
    *许多人认为它们与现代测试中使用的偏差分数相同或至少密切相关 - 我对此表示怀疑
    *许多人认为他们随着年龄的增长而稳定——他们不适合高分者,因为年龄相当的量表在 16 岁左右达到顶峰
    * 年龄比率分数很容易与比率量表分数(具有真零的等间隔量表,例如斯坦福比奈 (SB5) 上的 Rasch CSS 量表)混淆。
    * 人们隐含地认为智力随着年龄的增长呈线性增长,但实际上,当用 CSS 等量表进行测量时,智力会随着年龄的对数而增长。

    一个表明肥尾分布的花絮:根据 SB5 在高能力评估中的应用 Deborah Ruf 发布的 Riverside Publishing 服务公告显示,在 SB5 规范中,FSIQ CSS 的最高分数为 592,成人平均分数为 510-515。 (第 9-10 页)。关于块旋转 子测验 (幻灯片 19),至少,成人 sd 大约比 +8.5 sigma 范围高出 3 CSS 分,这意味着 592 分数大约是 9 个标准差,即使 FSIQ sd 更大,这仍然是相当于智商超过 200,预期稀有度从至少数百亿分之一到基本上“从未”。

    [稍后如果我有时间,Thompson 博士关于 La Griffe du Lion 的阈值方法如何受 LN 分布影响的观点很重要]

    • 回复: @res
  19. dearieme 说:
    @James Thompson

    “人们倾向于转向其他聪明人给出的排名。”认为梅纳德·凯恩斯是一个非常聪明的人的最好理由之一是,在剑桥充满了非常聪明的物理学家中,他被认为是一个非常聪明的人。

    • 回复: @James Thompson
  20. @dearieme

    是的。拉塞尔说,在与凯恩斯辩论过任何事情后,他的头很痛。

  21. Wency 说:

    在所有可能取得成就的领域中,为什么要关注国际象棋特级大师?比如说菲尔兹奖获得者呢?

    虽然特级大师不是傻瓜,但众所周知,他们的成功更多地与练习和学习有关,而不是拥有特别高的智商。智商 130 和对国际象棋不懈的痴迷会让你比智商 200 和对游戏有较高兴趣的人走得更远。

    我认为这种针对游戏的偏执在男性中比女性更常见,无论智商如何。那些痴迷于谈论体育或电子游戏的男性可能也存在同样的倾向。虽然女性有时会讨论这些事情,但我从未听过一个女性——而且我认识一些非常聪明和善于分析的女性——对体育、电子游戏或棋盘游戏进行了极其详细的分析。我认识很多有这种怪癖的男人(我就是其中之一)。

    否则是一篇好文章,但我认为比较国际象棋大师的数量是相当薄弱的证据。即使男性和女性的智商分布相同,我预计该比率也会高度倾斜。

    • 回复: @James Thompson
    , @res
  22. hyperbola 说:

    啊,又是进化心理学家!!!显然,他们的分歧和解释的源泉是取之不尽用之不竭的。这篇文章,加上这篇补充文章,让我相信尼安德特人可能比心理学家更聪明。

    尼安德特人的大脑比现代人更大——为什么我们更聪明?
    http://science.howstuffworks.com/life/inside-the-mind/human-brain/neanderthal-bigger-brains-humans.htm

    • 回复: @res
  23. res 说:
    @EH

    感谢您提供非常有用的评论! SB5 链接是我在高智商测试中见过的最好的参考之一。如果其他人对您的幻灯片链接感兴趣,请参阅以下博客文章,讨论整个系列: http://www.iqscorner.com/search/label/Rasch
    博客上的应用心理测量学 101 报告和简报看起来也很有趣。

    根据定义,偏差智商是正态分布的,但在尾部,它们不是等间隔量表,量表上的标记实际上间隔更宽,每个偏差智商点在 160 左右代表智力差异比接近 100 的智商点更大。

    考虑到对极其罕见的情况进行标准化测试的难度,我一直不确定这个定义与现实(分数)的吻合程度如何。你的第二句话很有趣。你能推荐一些讨论智商偏差与“智力”之间关系的线性关系的参考文献吗?下尾部是否也有类似的效果?有人可以访问 SB5 CSS 到 IQ/EXIQ 的转换信息吗?

    关于年龄比例的想法困扰我的另一件事是不同的发展速度问题。有没有人尝试在情报背景下分析/纠正/量化/测量这一点?大脑发育通常与其他身体发育平行吗?

    人们隐含地认为智力会随着年龄的增长而线性增加,但实际上,当用 CSS 等尺度来衡量时,它会随着年龄的对数而增加。

    这个想法对我来说是新的,但一旦我看到它就很有意义。您能推荐一些讨论此问题的参考资料吗?

    PS我希望你稍后有时间; ) 再次感谢!

    PPS 当我搜索相关主题时,我偶然发现了这篇涵盖各种 SB5 复合材料的论文,并认为其他人可能会感兴趣: http://www.hmhco.com/~/media/sites/home/hmh-assessments/clinical/stanford-binet/pdf/sb5_asb_4.pdf
    这篇论文带有 SB5 IQ 分数的 QQ 图: https://www.ncbi.nlm.nih.gov/pmc/articles/PMC4613563/
    有谁知道 SB5 CSS 的分布图吗?

  24. res 说:
    @Wency

    我同意你的观点,但国际象棋的一些优点是数据可用性和将技能降低到可以随着时间的推移进行检查的数字分数。

  25. dearieme 说:

    那很有意思。在我的屏幕上,Wency 的评论 22 Doc T 的评论编号为 3,然后双曲线的编号为 4。按照惯例,已经有评论 3 和 4 位于 2 和 5 之间。

    编辑:现在它已自行更正,只是该评论已编号为 5。大概当我单击“保存”时,编号将变为 25。Unz 以神秘的方式移动。

  26. res 说:
    @hyperbola

    对于鲸鱼来说也是如此。显然,尼安德特人在生理上与现代人类更加相似,但很明显,简单的大脑尺寸度量在单个物种/亚种内比在物种/亚种之间更相关。

    在某些时候,了解尼安德特人和人类遗传学在智力相关领域的差异将会很有趣。特别是如果我们能够区分大小和其他功能差异。

    • 回复: @hyperbola
  27. Anonymous [又名“新帐户”] 说:

    到目前为止,我只读过林恩的论文(感谢汤普森博士提供免费副本!)并且不确定他的一些引文,因为我之前读过相关论文。

    例如,Lynn 报告称,在 0.57-19 岁的参与者中,WJ-III 上的男性有 79SD 的差异。然而,在实际论文(Camarata & Woodcock 2006)中,报告的差异在 3.3-19 岁年龄组中为 34 分,在 3.1-35 岁年龄组中为-49 分,在 2.6-50 岁年龄组中为 79 分。我不确定 Lynn 是如何报告如此大的差异的,因为未加权均值对男性的影响只有 0.06SD 左右。

    https://www.researchgate.net/publication/222702039_Sex_differences_in_processing_speed_Developmental_effects_in_males_and_females

    同样,Lynn 在 van der Sluis 等人 0.3 年的论文中报告了 2006 SD(以 g 为单位)的差异,该论文特别指出

    >男性和女性在特定认知方面确实存在差异
    能力,但 g 不能被视为这些差异的根源。

    http://www.sciencedirect.com/science/article/pii/S0160289605000851?via%3Dihub

    我再次不确定 Lynn 的 0.3SD 数字从何而来。还有几个这样的例子。

    这不应该成为贬低他在这个主题上的研究的廉价借口。我只是好奇他是否在这里犯了一些错误或者意识到了一些我不知道的事情。我希望有更熟悉这项研究的人参与进来。

  28. Anon • 免责声明 说:

    坦率地说,拥有 4 亿个额外的神经元,却只能从中获得 4 个以上的 IQ 点,这是一个非常令人失望的性能提升。因此,男性大脑中的神经元数量比女性多 16%。然而,根据拉什顿的说法,黑人的大脑比白人小 6%,而且智商的差异是一个完整的标准差。至少,黑人男性的大脑应该接近白人女性的大小,但他们却比白人女性愚蠢得多。神经元只是故事的一部分。

    • 回复: @AP
  29. AP 说:
    @Anon

    这可以通过詹森的推测来解释,即男性和女性大脑大小的差异很大程度上是由于男性大脑有更多的神经元,但这些额外的神经元专门用于空间能力(例如告诉一个人在哪里),而这些能力不被认为是成为“智能”的一部分。

  30. pyrrhus 说:

    过去 60 年来数以千万计的 SAT 成绩(参见以下网址的数据) https://www.collegeboard.org),表明男性在语言技能方面略胜一筹,而在数学方面则要好得多(平均 40 分)。结果显示,男性智商高出 4-4 1/2 分。

  31. utu 说:

    男孩和女孩在15岁之前的智商大致相同,但从16岁开始,男性的平均智商变得高于女性,成年后优势增加到约4个智商点。

    我们也经常在这些页面上听到,智商在人的一生中非常稳定,但现在我们被告知男孩在 4 岁之后会积累 15 分。

    • 回复: @reiner Tor
  32. dux.ie 说:

    智商分布曲线的形状取决于人口统计数据和影响人口统计的因素数量。

    种群很少是统一的或协调良好的(没有选型交配)。因此,原始分布数据通常不是钟形的。如果包含足够的 IQ 分量或调整相应的权重因子,则可以产生钟形分布。例如,ACT 综合分数分布呈钟形,但英语和数学各自的子成分是多模态的,阅读和科学更接近钟形,因为它们可能更容易受到勤奋和预科课程的影响。英语部分可能会受到母语的影响,而数学部分可能会因为先天的流体智力而受到更多影响。

    最近有人指出,种族群体的原始 ACT 数据可以在网络上找到,但似乎已经消失了。对于那些拥有它们副本的人来说,可以验证黑人和西班牙裔的单独分布大多是双峰的,而亚洲人的分布是三峰的。关于选型交配可能会产生超级聪明的孩子的讨论很多,许多人都试图找到证据。然而,很多像我一样的人一直在错误的地方寻找。可能存在选型交配,但只有那些处于较低认知范围的人才会有很多孩子,才会出现在数据中。因此,默里关于人口分裂的说法是正确的,但较低的认知区域正在明显下降。

    与人口统计的影响相比,正态和对数正态之间的差异微不足道。

    2013年亚洲数学
    ACT IQLike|频率
    12 72.0| 114
    13 74.8|## 392
    14 77.6|###### 1043
    15 80.5|############### 2558
    16 83.3|######################## 3762
    17 86.1|######################## 3615
    18 89.0|################ 2604
    19 91.8|############### 2500
    20 94.6|############ 1878
    21 97.5|############### 2432
    22 100.3|################## 2795
    23 103.1|####################### 3680
    24 105.9|############################ 4469
    25 108.8|############################## 4757
    26 111.6|################################## 5370
    27 114.4|############################## 4795
    28 117.3|############################ 4332
    29 120.1|################### 3190
    30 122.9|################## 2773
    31 125.8|############ 1962
    32 128.6|############## 2299
    33 131.4|############## 2231
    34 134.2|###################### 3533
    35 137.1|################ 2614
    36 139.9|############ 1915

  33. Wally 说: • 您的网站

    “一名谷歌男性员工在他的工作场所实践中回顾了一些有关该主题的文献,然后被解雇了。”

    另请参阅此处的其他危险科学:
    https://www.unz.com/article/books-banned-by-banned-books-week/

    立即购买:

    看到:
    Goolag T 恤讲述故事
    https://forum.codoh.com/viewtopic.php?f=2&t=11320

  34. Tom Welsh 说:

    这不是很像争论针尖上能站多少个天使吗?当然,4 个 IQ 点在现实世界中并不构成任何显着差异。尤其是因为纯粹的智商被其他重要的心理和情感品质所掩盖。例如

    “世界上没有任何东西可以代替持久性。 人才不会; 没有什么比有才华的男人更普遍的了。 天才不会; 天才无知几乎是一句谚语。 单靠教育不会; 这个世界充满了受过良好教育的遗弃。 坚持和决心是无所不能的”。

    –卡尔文·柯立芝(Calvin Coolidge)

    我很惊讶自己写下这些话,因为过去 60 年来我一直非常倾向于捍卫智商的价值。

  35. 引用的文章来自 人类季刊卷。 58、1号 从 2017 年秋季开始,如果正确(= 总体上科学合理),可能会一度被称为世界新闻,至少在非 PC 世界的某些地方,不是吗?

  36. Santoculto 说:
    @res

    趋同(((心理)))认知进化/适应。

    又是同样的话题……

    或者汤姆先生很天真,认为(((左派)))撒谎或善意地误解了这个问题,或者他试图让观众开心(并且对这个[[[政治问题]]]的了解比实际展示的更多)。

  37. dearieme 说:
    @Tom Welsh

    我捍卫智商的价值,但只是按比例捍卫。如果说那些痴迷于智商的人和那些否认智商的人一样疯狂或坏蛋,那是不公平的,但他们似乎确实是蒙蔽了双眼。

    我喜欢 Doc T 博客的一个原因是,他确实表现出了一种分寸感(加上良好的写作风格、高度的耐心和礼貌,以及许多其他你期望(或至少希望)在老博客中看到的优点)。 ——时尚的英国学者)。

  38. szopen 说:
    @Tom Welsh

    ” 当然,4 个智商点在现实世界中并不构成任何显着差异。 ”

    假设标准差相等,女性为 98,男性为 102,正态分布(无厚尾)。

    在 130 的门槛上,将有接近 2/3 的男性(65%)和 1/3 的女性。
    145岁以下时,男性比例将上升至69%
    160岁时,男性比例将上升至75%

    现在,SD 的差异仅为 0.5 个点,在 IQ 130 时为 69%,在 77 时为 145%

    IMO 这在现实世界中存在显着差异(特别是因为 SD 差异似乎高于 0.5 点)

    • 同意: Dieter Kief
  39. reiner Tor 说:
    @utu

    智商在人的一生中非常稳定

    成年后发病。

    • 回复: @CanSpeccy
  40. @Tom Welsh

    我在柯立芝阵营。

    男子提前4分?

    哈欠……尽管它有这么多好处,但也可能领先 40 分。原因是女人知道男人是被他们的球而不是他们的大脑统治的。

    寓意:痴迷于智商的人还不如继续自慰。

    • 回复: @dcite
    , @Anon
  41. Joe Hide 说:

    致詹姆斯·汤普森:
    尽管数据量很大,但可读性很高。这是好东西!很棒的写作风格。我想要更多!

  42. hyperbola 说:
    @res

    我收录尼安德特人文章的原因之一是对尼安德特人智商的大量完全未经证实的猜测,更不用说对什么可能导致大脑更大但智商更低的疯狂猜测了。您究竟建议如何衡量尼安德特人的“智商”以进行比较?我想原则上有一天我们可能能够追溯克隆一个“尼安德特人”(没有证据证明我们已经成功——例如我们知道胎儿受到妊娠的严重影响),但是你建议如何重建培育(环境) )尼安德特人儿童发展的组成部分?所有这些猜测纯属垃圾。

  43. 林恩有力地证明了男性在智力上的优势

    据测量,就是这样。请记住,WAIS 的很大一部分是针对分析能力的。

    批评家现在必须让他了解他提出的详细结果。

    嗯,“必须参与”?并不真地。 4分?在现实世界中微不足道。此外,谁在乎呢?如果女性能够承担货物,她就可以做她想做的所有 STEM。

  44. 注:封面为围棋棋手悬坠,由 19日 世纪大师小笠原一斋。为您或您的附属图书馆订阅:

    该版本本身说:

    封面图片:围棋棋手的微型象牙雕刻(或悬坠) 18日 日本世纪雕刻大师小笠原一斋。参见王芒瑞的文章第142页

    • 回复: @James Thompson
  45. reiner Tor 说:
    @John Jeremiah Smith

    此外,谁在乎呢?如果女性能够承担货物,她就可以做她想做的所有 STEM。

    你来自哪个星球?在我们这个星球上,主流意识形态认为,在取得任何成就的人(例如数学学生)中,女性所占比例低于 50%,这就是对女性歧视的有力证据。阻止这种疯狂行为非常重要。

    • 回复: @Avery
    , @John Jeremiah Smith
  46. 别介意智商。

    更大的问题。

    曾经被称为“荡妇”或被告知你是“自找的”吗? Slutwalk 不会容忍这种情况。警告:工作不安全。

    发布者 奥兹 十月十日6,2017

  47. 这几乎支持了这样的观点:男性用逻辑方式争论,而女性则使用情感虐待。

    也就是说,如果您以逻辑方式处理数据。否则,这就是性别歧视、种族主义、恐同、可耻、充满负罪感的仇恨言论。

  48. Avery 说:
    @reiner Tor

    { 在有任何成就的人(如数学学生)中,女性所占比例低于 50%,这是对女性歧视的积极证据。阻止这种疯狂行为非常重要。}

    说得好。

    不仅如此,百分比还被用来在法律上歧视男性(……至少在西方)。
    因此,是的,停止这种疯狂行为并揭穿硬科学领域缺乏女性代表的虚假叙述是歧视的证据是非常重要的。这反过来又被用来在法律上歧视男性。

    • 同意: Dieter Kief
  49. @reiner Tor

    你来自哪个星球?

    地球,这一次。当你对……嗯,歧视理论感到感动时,请随意表达怨恨。坦白说,我根本不在乎这个。如果这位女士能比其他申请者更好地完成这份工作,她就会得到这份工作。

    • 回复: @Peripatetic commenter
  50. joe webb 说:

    我在某处读到,智商达到 120 后,白人男性将白人女性远远甩在身后,而智商 170 时,男性数量是女性的 30 倍。因此,玻璃天花板只存在于女士们的眼中,以及她们的情感大脑中。

    或多或少是真实还是不真实?

    在我一生的不经意观察中,我发现,尽管女性的智力更高,但她的情绪对男性的主导作用却远远大于男性。

    乔·韦伯

    • 回复: @John Jeremiah Smith
  51. @Anonymous

    林教授回复:

    在 Camarata 和 Woodcock 2006 年的论文中,19-79 岁年龄段的男性在流体推理 (GF) 方面拥有 2.21 IQ 点的优势,在一般智力能力方面拥有 07 IQ 点的优势。

  52. @joe webb

    在我一生的不经意观察中,我发现,尽管女性的智力更高,但她的情绪对男性的主导作用却远远大于男性。

    我注意到,对女性越好,尤其是如果有人评论她的行为有多聪明(永远不要说“哎呀,你很聪明!”),她对我就越好。

    但是,这就是我——一个普通的老傻瓜,不侮辱、不贬低、不虐待女性。愚蠢的我。我知道你们所有的聪明人都会扇他们耳光并告诉他们他们是愚蠢的。天哪,我真羡慕你们这些有男子气概的男生。

  53. @James Thompson

    在 Camarata 和 Woodcock 2006 年的论文中,19-79 岁年龄段的男性在流体推理 (GF) 方面拥有 2.21 IQ 点的优势,在一般智力能力方面拥有 07 IQ 点的优势。

    上帝之母,多么执着、尖刻的白痴啊。一个人参加两次或两次以上智商测试的平均方差为 3 分。或者说,已经很多年了。既然智商测试已经被明确证明可以绝对准确地测量智力,我确信这种情况不会发生。

    我在韦克斯勒上见过 8 个点的个体差异。

    这种性别智商差异是强迫症狂人要担心的。那和阉割焦虑。

    • 同意: utu
    • 回复: @utu
  54. Anonymous • 免责声明 说:
    @James Thompson

    肯定没有。良好的医学诊断主要需要两件事:良好的知识/记忆和良好的模式识别。男性在前者方面稍好一些,而在后者方面则明显更好。

  55. @John Jeremiah Smith

    如果一种药物的疗效提高了 4%,就值得研究。宣称对知识的探索在某种程度上是错误的,这是荒谬的。

    • 回复: @John Jeremiah Smith
  56. utu 说:
    @John Jeremiah Smith

    一个人参加两次或两次以上智商测试的平均方差为 3 分。

    如果像经常报告的那样,重测相关性为 0.9 或 0.95,则对于 SD=6.7 的人群,重测差异的 SD 分别为 4.7 或 15 IQ 点。

    • 回复: @John Jeremiah Smith
  57. @reiner Tor

    成年后发病。

    这种情况在女孩中比男孩发生得更早,而在某些个体中发生得比其他人更早,无论性别如何,这使得测量发育速度差异很大的儿童的智商变得毫无意义,这种变化部分可能是由于饮食和其他环境因素造成的。

    • 回复: @utu
    , @Santoculto
  58. @James Thompson

    您是否知道有任何关于相当明显问题的研究,即具有 1Q 190 所暗示的可能高处理速度(一位年轻科学家朋友在提到他的同行时提到的六西格码)的人在他的计算机中不会具有同等的速度优势?接口设备?

  59. utu 说:
    @CanSpeccy

    我自己也想知道他是什么意思。当然,它没有解决我提出的问题。

  60. @Anonymous

    到目前为止,我只读了吉姆·弗林教授的回复,并想起了林恩关于种族差异的著作中令人恼火的缺陷之一。这显然是一种不加区别地选择数据作为他的推测和计算基础的方法。正是他对原始人平均 63 及以下的智商分数的天真接受和使用,让我对他的工作产生了质疑(在罗恩·乌兹对他进行了猛烈的打击之前),我不确定他是否有所进步。

    • 回复: @James Thompson
  61. AnotherDad 说:
    @EH

    对数正态分布——分布实际上是什么样子的问题是一个有趣的问题。我同意你的观点,尾部确实比正态分布更肥。

    但你的智商数字并不那么有趣。智商并不是像“身高”那样客观、真实的衡量标准。

    智商分数正是基于正态分布、特定均值和标准差,某人相对于其他人的排名。前任。 145 的智商恰恰意味着——在这个测试中——“嘿,你比随机的白人集合中 99.8%(无论 3SD 数字是什么)更聪明”,除此之外别无其他。底层分布实际上是什么并不重要。这就是它的意思。

    这并不意味着您比普通人聪明 45%。在典型的智商测试中(即相当容易处理的问题),145 个人几乎所有问题都答对了,而普通人无法解决很多问题或答错。我们都知道,在其他条件相同的情况下,145 个人在大多数类型的任务\工作中会更有价值。根据认知需求,145 人的表现可能从“好一点”到“相当于三个普通人”,再到(对于最苛刻的心理过程)本质上无限好。

    您可以将 145 重命名为 165,以尝试强调这样的人确实比普通熊聪明得多……但这并不能给您带来太多好处。它仍然不会是一个线性尺度,每个点都具有相同的额外“智能”。而且你的新 165 智商仍然无法公平地衡量这个人比其他普通人好多少。智商根本不是像身高那样的衡量标准,而且也没有办法使其成为一个衡量标准。

    • 回复: @Wizard of Oz
  62. @Daniel Chieh

    如果一种药物的疗效提高了 4%,就值得研究。宣称对知识的探索在某种程度上是错误的,这是荒谬的。

    首先:“声明”不是一个词。假设你的意思是“声明”,我不会对你如何从你的屁股中拉出“寻求知识”进行尖锐反驳。

    宣称,甚至宣称,男性和女性之间 0.07 的智商点差异有任何意义,充其量是可笑的。

    • 回复: @Daniel Chieh
  63. @utu

    如果像经常报告的那样,重测相关性为 0.9 或 0.95,则对于 SD=6.7 的人群,重测差异的 SD 分别为 4.7 或 15 IQ 点。

    我利用美国国立卫生研究院 (NIH) 的资助为一名智商研究员工作了一年。该研究人员也是马里兰州罗克维尔切斯特纳特洛奇医院的住院精神科医生。 (自关闭以来)

    合法的智商测试,例如韦克斯勒测试,是由经过认证的心理学家以一对一访谈的形式进行的。是的,这是有原因的。测试包被密封,随后由管理员打开,并签字确认测试已密封。打包的测试并不便宜。试验材料完成后销毁。测试管理员验证结果并在记录中记录测试包的标识号。

    有一系列的成就测试被智商狂人称为“智商”测试,这些测试大部分都是基于这些虚假的智商统计数据,由塞勒和里奇温等人推广。这些测试不是智商测试。

    这并不是说成绩测试有什么问题——它们很好地达到了目的。然而,这个目的并不是为了测量智力。成绩测试可以有效评估学习或培训,而不是评估智力本身。

    还有相当有用的能力倾向测试组合。这些也很好地达到了目的,并且对于潜在能力和发展技能的实际评估比任何“智商”测试或成就测试更有用。

    • 回复: @Wizard of Oz
    , @utu
  64. @AnotherDad

    您对 145+ IQ 评分者可能具有更大效用的观察使得这也许是一个合适的地方来指出假装让 40% 以上的人接受大学教育所造成的损害。在弗林对林恩的回复中,我惊讶甚至震惊地发现,男性进入大学的分界点为 100,女性为 95,相应的大学生平均智商约为 111。令人惊讶的是,现在有一大群大学教师——全部都是软学科——智力中等,他们与愚钝的学生结合在一起,让自己相信自己是知识分子,他们对更广泛社会重要的任何事情的想法都是有价值的,并且是有价值的。值得坚持让别人认真对待吗?

    当语法、拼写和理解力出现问题时,他们会大喊大叫,最多只能向他们挥舞标语牌,他们听不懂但认为可能在说一些令人讨厌的聪明话。

    • 同意: Johann Ricke
  65. Harold 说:
    @EH

    “实际”分布无法通过任何干净的数学函数很好地建模到尾部足够远的地方。

  66. @James Thompson

    根据我对评估极高智商所提供的性能优势时未测量的界面因素的想法和表达,我认为这是一个相关的想法,我愿意用金钱来支持。

    与成功地对计算机进行编程以下国际象棋或围棋一致,显然可以对它们进行编程以进行全套智商测试。但这并不是我建议的那么简单的程序。我们需要的是一个硬件+软件,尝试模拟人脑的过程,然后释放一系列智商测试。

    即使使用模型 1.1.1,也可以开始进行一些调整,看看这些变化对不同 g 权重的测试有何影响。

    也许应该从全软件模型开始,不要使用非标准神经系统模拟部件作为概念证明。

    我可以为这样的项目找到资金。给我一个你自己的地址,或者让罗恩帮你联系我。

    它是否存在理论上的致命缺陷或者可能有用?

    • 回复: @James Thompson
    , @res
  67. @Wizard of Oz

    同意我们应该推动最高标准的报告。

    在林恩数据的贝克尔版本中,所有结果都是可追溯的,弗林修正是基于最新的荟萃分析。样本量和代表性的进一步修正将在适当的时候进行,但任何人都可以在当前数据库上进行前者。这将给出加权估计,而不是中位数。尼日利亚还有一项新的重大研究,样本量大,公立学校教育代表性好,得出的智商平均值为70,我认为是目前最好的估计。

    对于种族差异报告中的另一个令人恼火的缺陷,这次是关于尼斯贝特、阿伦森、布莱尔、狄更斯、弗林、哈尔彭和特克海默,请参阅:

    https://www.unz.com/jthompson/on-best-understanding-nisbett-and-co

    顺便说一句,两个错误并不等于一个正确,它们只是两个错误太多了。

    • 回复: @Wizard of Oz
  68. Anonymous [又名“此帐户”] 说:
    @James Thompson

    感谢您向林恩提出我的问题。不幸的是,他没有解释为什么他在评论的表 0.57 中报告了 Camarata & Woodcock 论文的 d = 7 值。我认为这和类似的东西当时只是错误

    • 回复: @James Thompson
  69. @James Thompson

    无论是从意识形态上还是作为一个批判性的业余爱好者,我都不会相信一项对尼日利亚人进行的大规模研究在测试中得出的智商平均值为 70,而非洲裔美国人的智商平均值为 85,西北欧人的智商平均值为 100。我愿意采取预防措施,不会不加区别地向智商如此平均的人开放我国的永久移民计划,无论出于什么原因。但…..

    否则我发现 70 这个数字缺乏它所传达的信息。一两代人内可能克服的寄生虫和疾病负担是多少?
    更重要的是:就弗林效应函数或曲线而言,非洲的那部分在哪里。如果弗林效应每 3 年增加 10 个百分点,难道不能合理地预期尼日利亚的平均得分可能在 90-70 年内达到 75 分吗?这与以色列相差不远!简而言之,这不是一个篮子,也不是随机选择的移民的第一选择。

    一个相关的问题是,有证据表明,弗林效应在假定的文化博览会“乌鸦矩阵”中得到了强有力的证明,并且与各种词汇和其他语言和数学测试相比,乌鸦矩阵是流体智力最令人高兴的衡量标准。怎么会?

    答案与我(我相信是真的)乌鸦队的测试很大程度上受到训练/练习效果的观察有什么关系吗?这方面一定有数据。我想知道乌鸦队的测试在什么时间段内进行了多少次(哦,太无聊了)测试或练习会产生明显更高的分数? (这与弗林效应的关系在哪里?我认为在某个阶段,它是其中的一部分,因为不成熟的人首先变得适度地测试复杂,然后习惯于学校的测试 - 和抽象的几何图形 - 这提供了一些准备对于乌鸦队来说)。

    • 回复: @James Thompson
  70. @John Jeremiah Smith

    您似乎忽略了 g 的概念。你到底懂不懂呢?并具体了解“g 负载”的概念,它肯定是在成就和能力倾向测试以及乌鸦矩阵等测试中使用 和 的相关性来产生智商估计值的基础?

    请举例说明您认为 Steve Sailer 宣扬“博瓜智商统计垃圾”的例子。鉴于他对事实和正确统计数据的关心,如果你能做的不仅仅是找到一个罕见的几乎看不见的小虫子来挑选,我会感到惊讶。

    • 回复: @John Jeremiah Smith
  71. @Wizard of Oz

    您似乎忽略了 g 的概念。你到底懂不懂呢?

    我明白g吗?不,当然不。像我这样没有受过教育、无知、被社会排斥的人怎么可能 或者 理解一个如此复杂、如此纯粹的概念,如g?

    欢迎你们这些家伙和你们对智商的痴迷在你们能找到的每一个别针头上跳舞。他妈的把自己打垮了。当有一天,宇宙中任何地方的任何掌权者都让黑鬼和小鸡们变得愚蠢时,你就会胜利。

    • 巨魔: Daniel Chieh
  72. @John Jeremiah Smith

    就判断而言,这与“意义”无关。这是关于根据发现的违规行为进一步开展调查。这同样适用于研究非常无用的事物,例如行星摆动等。

    John J Ratey 博士在 SPARK 中研究了智力和心脏健康(相关,可能是由于血流改善)或 Whalley LJ 等人对儿童 DHA/Omega-3 摄入量和心脏健康的影响,也存在相当小的差异。后来的智力(可能是由于神经保护特性)。对这些进行调查在道德上也不是错误的。哎呀,即使铅中毒在一些研究中也只显示出 2 智商的缺陷(Lanphear 等人),但这并不意味着研究它毫无意义。

    感觉你对这件事的情绪化程度超过了必要的程度,而且这可能是个人原因。祝你生活愉快。

    • 回复: @John Jeremiah Smith
  73. @John Jeremiah Smith

    地球,这一次。当你对……嗯,歧视理论感到感动时,请随意表达怨恨。坦白说,我根本不在乎这个。 如果这位女士能比其他申请者更好地完成这项工作,她就会得到它.

    当然,你掩盖了一个重要的问题:你如何确定她是否能胜任这份工作?

    还有一个附属问题:在实践中,是否有任何证据表明女性比男性做得更好?

    将我的注意力限制在需要高智商的科技领域工作,它是如何运作的?

    在科技领域,对申请人进行智商测试往往是不切实际的,因此我们依靠面试,有些面试可能非常艰苦,以及参考资料,看看他们以前做过什么,等等。

    然后我们向他们提供这份工作,希望他们接受。

    然后他们开始工作,我们看看他们是否真的有能力。

    当然,问题在于,如果我们选择了一名女性候选人,而她表现不佳,那么解雇她比我们在男性候选人上犯了错误要困难得多。

    将我的个人经历纳入考虑范围,我之前和现在都曾与科技领域的女性一起工作过。我见过的超级明星很少,他们往往拥有与男性不同的技能。与男性相比,她们通常更注重细节,但做事的决心却远不如男性,也更不愿意进入未知领域。在软件维护中,第一个是资产,但在软件开发中,另外两个是重要资产。我怀疑其他科技领域也是如此。

    我从未在招聘委员会中考察过女性候选人,因此我无法讨论这些问题是否会出现在面试评估中。

    然而,我的观点是 在候选人阶段,你根本不知道她是否会比男性候选人做得更好 做出错误的选择会带来很多负面影响。因此,我们使用启发法,例如性别的平均智商、男性和女性智商差异的已知差异以及完成工作的简单血腥决心。

    而且因为我们可以比女性候选人更容易地解雇男性候选人,并且成本更低......

    • 回复: @John Jeremiah Smith
  74. @Daniel Chieh

    就判断而言,这与“意义”无关。这是关于根据发现的违规行为进一步开展调查。这同样适用于研究非常无用的事物,例如行星摆动等。

    是的,我知道。我确实有清醒的时刻,丹尼尔。当然,我所处的浓重情感迷雾模糊了焦点,掩盖了真正的意义,但十字架必须背负着,不是吗?

    告诉我,为什么种族/性别智商比较的研究最经常且可以预见地出现在专门用于政治言论的场所?显然,肯定是这样,因为那些自认为是专家的人最有资格评论科学研究——对吧,丹尼尔?并为这种崇高的努力做出贡献,以绝对量化现代世界如此重要的问题。还有什么比男女平均智商 0.07 分的差异更重要的呢?让人不禁激动得发抖,nicht wahr?

    我真的必须赞扬我们政府中的伟大人物,他们坚持不懈地努力减少因智商表现不佳而流向黑人和墨西哥人的税款。此外,我们以绩效为基础的经济模式确实令人钦佩,因为明显处于劣势(而且情绪低落)的女性将被剥夺平等的就业机会,因为当然,她们并不平等。

    谢谢你指出了我对智商测量的蔑视的不成熟的情感基础。

  75. utu 说:
    @John Jeremiah Smith

    “测试材料完成后销毁。”

    这是伪科学的一个有趣的方面,测量工具必须被销毁,以免污染和影响未来的测量。有点像海森堡问题。问题是,如果一切都如此保密,那么谁来验证伪科学家的存在呢?如果信封一开始是空的怎么办?万一是一个连信徒都不相信的大骗局呢?它闻起来像山达基教和过去的许多其他邪教和骗局,如布拉瓦茨基或葛吉夫。这些都是非科学的努力,无法通过真正的科学检验。但他们在邪教徒和傻瓜的有限环境中坚持并在某种程度上工作。

    这些伪造的智商统计数据大部分是由 Sailer 和 Richwine 等人推广的

    这两个测试正在推动哪些测试(如果有的话)是最不重要的问题。谁是他们的推动者?他们的真正工作是什么是您应该担心的?

    • 回复: @szopen
    , @John Jeremiah Smith
  76. @Peripatetic commenter

    然而,我的观点是,在候选人阶段,你根本不知道她是否会比男性候选人做得更好

    真的。你也不知道是否有男性申请人会做得更好。

    有能力倾向测试、成就测试、能力测试、“动手”实时测试 原位等等。大量的测试,大量的时间去面试和评估。选择大胸妹子是一种失败,同样,不通过冷静地评估优点来选择最佳候选人也是失败的。

    所以,抱歉,但我们不同意。智商完全无关。能力,表现出来的能力是关键。总是。这是根据我 40 年的工作经验得出的。你的旅费可能会改变。

    • 回复: @Peripatetic commenter
  77. szopen 说:
    @John Jeremiah Smith

    读完你在这个帖子上的几条评论后,我不确定你是巨魔、天真还是更糟糕。

    在现实世界中微不足道。

    正如我在这篇文章中已经提到的,假设正态分布(无厚尾)、相同的 SD 并假设 IQ 测量对学业成绩非常重要的东西,那么这意味着(单独使用 _IQ 分数_,即没有诸如差异之类的心理因素)在没有歧视的情况下,超过 2/3 的博士应该是男性。

    当然,男性的可变性很大(因为在很多其他特征上,男性的可变性更大,而且有研究似乎证实男性在智力方面也有更大的可变性)。

    如果女性可以搬运货物,她就可以做她想做的所有 STEM

    如果这位女士能比其他申请者更好地完成这份工作,她就会得到这份工作。

    你似乎被误导了。我认为大多数(如果不是全部)IQ 相关话题的常规参与者都同意上述观点。
    我们希望看到只有优点才能决定谁能找到工作。
    然而,在现实世界中,雇用女性比雇用同等能力(甚至更有能力)的男性面临着很大的压力。在我最近参与的一项资助中,衡量资助成功与否的衡量标准之一是有多少女性参与其中以及有多少女性担任领导职务。
    为什么?因为STEM领域的女性太少了。为什么?因为,显然,这是因为歧视。
    如果你不知道这一点,那么你要么太天真,要么生活在泡沫中。或者你是巨魔。

    我注意到,对女人越好,尤其是如果有人评论她的行为有多聪明(永远不要说“哎呀,你很聪明!”),她对我就越好。

    这与您回复的评论完全无关。这家伙并不是在说女人是好还是不好,而是说她们情绪化多还是少。我并不是说女性是否更加情绪化;而是说女性是否更加情绪化。我只是指出你的评论是没有争议的,暗示了巨魔行为。

    一个人参加两次或两次以上智商测试的平均方差为 3 分

    这是事实,但在测试中,所有男人都朝同一个方向前进的可能性极小。也就是说,变化(有些结果比平常更差,有些变得更好)会相互抵消。这就是基础知识。

    首先:“声明”不是一个词。假设你的意思是“声明”,我不会对你如何从你的屁股中拉出“寻求知识”进行尖锐反驳。

    很多网友评论 unz.com 母语不是英语。例如,我来自波兰。我们大多数人都意识到这一点。

    • 同意: reiner Tor
    • 回复: @John Jeremiah Smith
  78. szopen 说:
    @utu

    只是出于好奇;

    utu,你对心理学的其他部分有什么看法?例如,“大五”、“情商”、“刻板印象威胁”、“启动”、“精神智力”等等呢?

    • 回复: @utu
  79. @utu

    这两个测试正在推动哪些测试(如果有的话)是最不重要的问题。谁是他们的推动者?他们的真正工作是什么是您应该担心的?

    嗯,从某种意义上来说,也许是这样。但谁有时间这么做呢?摧毁美国的主要力量并不是某个哀叹 0.07 分差异的智商“研究人员”俱乐部。

    我只是有时会生气,仅此而已。也许比较鞋码可能会浪费更多的时间、金钱和精力,但也不会浪费太多。

    • 回复: @utu
    , @Wizard of Oz
  80. @John Jeremiah Smith

    你是否总是如此不诚实以至于忽略了我论点的另一部分?

    我们解雇男性比解雇女性更容易,因此女性候选人必须比男性候选人优秀得多才值得考虑。

    当然,其他因素也会发挥作用。例如,如果公司里有人为女性候选人的能力提供担保。

    然而,在所有条件相同的情况下,男性候选人的风险较小。如果她比男性候选人好 25%……那么我会倾向于雇用她。

    • 回复: @John Jeremiah Smith
  81. @szopen

    读完你在这个帖子上的几条评论后,我不确定你是巨魔、天真还是更糟糕。

    更差。我是一位训练有素、聪明、经验丰富的批判性思考者,多年来在一家科技公司担任高层管理职务,取得了相当成功的成绩。

    Sz先生,我不会告诉一个修补匠的妈妈你对神话巨魔的看法。没有一个是孤独的。

    • 回复: @szopen
  82. szopen 说:
    @John Jeremiah Smith

    如果你是批判性思考者,你为什么会犯这样一个基本的思维错误(因为个人分数的差异而挥手小组差异)?

    我不得不说,你根本不在乎我的意见,你很快就吹嘘自己很聪明,有批判性思维,而且很成功😀😀

    • 回复: @John Jeremiah Smith
  83. @Peripatetic commenter

    你是否总是如此不诚实以至于忽略了我论点的另一部分?

    哦,当然。不诚实是我的一贯作风。

    我们解雇男性比解雇女性更容易,因此女性候选人必须比男性候选人优秀得多才值得考虑。

    不,解雇女性和解雇男性一样容易,假设“随意”就业。一些公司愚蠢地担心由于政治气候而遭到女性的诉讼,但这与择优聘用无关。

    然而,在所有条件相同的情况下,男性候选人的风险较小。如果她比男性候选人好 25%……那么我会倾向于雇用她。

    你自便。我的立场仍然坚定地遵循聘用绩效而非智商的原则。你可以雇用任何你喜欢的人——我不会介意。

  84. @szopen

    如果你是批判性思考者,你为什么会犯这样一个基本的思维错误(因为个人分数的差异而挥手小组差异)?

    没有犯错误。无论如何,不​​是我的事。我断言,出于充分、实际和可验证的原因,在雇用有价值的员工时,智商并不意味着狗屎。功绩才是最重要的,而且才是最重要的。现在,你可以将几个概念融入“优点”中——忠诚、职业道德、勤奋、奉献、职场社交技能等。

    但智商呢?不,可笑的是“不”。

  85. utu 说:
    @John Jeremiah Smith

    摧毁美国的主要力量并不是某个哀叹 0.07 分差异的智商“研究人员”俱乐部。

    我希望你是对的,但是为什么你似乎会被它触发呢? “0.007分”的问题当然很荒谬,但智商崇拜是否会影响人们对现实的感知以及他们认识到摧毁这个或那个(更不用说美国)的力量的能力可能是一个非常重要的问题。据称该方法是科学的。据说科学是关于真理的。谁掌握了真相,谁就有了致命的论点。真相是最有力的修辞手段。有充分理由向苏联人民宣传他们的系统是经过科学证明的过程的结果。哪些权力精英从该博客以及 Murrays、Cochrans 和 Sailers 等人传播和流行的模因中受益最多?这让我们想到了你提到的力量。

  86. utu 说:
    @szopen

    我的想法?你很幸运,因为我充满了意见。我的起始立场是永远不要给予怀疑,假设它是垃圾,直到事实证明并非如此。它运作良好。

    • 同意: John Jeremiah Smith
    • 回复: @Wizard of Oz
  87. @utu

    我希望你是对的,但是为什么你似乎会被它触发呢?

    啊,基本上就是胡思乱想。如上所述,我和 NIH 的一位研究人员一起工作了一年。现在,研究对象实际上是临床心理学,特别是偏执行为、偏执妄想。我从未阅读过发表的论文,那时我已经继续前进了。然而,我观察到,嗯,也许有 30 份 WAIS 面试?这足以让我相信,如果将智商视为真实智力的某种反映,那么智商对个人在社会中发挥作用的能力影响相对较小,特别是在生产、合作、互动功能方面。

    卡皮切?因此,我已经准备好驳斥以智商为中心的傻瓜主义。当然,对于某项特定的工作(但极少),一定程度的较高智力可能有助于提高工作效率。但是,美国 99% 的工作岗位都允许非犹太人或非亚洲人担任吗?决不。

    • 回复: @phil
    , @utu
    , @James Thompson
  88. @Anonymous

    同意它需要更多解释,并且需要对所选子测试进行论证。表 3 显示“一般智力”没有差异,而“流体推理”则为 2.2 个标度点,除以假定的标准差 11.68 得到 0.19 此外,女性比男性多,而且“流体推理”的样本量相当小。这个分析。

    我将不得不进一步研究这一点,但感谢您找到相关表格。

  89. phil 说:
    @John Jeremiah Smith

    幸运的是,在评估智商的影响时,我们不仅仅可以参考人们的主观评价。多元回归分析发现,人们收入差异的大约 10% 是由智商差异造成的。 Charles Murray的研究在这一点上与主流是一致的。如果算上投资收入,智商差异可能会占方差的 20% 以上。例如,参见 Marc Dailiard 的分析: 人类品种.org.

    格雷戈里·克拉克(Gregory Clark,《儿子也升起》,普林斯顿大学)研究了人们追求社会地位而不是收入本身的假设,并发现社会地位与身高一样可以遗传多代。

    当然,人们的收入还受到性格因素、劳动力市场的结构性转变(例如某些行业或职业繁荣而另一些行业或职业衰退)、健康状况等因素的影响。

    正如加雷特·琼斯所强调的,智商在国家层面上显得更加重要,这也是撒哈拉沙漠以下国家的智商成为热门话题的原因之一。这项研究对拉丁美洲也有重大影响。

    如果您参加国际情报研究学会的年度会议,您会发现所有与会者都可以容纳在阿尔伯克基或圣安东尼奥等城市的酒店的一间会议室中。相比之下,美国经济协会的年会可能分布在纽约或费城6家酒店的所有会议室。

    心理学家的平均智商低于经济学家,经济学家的平均智商又低于物理学家。一些智商研究存在严重缺陷,但做得好的研究具有很大的相关性,有它比根本没有智商研究要好得多。

    • 回复: @John Jeremiah Smith
  90. @Wizard of Oz

    谢谢。很多积分。是的,你可以训练人们在智力测试中取得好成绩。应试教学并不是很有趣,除非你可以在另一项独立测试中提高智力。 Faven 不应重复给予。
    弗林效应预计会出现在贫穷国家,但收敛速度可能很慢。可能是90年,也可能更长。参见《弗林效应情报》特刊。
    寄生虫负载和一般健康状况不佳很可能是降低分数的因素,而公共卫生的逐步改善可能会带来智力的真正提高。

    • 回复: @Wizard of Oz
    , @Wizard of Oz
  91. dearieme 说:

    请允许我重复我之前的问题:“是否有其他话题女性可能会取得更好的分数,但在测试中通常不会测量这些话题?”肯定有很多,但我想到了一个。

    作为智商测试的一部分,你可以让(比如说)三名女性和三名男性走到考生面前,然后(提前不通知)测试中会出现关于他们的衣服、头发、鞋子、举止的问题,等等。

    我敢打赌,女性考生的得分会比男性高。现在,如何将这个想法转化为实用的廉价纸笔测试呢?也许使用视频?

    敏锐的观察力和对观察结果的记忆力肯定是智力测试的合法组成部分吗?换句话说:智商测试是否因为包含和排除特定的智力技能而对女性存在(轻微)偏见?这些测试组件的选择本质上是任意的吗?

    我想知道聪明的女人在服装测试中是否会比愚蠢的女人做得更好?换句话说,它(大概针对每种性别)是否与智商测试的一个或多个其他组成部分广泛相关?

    T 博士现在无疑会告诉我这个想法在几十年前就已经被测试过。

  92. dearieme 说:
    @dearieme

    继续:是否对廉价纸笔测试的渴望无意中使测试对女性产生了偏见?这对我来说听起来似乎是合理的。纸笔可能适合以物为导向的人(男人和男孩),而不是以人为导向的人(女人和女孩)。

    • 回复: @James Thompson
  93. @James Thompson

    非常感谢。我认为我国还没有准备好接受大规模非洲移民,甚至是东非游牧民族。几代人以来,我们一直在努力为我们的土著游牧狩猎采集者做好事,40多年来,我们的支出大幅增加,但我们似乎还没有掌握诀窍。我的学校牧师曾是苏丹的传教士(并通过在英国监狱服务为我们做准备)。我们从未成立过一个调查委员会来调查他做了多少善事。但这段记忆鼓励了和平队式的想法,即我们可以鼓励 30 多名公务员到非洲休假……。啊,我可以继续穿这身衣服,但早餐锣响了。

  94. utu 说:
    @John Jeremiah Smith

    Capiche? – 是的,是的,但让我告诉你,我认为你如此虔诚地进行的测试与智商测试和以智商为中心的傻瓜主义属于同一个糟糕的领域,你已经准备好驳斥了。我会和你在一起,但你仍然没有对我的品味不屑一顾。

  95. @phil

    菲尔,你没注意。聪明的人往往有更多的收入,这是否令人惊讶?真的,想一想:是不是很震惊?一点也不……但是……一生中10%的差异?嗯……似乎是一个要求,但事实并非如此。

    所有这些都不在这里也不在那里。你们这些智商狂人,可以尽情地谋划,但永远不会成为一座豆子山。哎呀哎呀,女人就是低人一等,是的,智商低了0.17分。我们怎么能让他们这些愚蠢的女人担任公职,甚至在银行工作?或者,该死,干脆就被雇佣了?他们应该呆在家里,跟城堡的领主上床,打扫房子,看管孩子。

    内格罗斯?弥赛亚人?上帝禁止。他们太笨了,几乎无法向北走。中国佬?好吧,有时很聪明,但从来没有创造力,对吧?中国人从来没有发明过一件该死的东西,对吧?

    我知道你是什么人,菲尔。我一生中的大部分时间都见过像你这样的人,读过像你这样的人,避开像你这样的人。你所有的“研究”,所有的“测试”,所有的先入为主的想法到底取得了什么成果?之宝。齐尔奇。没什么。梅西卡人入侵了这个国家。黑人在政府部门任职。担任高管职务的女性虽然笨得像石头一样。

    你对智商的痴迷是否真实,一点也不重要。重要的是,你们这些智商迷正在迅速失去阵地。没有人关心谁聪明谁不聪明。重要的是财富和权力。

    • 回复: @Santoculto
    , @utu
    , @phil
    , @phil
    , @szopen
  96. @dearieme

    请允许我 JT 将这个想法延伸到当你提到标准化 tssts 时我想问的一个问题,然后我在 Flynn 对 Lynn 的评论中找到了更多参考资料。

    如果测试已经标准化以确保男性和女性具有相同的平均分数,为什么大样本中的男性得分高于女性?我可以看到这种情况发生在分数远高于(或我认为低于)平均水平的测试对象中的逻辑可能性,但我很难理解它。

    • 回复: @James Thompson
  97. @utu

    我可以看到可能的收敛点。喜欢再来一品脱吗?

  98. Santoculto 说:
    @CanSpeccy

    可能??

    差异更大,因为这是一个发展时期,特别是在个人/儿童中,但个体差异似乎在一生中都是恒定的,并且可能由于环境噪音而导致人们往往表现更好,例如一个人在语言上比非语言上更聪明,这很可能是她/他在语言方面会比在非语言方面(或n)表现得更好。

    更好=更自信。

  99. @utu

    您认为哪些群体(您的“权力精英”正在回避问题)从这些模因中受益?这些表情包的声音是否足以产生任何实质性影响?他们没有做太多事情来帮助白人工人阶级和中产阶级阻止廉价劳动力的流动,不是吗?谁从中受益?史蒂夫·赛勒尤其反对美国的移民政策和做法,以保护美国工人的利益。

    • 回复: @utu
  100. Santoculto 说:
    @dearieme

    智商测试会歧视社交/同理心/情感技能,而女性传统上往往比男性更好和/或倾向于。智商测试在没有情绪参与或情绪中和/hipo表达的情况下分析认知技能。

    但情商的终极测试将是,这个终极测试之一:你有多善于理解与你不同的人?

    平均而言,女性很容易理解他人,就像男性很容易理解其他男性一样。但也不太了解男人……

    其他终极测试:您对理解/识别许多不同类型的人的真实行为模式的能力有多强?

    同样,女性的平均水平似乎并不比男性更好,只是更倾向于。

    更倾向=/=知识渊博
    强度/频率=/=质量

    女性比男性更倾向于使用防御性和攻击性同理心/换位思考/理解具有给定意向的其他模式。由于女性更具有同理心,因此给人的印象是她们的情感技巧比男性要好得多。在我看来,尝试解除其他人的武装并建立值得信赖的团体更为合理,但平均而言,女性的进化程度还不足以识别威胁/掠夺者。

  101. @John Jeremiah Smith

    你对智商测试的拒绝似乎太过分了,所以我向你提出了这个测试问题。如果你被可靠地告知,一个第一世界政体的立法者在超过 15 年的记录中测量的智商平均为 125 sd 6,另一个为 115 sd 7,你会不会相信前一个政体可以被判断为更好的治理国家?有什么常识性的衡量标准吗?我尝试考虑欧洲国家,但也许美国各州也可以。也许是华盛顿州对阵密西西比州?明尼苏达州诉阿拉巴马州?……

    • 回复: @John Jeremiah Smith
  102. @James Thompson

    请允许我微小的阿斯伯格成分提及#102

    • 回复: @James Thompson
  103. Santoculto 说:
    @John Jeremiah Smith

    财富和权力更多地与精明/黑暗情商和认知技能/智力的结合有关。

    它与智力间接相关,至少是 IQistics 所喜爱的智力类型。

    并不总是关于黑暗情商,但毫无疑问是“最好”的同理心和“最好”的系统化的结合。

    事实上,如果你只是重复当前的叙述,那么你提高社会地位/成功的机会就会更高,特别是如果你能够在这种意识形态信号之上发挥创造力。

  104. utu 说:
    @John Jeremiah Smith

    重要的是财富和权力。 最后。好的。现在您应该准备好进行下一步了。你真的认为“智商狂人正在迅速失去地位”吗?与此相反的。它们对于系统来说非常重要。他们正在为所谓的精英政治的基础提供缺失的关键吗?正如我之前所说,真理是最有力的修辞手段。谁拥有它,谁就获胜。但事实又是怎样呢?只要人们认为如此,这确实并不重要。这是这个博客以及默里、科克伦和赛勒等智商狂人的工作,让人们相信智商决定论的真相,以及统治阶级和精英拥有更高智商,因此最值得担任他们的职位的迷因。像女性或黑人智商较低这样让你们兴奋的问题只是分散注意力、安慰奖、大众大师餐桌上的残片,但主要是激励有用的白痴继续免费工作和工作。违背了他们对制造同意书以及为了维持现状而传播模因的工厂的兴趣。

  105. utu 说:
    @Wizard of Oz

    史蒂夫·赛勒尤其反对美国的移民政策和做法,以保护美国工人的利益。

    如果史蒂夫·塞勒仅仅争论美国工人的利益,或者如果布赖特巴特新闻网仅仅关注民族班农主义,而没有强烈的支持犹太复国主义和反伊朗的角度,那么他们将被允许只在公共厕所的隔间上潦草地写下他们的信息。

    • 回复: @Wizard of Oz
  106. 詹姆斯·汤普森试图通过分析唯一可用的原始数据——智商,为某些领域女性代表性不足的问题提出科学论据。 我必须同意约翰·耶利米·史密斯的观点,即这是一个糟糕的方法,但我不能同意他的其他一些论点。 让我用另一个数据集来证明同样的观点。

    女性在管理层中获得平等代表权的唯一方法就是像男性一样:分享公平的份额。 如果女性创造了公平的企业和就业机会,那么她们将在管理层中拥有平等的代表权,因为她们将成为自己公司的首席执行官。 玻璃天花板的存在是因为女性完全依赖男性来晋升。 如果女性没有表现出创建成功企业的同等能力,那么男性为什么要认为她们同样有资格经营企业。

    女性首席执行官不追踪智商的原因有很多。 原因之一不是某个白人男性好老男孩系统。 成功的企业是残酷竞争的幸存者,并且在这个世界上尽可能客观。

    • 回复: @John Jeremiah Smith
  107. @Wizard of Oz

    你对智商测试的拒绝似乎已经太过分了

    我并不拒绝智商测试——我只是说我不会因为某人的智商分数高于其他候选人而雇用他或她。总的来说——尽管有不同意见——我相信一个人需要 100 分的智商才能发现一个人与另一个人之间的真正差异。从 110 到 110,再到 120 到 120,再到 130 到 XNUMX,等等,你会发现意识、解决问题、词汇量、分析能力等方面有明显的差异。只是我的观点。

    至于你的“立法者”的建议,我认为完全脱离了现实。政府最突出的方面是其腐败倾向,而这往往会排除任何“情报”影响。

    • 回复: @Wizard of Oz
  108. @utu

    重要的是财富和权力。最后。好的。

    阿尤普。你是完全正确的。或者,我应该仅仅根据一般原则驳回你所说的一切吗? 😉

    如果你不断地提出这样不可避免的、令人沮丧的观点,那么没有人会和你说话。我认为你最好在某个地方发现自己的阴谋,然后在这个博客或任何博客上彻底解决它。

    有时我觉得我生活在巨蟒剧团圣杯的模仿中。是的,我说过,模仿!

  109. @utu

    那么,重复一遍,史蒂夫的模因适合哪些权力精英以及为什么它适合他们?这对让他写的东西成为广泛传播的模因有多大帮助(我对此表示怀疑)?

  110. @Mark Presco

    女性在管理层中获得平等代表权的唯一方法就是像男性一样:分享公平的份额。 如果女性创造了公平的企业和就业机会,那么她们将在管理层中拥有平等的代表权,因为她们将成为自己公司的首席执行官。 玻璃天花板的存在是因为女性完全依赖男性来晋升。

    有效的对立点,同意。 为了捍卫我的立场,请允许我重申一下,我仅指雇用、工作分配、随意雇用的任务绩效,即作为人力资源总监,我雇用谁,为什么?

    建立一家成功的公司并成为首席执行官并不是“我要雇用谁以及为什么?”的问题。一组条件;它具有创业性和参与性。

    • 回复: @Mark Presco
  111. @utu

    我觉得我必须添加“天哪!一个该死的革命者最好注意他所说的话。”

    这并不是特别危险,但不要把谨慎抛在脑后。联邦调查局确实会派出非常友好、健谈的人员来与您交谈。使用诸如“您认为对此应该采取什么措施?”之类的短语。以及“某某说某某。这是你会同意的事吗?”和“你这里的地方真好。为了实现这一切,你一定付出了很大的努力。” “你的妻子在县学校工作,对吗?”

    友好的。性格温和。吓坏了天哪。

  112. Santoculto 说:

    领先4分……

    但男性没有U型分布吗?

  113. @John Jeremiah Smith

    为了捍卫我的立场,请允许我重申一下,我仅指雇用、工作分配、随意雇用的任务绩效,即作为人力资源总监,我雇用谁,为什么?

    为了简洁起见,我故意删除了一段阐述你的立场的段落,因为我只是认为最终的例子显然适用于所有领域。

    攀登企业阶梯,或者就学术界而言,同样具有竞争力。 STEM 领域的成就者大多是男性,因为白人是地球上最具竞争力的人。 女性必须自己将自己列入这份名单,然后才能被视为平等。

    男性和女性在工作场所的方式还有很多其他差​​异,其中最重要的一点是,男人就是他的工作。 我所钦佩的历史上的伟人因其所做的工作而被人们铭记。 需要我为你说出它们的名字吗? 我很惊讶,作为一名人力资源人员,你似乎完全没有意识到这一点。

    您是否因平权行动和多元化而聘用员工? 对您来说,平等是否意味着平等的结果而不是平等的贡献?

    • 回复: @John Jeremiah Smith
  114. phil 说:
    @John Jeremiah Smith

    在最近对智商研究人员的调查中,史蒂夫·赛勒和阿纳托利·卡林被大多数受访者挑选为准确报告了智商研究。另一方面,《纽约时报》、《经济学人》、国家公共广播电台等媒体因忽视或歪曲这项研究而受到广泛批评,尽管该研究在许多情况下已经被复制。

    没有哪个单一因素可以解释人们劳动力市场收入差异的大部分,但在重要因素中,智商位居第一或第二(责任心/职业道德也非常重要)。诺贝尔奖获得者詹姆斯·赫克曼在《钟形曲线》出版后对它进行了批评性评论,但后来称赞这本书打破了将智商纳入收益讨论的禁忌。赫奇曼自己的回归使他得出结论:提高 A 的认知能力

    • 回复: @John Jeremiah Smith
  115. phil 说:
    @John Jeremiah Smith

    没有任何单一因素可以解释人们劳动力市场收入差异的大部分。在重要的因素中,智商和责任心/职业道德尤其重要。诺贝尔奖获得者詹姆斯·赫克曼在《钟形曲线》出版后写了一篇批评性评论,但后来称赞这本书打破了讨论智商在人们生活中的作用的禁忌。他自己的回归使他认为,改善非裔美国人生活的最重要的政策改革在于学前教育,以提高他们的认知技能和责任心。我碰巧怀疑他的政策建议的有效性,但我很高兴他以建设性的方式参与讨论,而不是简单地唱反调。

    你的评论似乎不是对智商相关性的建设性参与,而是对政治和社会世界发展方向的哀叹。我对你的人身攻击和脏话感到失望。

    我感谢汤普森博士的这篇文章和新一期的《人类季刊》。其他出版物不会触及智商的性别差异问题。

  116. @Mark Presco

    您是否因平权行动和多元化而聘用员工? 对您来说,平等是否意味着平等的结果而不是平等的贡献?

    你已经离得太远了。抱歉,没有兴趣。

    提醒:智商测试分数是否可以正确衡量个人作为员工的吸引力?答案:不。只有表现出的优点、经验和能力才是衡量是否适合就业的适当标准,或者,如果是缺乏经验的新员工,则需要衡量能力和态度。

  117. @phil

    胡扯。智商和 2 美元可以买一杯咖啡。剩下的就是谁是你爸爸了。

  118. @John Jeremiah Smith

    考虑到影响特定日期任何特定测试分数的各种因素,并且可能对人们产生不同的影响,我想我同意,可能需要报告 8 或 10 的差异才能给予预测或判断信心的权利。我认识的一位粒子物理学家,他在学校期间获得过诗歌奖、数学奖以及科学奖和数学奖,他的智商测试结果显示为 183 和 137。(我看到了记录)。所以,不要认为我是一个信徒(在任何事情上)。但如果我必须从智商在 120 左右的候选人中或智商在 125 左右的候选人中选择软件工程师或高管,如果我只知道这些,我会毫无疑问地从 125 人中进行选择。你会?

    • 回复: @John Jeremiah Smith
  119. szopen 说:
    @John Jeremiah Smith

    哎呀哎呀,女人就是低人一等,是的,智商低了0.17分。我们怎么能让他们这些愚蠢的女人担任公职,甚至在银行工作?或者,该死,干脆就被雇佣了?他们应该呆在家里,跟城堡的领主上床,打扫房子,看管孩子。

    你是否意识到你正在用想象的观点创造想象的对手?您甚至了解“平均”和“每个”之间的区别吗?

  120. @Wizard of Oz

    谢谢。我认为哈萨比斯很好地涵盖了这个领域。
    他的团队在此类问题上遥遥领先。我会先看看他们在做什么,因为除非招募到最好的球员,否则可能会浪费时间。

    https://www.unz.com/jthompson/artificial-general-intelligence-von

    • 回复: @Wizard of Oz
    , @res
    , @Wizard of Oz
  121. @John Jeremiah Smith

    我不知道你是否会觉得它相关,但施密特在这个问题上做了很多工作。我在博客中多次提到过他,但这里有一个相关的。

    https://www.unz.com/jthompson/jobs-iq-tests-versus-interviews

    保罗·萨克特(Paul Sackett)也有出色的作品。

    https://www.unz.com/jthompson/what-do-college-admission-tests-predict
    https://www.unz.com/jthompson/can-tests-predict-academic-outcomes

    • 回复: @szopen
    , @John Jeremiah Smith
  122. szopen 说:
    @James Thompson

    不知道大家有没有读过之前的交流(关于“智力增长”)。

    RaceRealist88 之前提到过 Ken Richardson 和 Norgate(“智商真的可以预测工作表现吗?”)的批评,包括他们对您引用的亨特和施密特荟萃分析的强烈批评(但不仅仅是他们的)。我觉得理查森提出了许多精彩的观点,尽管我在阅读他的论文时隐约感觉作者有偏见;尽管如此,我认为它们破坏了我对智商与工作绩效联系的坚定信念。我很高兴看到其他研究人员对他们的论文做出回应。

    另请参阅此处 RR88 的博客文章: https://notpoliticallycorrect.me/2017/10/04/doctors-iq-and-job-performance/

    还有两项关于智商预测绩效的荟萃分析:Lang et al 2010(“一般心理能力……”)中针对德国的分析,它避免了理查森批评的部分但不是全部(例如,除了主管评级之外,他们还考虑到笔试和口试等)以及 Salgado 等人 2003 年针对欧洲的情况(“欧洲共同体不同职业的一般心理能力有效性的元分析研究”)。这三者都发现了工作复杂性和智商预测值之间的联系。

  123. @dearieme

    谢谢。有许多测试可以让男性和女性展示特定的技能。如果有时间和资源,我们可以研究着装、举止、汽车识别、超市导航、讲笑话、常识测验,并找出组织活动的最佳方式。其中许多已经被研究过。然而,测试必须比观察现实生活更短、更便宜。因此,诸如数字跨度、数字符号编码、简单数学或简短词汇测试之类的简单事物的优点,仅仅是因为它们具有预测能力。您需要具有良好预测/耗时比率的东西。
    如果注意人们的穿着可以预测生活中的成就,那么一定要研究它。我认为让人们给植物命名会更好,也更容易做到。
    这是一项女性可能做得更好的测试。
    https://www.unz.com/jthompson/look-in-my-eyes-you-sensitive-clever

    • 回复: @Wizard of Oz
  124. @Wizard of Oz

    争论的焦点是,男性总体上更聪明,在空间任务上更聪明,尽管测试标准化试图消除这种差异,但它仍然在测试和现实生活中出现。然而,与往常一样,我们必须检查所有数据,以确保所声称的男性优势确实如此,并仔细查看样本的代表性和所使用的广泛测试范围。

    • 回复: @Wizard of Oz
  125. @Wizard of Oz

    当我慢慢地处理这些评论时,允许并回答。顺便说一句,我不认为你对细节的关注是一种障碍。

  126. @James Thompson

    这使得 PC 标准化心理测量学家听起来有点像财政部和税务局的普洛兹先生或博士,他们的政治任务是起草聪明人无法回避的税法。显然,如果竞争激烈的雄性在为适应雌性而设计的测试中仍能找到比雌性做得更好的方法,那么它们就失败了。他们不应该因为他们的产品而获得报酬,我希望看到他们面对小型皇家委员会、公共账目委员会,并证明他们的工作和他们获得报酬的合同是合理的。

  127. @James Thompson

    我认识的一个女人,她童年时的智商被测了两次,大约为 175(我不知道是哪次测试,所以可能是卡特尔),她对植物、衣服、室内设计、珠宝、鸟类有着敏锐的观察力,几乎能打败任何男人在所有这些方面都相差一英里,并且是一个信息丰富且有趣的无笔记演讲者。但她的部分优势一定是她有单词/名字来帮助组织材料和记忆。比较一下那些不仅看到多边形,还看到三角形、正方形、五边形、六边形等及其数学属性的人。除了许多测试中流体智力的巨大差异之外,这不会压倒任何东西吗?

  128. Santoculto 说:
    @szopen

    我的印象是,大多数男性在空间任务上往往比大多数女性更好,无论他们/我们的一般智力是否倾向于 U 形分布,而大多数女性在其他可能与语言和语言相关的问题上往往比男性更好。情感技能,即使是这种效率,也往往不如那些与空间技能相关的技能那么有影响力,这可能是因为我上面所说的,如果女性在心理/“移情”问题上确实比男性聪明得多,她们将有能力也了解男人的行为。

    我的意思是,即使一般认知技能较低的男性在空间任务上的平均水平仍然比一般认知技能相对较高的女性更好。机械工人/男性的例子(这表明他在空间/“系统化”任务上表现更好)在一般智商/认知技能方面得分约为 90 分,而一名学术女性的得分约为 110 分,但对汽车功能一无所知。

    我想知道东亚人和黑人在认知技能方面的性别二态性平均是否比欧洲白种人要低。至少在心理趋势上,黑人女性似乎比欧洲白人女性更“男性化”。关于东亚人,我真的不知道,但印象是,他们的性别二态性往往比欧洲白种人少,他们的身体特征已经存在。

    分析空间/语言技能倾斜和……例如宗教信仰之间是否存在某种不同的相关性也会很有趣。

    • 回复: @szopen
  129. szopen 说:
    @Santoculto

    标准的女权主义者反应是“但是经过一周的训练,女性可以像男性一样玩俄罗斯方块!”或者,更复杂“是的,尽管经过男性和女性的训练后,双方都取得了进步,所以差距仍然存在,但男性和女性玩第一人称射击游戏的学习曲线是相似的,所以这都是文化上的”。

    • 回复: @Santoculto
  130. Anonymous [又名“newAccount2”] 说:

    顺便说一句,为了消除性别差异而对 WAIS 等进行的任何调整是否只对女性有利? 这似乎是林恩或杰克逊等一些研究人员做出的假设,但我看到了不同的意见。 例如,请参阅 Lichtenberger 和 Kaufman 的评论:

    WAIS-III 上对男性有利的 VCI 差异大于 WAIS-IV 上的相应差异,但并不重要,因为它与该主题的大量文献相反:

    一组已被多次重复的发现是,平均而言,女性在某些言语能力测试中得分较高,尤其是那些需要快速访问和使用长期记忆中的语音和语义信息的测试。 (Halpern、Beninger 和 Straight,2011 年,第 254 页)
    女性擅长的语言能力类似于在韦克斯勒 VCI 子测试中表现良好所需的能力。

    此外,主要智力测试中语言分测试的性别差异结果受到污染,因为测试开发人员在测试开发阶段一直试图避免性别偏见,无论是在电池分测试的选择上还是在每个项目的选择上。子测试。 马塔拉佐(1972)指出:

    从一开始,最著名的个人智力量表(比奈、特曼和韦克斯勒)的开发者就非常小心地从最终量表中平衡或消除任何根据经验发现会导致某一性别得分高于另一性别的项目或子测试。其他。 (第 352 页;原文斜体)
    根据韦克斯勒(Wechsler,1958)的说法:“采用这种程序的主要原因是它避免了对男性和女性制定单独规范的必要性”(第 144 页)。平衡或消除性别偏见项目主要影响语言分测试,例如信息或理解(以及 WMI 分测试算术),其中项目的内容可能有利于一种性别而不是另一种性别。相比之下,像块设计、矩阵推理、符号搜索和编码这样的子测试是由“与内容无关”的项目组成的,这些项目不可能有任何偏见。因此,从科学角度解释 PSI 和 PRI 上的性别差异是合理的。

    https://books.google.at/books?id=SUqcospVEMcC&lpg=PP1&dq=alan%20kaufman%20wais&hl=de&pg=PT234#v=onepage&q&f=true

    • 回复: @Passer by
    , @Passer by
  131. @James Thompson

    感谢您的回复,包括链接。我是
    有兴趣看到你将哈萨比斯的前沿工作称为模仿神经网络,当然我无法亲自验证,也许并不真正理解。我突然想知道大跃进是否——?神圣之举——可能是找到与人脑完全不同的学习和解决问题的方法,并且人脑无法使用这些方法,除非以某种形式写下人工智能机器所做的一切。但也许你没有使用“神经网络”来表示与人类相同类型的神经网络。

  132. @Wizard of Oz

    但如果我必须从智商在 120 左右的候选人中或智商在 125 左右的候选人中选择软件工程师或高管,如果我只知道这些,我会毫无疑问地从 125 人中进行选择。你会?

    呃,不是要回避这个问题,但不是。如果我只知道一群候选人的智商分数,我就不会雇用他们中的任何一个。

    更像是“哦,见鬼,不。”

    • 回复: @Wizard of Oz
  133. @szopen

    你是否意识到你正在用想象的观点创造想象的对手?您甚至了解“平均”和“每个”之间的区别吗?

    哈哈。为什么,是的,我愿意。

    你是否明白,以 0.07 的 IQ 分差异作为衡量一个人的标准是毫无意义的? 什么,有什么吗?

    • 回复: @szopen
  134. @John Jeremiah Smith

    很好的躲避,多亏了我没有坚持让你必须选择一个快速聪明的人,所以把它打开了。

  135. Santoculto 说:
    @szopen

    每个人都可以在短期内做出微小的改进,但我们谈论的是长期的,如果不是至关重要的。

    认知/心理偏好就像性偏好一样,很难改变,但它可以因人而异(当然也是有限的)。

    它也是文化,我们是生物文化产品,但不是“文化创造……”,“我们首先创造文化,我们自己的文化,而且往往个体文化倾向于与集体文化相适应,即在长期的文化稳定中地方”。

  136. @James Thompson

    我不知道你是否会觉得它相关,但施密特在这个问题上做了很多工作。

    我已经雇佣了几十个人。对于大多数人来说,智商从来都不是一个考虑因素——经验、能力和面试评估总是能产生真正的影响。

    我要提到的是,智商(如果有的话)是立即拒绝的基础,因为智商太低或太高。根据特定工作的要求,智力分数低的人往往学习速度慢,而我们当然不需要这些。高智商的人更有可能成为麻烦制造者——如果您看到申请某些职位的 SB 分数超过 150,该分数可能有助于排除该人。一个非常好的面试和工作经历可能会产生影响,但任何高智商但工作经历参差不齐的人通常都是两条腿的麻烦;如果是女性就更糟糕了。

    • 回复: @Wizard of Oz
  137. @Wizard of Oz

    很好的躲避,多亏了我没有坚持让你必须选择一个快速聪明的人,所以把它打开了。

    嘿,维兹,对不起,我去过那儿,做过那件事,现在知道得更多了。单因素速聘,闲时后悔。

  138. szopen 说:
    @John Jeremiah Smith

    相差4个智商点,上面我已经写过会有什么后果了。与你的说法相反,如果学术工作仅根据智商来授予,那么它们并非毫无意义。

    • 回复: @John Jeremiah Smith
  139. @szopen

    请参阅 J.Thompson 的评论 #57 在重新 0.07分的差距。

    那么,您认为“学术”工作可以仅根据智商来授予吗?我只能假设,一个智商为 160 的水管工会比一个智商为 159 的物理学家更喜欢核物理讲师的职位?

    天哪,听起来像 地狱 一个计划!让我们开始吧!

    • 回复: @szopen
  140. @Wizard of Oz

    很好的躲避,多亏了我没有坚持让你必须选择一个快速聪明的人,所以把它打开了。

    哦,我忘了。抱歉闪避了。

    面对在 2 小时内绝对需要雇用某人的情况,您选择了最漂亮、乳房最大的女性。毕竟,在接下来的 30 天内,您将必须招聘、评估和聘用真正能够胜任这项工作的人。

    • 回复: @Wizard of Oz
  141. szopen 说:
    @John Jeremiah Smith

    再读一遍。我写过“如果学术工作会”,则不应该。你不能读书吗?

    回复 .07:汤普森博士在此报告的研究(您正在评论该研究)报告了 4IQ; _其中_一篇参考文献报告称一般能力没有差异;你不是在评论 4 个智商点的发现,而是在攻击其中一项研究

    有一个普遍报道的神话,即沟通范围约为 30 IQ 点 - 如果更大,则认为沟通是不可能的。有趣的是,你发现智商 150 的人很麻烦。

    • 回复: @John Jeremiah Smith
  142. szopen 说:
    @James Thompson

    我多次看到这样的说法:gf 经常被发现与 g 相同或几乎相同;怎么会有这么大的差别呢?他们的意思是“一般智力”是“g”还是其他什么?

    而且,他的意思真的是0.07分0r 0.07 SD吗?

  143. @szopen

    再读一遍。我写过“如果学术工作会”,则不应该。你不能读书吗?

    我可以读书吗?有人告诉我我的阅读和理解能力相当好。你会读...吗?你明白“If”这个词在条件实例化方面的含义吗?

    你说我直接引用本文作者的观点来攻击一项研究。我再问:你能读书吗?

    我不知道你提到的是什么:“有一个普遍报道的神话,即沟通范围约为 30 IQ 点。”也许这是您未能相信的一些经过验证的研究?

    “有趣的是,你发现智商 150 的人很麻烦。”没这么说;你似乎故意歪曲和歪曲实际所说的内容。我说过,我们发现(企业人力资源服务)高智商但工作经历不佳的人是麻烦制造者。

    你的里程可能会有所不同,而我是 相当 当然可以。再见,宝贝。

    • 回复: @szopen
    , @res
    , @reiner Tor
  144. res 说:
    @James Thompson

    感谢您再次提出这个问题。激励我终于看他的演讲。真正让我印象深刻的一件事是 1:56 提到的改进率(和一致性)(每月约 00 个排名)。

    自从李世石获胜后,我就没有密切关注,所以回去检查排名提高率是否持续,AFAICT确实如此。在单台机器上达到最高 1,000 Elo 点,比谈话时高出 XNUMX 个 Elo 点! https://en.wikipedia.org/wiki/AlphaGo#Versions

    他的三个围棋流派在大约一分钟后迫使它们脱离局部最大值评论非常有趣。我不知道他现在是否还有这样的感觉。我想这个评论可以被视为“相对”。

    58:15 左右开始的一些关于直觉和创造力的有趣想法

  145. @John Jeremiah Smith

    我相信你有正确的技巧来骚扰女性离开,而无需对你的公司提起昂贵的诉讼。 (我确信你裸体比哈维·韦恩斯坦好看,而且不会邀请她一起洗澡。也许你可以让她向她的女朋友解释/吹嘘,你对她投以小狗般的眼神并爱上她的方式是变得如此尴尬,尤其是在你的妻子不断不定期地拜访之后)。

    我对此类测试问题的原始形式是(大约)假设您必须为纽约市一份要求较高的软件工作招揽申请人,并向您提供人员名单及其电话号码。一份名单上的名字完全是犹太人的名字。另一个人的名字包括勒罗伊、达肖恩、德维恩、卡修斯和马凯拉。您首先选择哪个列表?在直接智商案例中,您将被邀请面试六名具有适当论文资格的候选人,并有机会从 125 人或 115 名单中选择六人……。

    • 回复: @John Jeremiah Smith
  146. @John Jeremiah Smith

    除了秘书和簿记员之外,我几乎所有的经验都是聘请研究主管或任命教授,你提醒我需要解雇一个在学校名列前茅、讲授大学数学并获得一等法律学位的人!不寻常但并不独特。

  147. szopen 说:
    @John Jeremiah Smith

    (1) 鉴于我提到了我之前的评论(它是#83,如果你错过了),其中我明确指出“假设智商衡量某些东西……”,我想说应该清楚的是,“如果”意味着“假设地,如果……” ”。但由于我不是母语人士,而你是,我认为这是我的错误。让我重申一下“在一个假设的世界中,所有博士学位都将仅根据智商授予,男性将占博士学位的 2/3”

    (2) 正如 Thompson 博士在此引用的,Lynn 尚未发现 0.07。他写道“在卡马拉塔和伍德科克 2006 年的论文中……”,即他引用的其中一篇论文。然而,Lynn 发现总体 4IQ 得分。

    换句话说,就好像我会写“A 发现男性比女性挣 100 美元,B 发现男性与女性挣相同(实际上多 1 美分,C 发现男性比女性挣 150 美元。基于这些)研究,使用复杂的算法和我自己的研究,我认为男性比女性多赚 125 美元”,你会开始“1 美分的差异?这太荒谬了!”。攻击方法,怀疑作者得出结论的方式,解释研究 A应该给予更多的重视都是有效的批评。通过嘲笑引用的一项研究发现的差异来攻击作者则不然。

    (3) 我提到网络上经常出现的都市传说,最大通讯范围是+-2SD,即30IQ点。事实上,这个特定神话的基础是一项古老的研究,其中作者(似乎是基于她的直觉或个人观察)声称领导者的智商不可能超过 30 点。这个神话如此流行和持久,我想你会认识到它,特别是因为你声称你熟悉智商测试。

    (4) “你似乎故意歪曲和歪曲实际所说的内容。”不是我的本意。道歉。

    我想说,高智商可能会成为某些工作的障碍。但请记住跟踪高智商儿童直至成年的纵向研究结果(汤普森博士前段时间也提到过),与普通人相比,无论以博士学位数量、专利数量来衡量,他们都取得了令人印象深刻的成就或者收入——我想说智商毕竟不是那种无用的衡量标准。

    我并不是说所有高智商的人都会自动成为更好的员工。我声称,对于一大群人,唯一给出的信息就是他们的平均智商,如果我必须用真钱赌他们在任何需要智力的工作上的结果,我宁愿赌在那些有智力的人身上。平均智商较高(而不是胸部最大的女孩)

    • 回复: @John Jeremiah Smith
  148. @Wizard of Oz

    我相信你有正确的技巧来骚扰女性离开,而无需对你的公司提起昂贵的诉讼。

    将它们放在地下室清洁办公室的桌子上,没有互联网接入。最多两周,他们就走了。

    我对此类测试问题的原始形式是(大约)假设您必须为纽约市一份要求较高的软件工作招揽申请人,并向您提供人员名单及其电话号码。

    嗯,我一直在现实世界中工作。员工既是资产,也是负债。在我看来,员工的选择不应该只是一个仔细评估的过程。

    • 回复: @res
    , @Wizard of Oz
    , @reiner Tor
  149. dcite 说:
    @jacques sheete

    哈欠……尽管它有这么多好处,但也可能领先 40 分。 原因是女人知道男人是被他们的球而不是他们的大脑统治的。

    嗯,她们在男性意识中处于非常前沿和中心的位置。 多年来,无花果叶已经证明了其隐喻和社会用途。 但我曾经在一本心理学书中读过一段话,其中说男人更有可能能够专注且集中地专注于一件事。 或者两个。 情感领域之外的机械和抽象事物。 不满和义务。 这是有道理的。 我无法想象我认识的女孩中有哪个女孩会 24/7 痴迷国际象棋 40 年,即使是那些喜欢下棋并且表现出色的女孩。 我看到的最接近它的是芭蕾舞演员和一些音乐家。 他们比那样的男性还要变态。 智商并不是造成这种差异的主要因素,但它当然会影响汇总数据。

  150. @szopen

    我并不是说所有高智商的人都会自动成为更好的员工。我声称,对于一大群人,唯一给出的信息就是他们的平均智商,如果我必须用真钱赌他们在任何需要智力的工作上的结果,我宁愿赌在那些有智力的人身上。平均智商较高。

    明白了。我没有误解你的喜好;我纯粹出于现实世界的实用性而反对它们。

    是的,如果所有其他因素都达到或接近最优,那么聪明的人可能是更好的员工。然而,根据我的观点和经验,智力并不是最重要的因素,甚至不是选择生产性员工的特别重要的因素。工作经历、具体经验、态度、能力和足智多谋更为重要。

    这是我的经验,我的看法。一如既往,你有权提出不同的、甚至直接反对的意见。

    这个话题已经接近尾声了。也许不久之后,Unz 先生的代理就会出现同样有趣的东西。 😉

  151. res 说:
    @John Jeremiah Smith

    我不知道你提到的是什么:“有一个普遍报道的神话,即沟通范围约为 30 IQ 点。”也许这是您未能相信的一些经过验证的研究?

    (不是szopen,但我对这个话题有一些兴趣)

    此链接讨论了这个想法,并努力寻找参考资料(另请参阅评论): http://blogs.discovermagazine.com/neuroskeptic/2017/08/31/myth-30-iq-communication-range/

    根据 Google 书籍,Steve Sailer 在 Space Cadet 第 112 页的评论中提到了 Heinlein 的引言: https://books.google.com/books?id=xH9hbmGYlVsC
    这个变体是领导者/追随者的差异,恕我直言,这更有趣。它在很大程度上解释了为什么士官/军官的划分对于军事领导力很有用。我对 iSteve 关于这一主题可能进行军事研究的猜测感到好奇。

    显然,这个想法过于简单化了。沟通差距没有 30 IQ 点(又名 2 SD)的硬性阈值,这在很大程度上取决于所讨论的主题。但我确实认为这个想法有其道理。有趣的是,我认为 115 左右的智商是观察这一现象的最糟糕的有利位置。这些人很可能是大学毕业生,不会花太多时间与智商低于 85(-30,最低 1/6)的人进行超越肤浅的互动。智商高于 145(+30)的人大约占总人口中的千分之一,并且倾向于(IME)集中在学术、专业和地理上。除非 IQ 1 的人在这些集中区域花费大量时间,否则我认为他们不太可能遇到许多 1000+ 的类型。

    对我来说,我看到的最大区别是抽象/具体的沟通能力,这种差异似乎与智商没有明显的关系。

    这有点相关: http://charltonteaching.blogspot.com/2014/04/how-big-does-iq-gap-have-to-be-before.html

    从另一种角度来看,这意味着智商超过 145 的人在与 5/6 的人口沟通时会遇到困难(而且情况会很快变得更糟!)。

    PS 有人对此有其他想法吗?

    PPS 我很好奇,如果你不考虑智商,你(集体)是如何得出以下结论的。你用什么来鉴定这 150 个人的智商?

    “有趣的是,你发现智商 150 的人很麻烦。”没这么说;你似乎故意歪曲和歪曲实际所说的内容。我说过,我们发现(企业人力资源服务)高智商但工作经历不佳的人是麻烦制造者。

    • 回复: @szopen
    , @John Jeremiah Smith
  152. res 说:
    @John Jeremiah Smith

    嗯,我一直在现实世界中工作。员工既是资产,也是负债。

    您在这里花了相当多的时间来展示您的人力资源经验和专业知识。鉴于此,我认为有必要询问一下您正在招聘什么类型的行业和职位。雇佣教授(就像绿野仙踪那样)和雇佣扫帚推手之间有很大的区别。您认为您的员工中有多少比例的职位可能会受益于高于平均水平的大学毕业生智商?人力资源人员真的能从智商中受益吗?

    • 回复: @John Jeremiah Smith
  153. @John Jeremiah Smith

    在我看来,员工的选择不应该只是一个仔细评估的过程。

    确实如此,但你必须从某个地方开始,正如你之前关于消除智商分数太低或太高的人所承认的那样。确实,如果他们都在相关领域拥有很高的荣誉学位,那么你就不会太担心智商。

    • 回复: @John Jeremiah Smith
  154. szopen 说:
    @res

    我试图找出这一说法背后的研究成果,但失败了。然而,与我交往的很多人(他们的智商很可能高于平均智商)在听到这个想法后立即爱上了这个想法。这可能是一些心理问题造成的现象(“哈!所以这就是为什么我在学校是社交隐士,因为他们都是白痴所以他们无法理解我!”),但我必须说,在文法学校我绊倒了很多次都是我无法理解的人。有一次,我的老师要求我帮助班上的一个同学做数学题(或者也许是他自己要求的?)。

    这是一个典型的坏男孩,在女孩中取得了很多成功,你知道这种类型。后来他开始贩毒,然后我听说他成了告密者,不得不逃到另一个城镇。反正。那是唯一一次,我在他眼中看到了恐惧。我正在向他解释一些琐碎的数学方程,也许是皮塔戈拉方程,或者其他什么。在第十个例子之后,我非常沮丧,我开始对他尖叫定义;然后我意识到他完全不知道我在说什么。我可以谈论喝酒、嫖娼,嘲笑他的蹩脚笑话,而且我知道我必须说什么样的笑话才能让他发笑。但他似乎无法理解最基本的抽象概念,比如如何识别三角形的哪条边是“斜边”(在波兰语中,斜边有一个非常合乎逻辑、直观且简单的名字)。然而,当我们谈论简单、具体的事情时,他看起来就像一个正常人。

    我和这个人还有很多其他的互动。女人、男人,看似正常,却无法抽离,都运用着逻辑规则。我认识的教授也很少比我聪明,我经常想知道他们是否对我有同样的感觉。

    • 回复: @res
    , @res
  155. @res

    PPS 我很好奇,如果你不考虑智商,你(集体)是如何得出以下结论的。你用什么来鉴定这 150 个人的智商?

    出于某种原因,智商得分为 130 或更高(通常是斯坦福-比奈)的个人倾向于在其申请材料中包含该信息。

    我知道这非常多 形式 查找学术职位申请中引用的智商分数。我怀疑这里一些评论者的观点可能受到那个,嗯,传统的影响。

  156. @res

    您在这里花了相当多的时间来展示您的人力资源经验和专业知识。鉴于此,我认为有必要询问一下您正在招聘什么类型的行业和职位。

    你正在准备做一些贬低的事情,这样你就可以对我的经验和观点的陈述获得个人实现的优势感。

    我没有兴趣向您提供我的工作经历的任何细节,即使您决定(当然是无私的)提供您自己的工作经历。我对向你证明什么没有丝毫兴趣。这是一个评论论坛,您可以仅从表面上理解它们,也可以根本不理解它们。

    • 回复: @res
  157. reiner Tor 说:
    @John Jeremiah Smith

    将它们放在地下室清洁办公室的桌子上,没有互联网接入。最多两周,他们就走了。

    我认为这可能是歧视诉讼的基础:性别歧视的老板把她放在地下室清洁办公室的一张桌子上,没有互联网接入(也切断了她任何晋升的希望),希望得到两周内摆脱她。

  158. @Wizard of Oz

    确实如此,但你必须从某个地方开始,正如你之前关于消除智商分数太低或太高的人所承认的那样。

    我们发现,高智商但工作经历不佳的人并不是好的候选人。

    重申:“只有表现出的优点、经验和能力才是衡量是否适合就业的适当标准,或者,对于缺乏经验的新员工来说,才能衡量能力和态度。”

    我会坚持下去。你们可以雇佣所有能找到的天才。我希望你能完成足够的工作来维持公司的业务。

  159. reiner Tor 说:
    @John Jeremiah Smith

    有人告诉我我的阅读和理解能力相当好。

    在匿名论坛或评论线程中,任何人都可以撰写或吹嘘任何内容,但吹嘘自己的阅读和理解能力是没有用的。无论如何,人们可以根据自己的评论历史得出自己的结论。

    • 回复: @John Jeremiah Smith
  160. res 说:
    @szopen

    你的经历和我有一些共同点。它导致了我成长过程中生活的一些划分(即使是现在,程度也较轻)。我有一些学术能力最差的亲密朋友,但在学校环境之外(例如,如果我们正在建造一些物理设施),我并没有发现他们的聪明程度明显低于平均水平(奇怪的是,如果有的话,有些相反)。我一直很好奇这是为什么。

    我一直发现引人注目的一件事是,当客观措施存在时(例如学校)和不存在客观措施时(例如大量工作),动态存在差异。有些人似乎完全无法识别能力/专业知识,除非有客观证据(或合理的传真,如简历项目符号)。

    关于人们爱上这个想法。它为各种不良结果提供了方便的理由。我认为对于大多数人(包括我)来说,很难冷静地思考这个想法。对于一些人来说,这是一个很好的失败合理化工具,而对于另一些人来说,这似乎是一种侮辱。

  161. res 说:
    @szopen

    我试图确定这一说法背后的研究,但失败了

    根据 Heinlein 的引用日期(Space Cadet 于 1948 年出版),我发现 iSteve 的军事研究想法(特别是美国二战时期)是合理的。但让我们再看一下。

    以下是一些着眼于智力和领导力但没有考虑追随者智力的工作: http://www.timothy-judge.com/Leader%20IQ--JAP%20published.pdf

    Dean Simonton 的这篇论文(参见 libgen)看起来非常相关: http://psycnet.apa.org/record/1986-14442-001

    群体中的智力和个人影响力:

    四种模型逐渐发展起来,为智力与个人对其他群体成员的影响之间的曲线关系提供了概念基础。模型 1 假设影响力是智力百分位数的函数,预测智商超过 120 左右的智力与影响力的关系可以忽略不计。模型 2 增加了对潜在追随者的理解程度的考虑,产生了一个非单调函数,其预测峰值 IQ 约为 108(或高于平均值 0.5 SD)。模型 3 纳入了批评因素,该因素承认群体成员容易受到智力上级的影响,从而预测第二个非单调函数,其最佳智商约为 119(或高于平均值 1.2 标准差)。模型 4 扩展了不同群体的平均智商各不相同的事实,因此预测群体平均智商与其最有影响力成员的智商之间存在高度相关性,但领导者与追随者之间的差距在 8 到 20 分之间(取决于子型号)。

    该论文还进行了一些历史讨论,包括:

    因此,McCuen(1929)推测“在民主社会中,领导者不能与群体脱离太远”(第95页)。

    麦昆工作的一个问题是他正在研究斯坦福大学的领导者(因此范围限制)。我无法找到他的作品的链接,但在很多地方都引用了它。

    西蒙顿的论文中有很多额外的细节。图3引起了我的注意。它根据模型 3 的智商来估计关注者的预期比例。峰值为 119,这与我估计的“问题”最不明显的智商 115 非常接近。

    另请参阅 FKA Max 评论,其中提到 1985 年 Simonton 论文并链接了更新的论文: https://www.unz.com/jthompson/the-secret-in-your-eyes/#comment-1820124

    这份 74 页的美国陆军文件包含一些相关信息,包括: http://www.dtic.mil/get-tr-doc/pdf?AD=ADA126852

    Hollingworth (1926) 报告称,领导者可能比他们所领导的下属更聪明。他对智商的调查表明,在平均智商为 100 的群体中,领导者的智商在 115-130 之间。此外,智商较高(即 160)的人领导平均智商为 100 的团队会遇到更大困难。智商为 160 的领导者会增加其沟通(语言能力)不被理解的可能性,而成绩较低的人则无法理解。团体。但是,如此高素质的人更有可能获得领导更高的成功,实现平均智商为 130 或更高的群体。
    ...
    因此,在领导选拔过程中,应在实际选拔之前考虑候选人的智商分数。为了体验成本效益高、富有成效的领导力,潜在领导者的智商分数将略高于他/她将领导的团队的平均分。
    ...
    Stogdill(1948)指出,领导者和追随者之间智力的极端差异不利于这项工作。 Ghsell(1926)支持这一发现以及 Holllngworth(1929)和 McCuen(1963)的发现,他发现智商得分高和低的人在领导(管理)职位上取得成功的可能性低于智商得分相同的人。中级水平。

    该论文中没有正确引用 Hollingworth (1926),但似乎是她的书《天才儿童》。本页和以下内容具有最相关的内容(请注意,这是指儿童!令我惊讶的是,军队报纸忽略了这一细节): https://babel.hathitrust.org/cgi/pt?id=mdp.39015008033394;view=1up;seq=156

    在观察不同儿童群体中谁是受欢迎的领导者时,作者发现,在其他特征有利的情况下,领导者的智力与被领导者的智力以相当可预测的方式相关。在平均智商为100的孩子中,领导者的智商很可能会落在115到130智商之间。也就是说,领导者可能比所领导的团队的平均水平聪明得多。如果在一个普通的孩子群体中,有一个孩子的平均年龄与他们的平均年龄相仿,体型较大,英俊,可亲,勇敢,大方,坚强,智商在115到130之间,这样的孩子很可能是一个领导者(适当考虑与性别有关的领导力的社会态度)。然而,智商超过 130 时,在上述群体中成为领导者的机会似乎会下降,直到智商超过 160 时,孩子成为受欢迎的领导者的机会就很小了。在平均智商为 130 的群体中,智商高达 160 的孩子很可能处于领先地位,因为这样的群体所忠诚的洞察力高于平均群体的洞察力,其他特征也很有利。

    PS Santoculto(和其他人),如果您还没有看过西蒙顿关于创造力的书籍,您可能想看一下。这是一个例子: https://www.amazon.com/Origins-Genius-Darwinian-Perspectives-Creativity/dp/0195128796
    汤普森博士在他的博客(和评论)中引用了西蒙顿的一些内容,但我没有看到明确提到的创造力书籍。我认为本文中引用的一段话值得在这里重复: https://www.unz.com/jthompson/asians-bright-but-not-curious/

    Simonton (1998, 2009) 的分析发现,更多的原创性思考者在变态人格方面具有中等偏高的特征,例如低责任心和低宜人性,而这些特征与非常高的智力相结合。

    • 回复: @Santoculto
  162. @reiner Tor

    在匿名论坛或评论线程中,任何人都可以撰写或吹嘘任何内容,但吹嘘自己的阅读和理解能力是没有用的。无论如何,人们可以根据自己的评论历史得出自己的结论。

    哈哈。人们可能会根据当前的空气温度和湿度得出自己的结论,尽管它会产生不同的结果。

    我一直说“这是我的经验,我的观点”,人们一直坚持要求我记录我的专业知识和我的正确性,以及我的理论和证明的链接。

    这让我很兴奋。

    • 回复: @reiner Tor
  163. res 说:
    @John Jeremiah Smith

    我在本博客的其他评论中提供了有关我的背景的足够信息。我认为我的问题与我明确指出的原因相关。你拒绝回答就很说明问题了。

    重复一下原因:

    雇佣教授(就像绿野仙踪那样)和雇佣扫帚推手之间有很大的区别。您认为您的员工中有多少比例的职位可能会受益于高于平均水平的大学毕业生智商?

    PS 鉴于您上面模糊地讲述了您的背景(恕我直言,这是明显的权威主张),我发现您的第一句话非常有趣。谢谢你的笑声。

    • 回复: @John Jeremiah Smith
  164. Santoculto 说:

    即使女性在人文学科中的优势也值得怀疑。

  165. reiner Tor 说:
    @John Jeremiah Smith

    我没有写任何关于你的观点或专业知识的价值的文章。我只是指出,吹嘘这里很容易观察到的东西(你的阅读理解能力肯定属于这一类)是无用的练习。如果别人同意你的观点,那就是多余的,否则就是徒劳。

    • 回复: @John Jeremiah Smith
  166. @res

    我在本博客的其他评论中提供了有关我的背景的足够信息。我认为我的问题与我明确指出的原因相关。你拒绝回答就很说明问题了。

    哦,我回答了。几次。但是,请务必提供您的姓名、地址和电话号码。我会打电话告诉你你需要知道的一切。

    • 回复: @res
  167. @reiner Tor

    我没有写任何关于你的观点或专业知识的价值的文章。我只是指出,吹嘘这里很容易观察到的东西(你的阅读理解能力肯定属于这一类)是无用的练习。如果别人同意你的观点,那就是多余的,否则就是徒劳。

    这纯粹是胡说八道。

  168. Santoculto 说:
    @res

    是的,我认为艾因森克已经谈到过,高度创造力的人往往在精神病方面得分更高。嗯,精神变态者和反社会者往往在他们的掠夺或寄生策略上非常有创造力。我喜欢认为高度创造力具有共鸣的心理特征,在几乎所有心理维度上都过于敏感,即在艺术/文化存在象征氛围上。

    全向性,而不是真正的两向性。真正的中间性格可能情绪过于稳定。全向性[荒谬的推测]可能就像中间性[“情绪稳定性增强”]一样,但具有矛盾的更高的情绪敏感性。不要将“情绪不稳定”与“情绪敏感”混淆或混为一谈。一个必然不是同一件事。

    就我而言,我的情绪非常敏感,毫无疑问,我可以模仿一个真正情绪不稳定的人,但最初来说,我并不是情绪不稳定。

  169. res 说:
    @John Jeremiah Smith

    哦,我回答了。几次。但是,请务必提供您的姓名、地址和电话号码。我会打电话告诉你你需要知道的一切。

    有趣的。鉴于您之前的声明,这是一件非常奇怪的事情:

    我没有兴趣向您提供我的工作经历的任何细节,即使您决定(当然是无私的)提供您自己的工作经历。我对向你证明什么没有丝毫兴趣。这是一个评论论坛,您可以仅从表面上理解它们,也可以根本不理解它们。

    感谢您清楚地表明支持您的各种不受支持的断言的实质内容是多么少。

    • 回复: @John Jeremiah Smith
  170. @res

    你们这些患有短角综合症的人让我很开心。你的不足感给你带来了什么?一场生死攸关的辩护,为你的荒谬重要性辩护,即考试中百分之七的分数(相当可疑的优点)意味着你的自我受到保护,免受那些讨厌的女孩的伤害。你会不遗余力地确保这个毫无意义的统计数据。你会撒谎、欺骗、侮辱、欺骗、掩饰和愤怒地尖叫。

    德迪奥斯夫人,你真可怜。也被封锁了。再见了,小男孩。

    • 巨魔: szopen
    • 回复: @res
    , @reiner Tor
  171. res 说:
    @John Jeremiah Smith

    Ad hominems——对与你争论的人说的最好的方式——“你赢了。”

    • 同意: Daniel Chieh
  172. anon • 免责声明 说:

    “我只能假设,一个智商为 160 的水管工会比一个智商为 159 的物理学家更喜欢核物理讲师的职位?天哪,听起来真是一个糟糕的计划!让我们开始吧!”

    https://en.wikipedia.org/wiki/Leonard_Susskind

    伦纳德·苏斯金德(Leonard Susskind,1940年出生)[2][3]是一位美国物理学家,斯坦福大学理论物理学教授,斯坦福理论物理研究所所长。 …… 他16岁时开始担任水管工=

  173. reiner Tor 说:
    @John Jeremiah Smith

    考试中百分之七的分数的概念

    这对你的阅读理解能力有何影响?没有人在这里写到 0.07 分的差异是有意义的。

    当然,你也有可能只是不诚实。

    • 回复: @John Jeremiah Smith
  174. @James Thompson

    您可能会对 ABC 健康报告的采访感兴趣,只要它表明环境在足够广泛的意义上(包括技术和教学方法)可能会影响我们神经网络的硬件或固件。这是一种夸张的解释吗?我想知道它与弗林效应有何关系。

    http://www.abc.net.au/radionational/programs/healthreport/childhood-literacy/9031078#transcript

    • 回复: @James Thompson
  175. @Wizard of Oz

    如果她有一些研究可以看的话那就没问题了。你可能看过我之前关于阅读的一些帖子
    https://www.unz.com/jthompson/dyslexia-dilemmas-are-your-shortcomings

  176. @reiner Tor

    这对你的阅读理解能力有何影响?没有人在这里写到 0.07 分的差异是有意义的。

    是的,他们做了。不是你,真的。然而,你的 广告人身攻击 愚蠢的行为将被忽略。这与个人无关——祝你生活愉快。

    • 回复: @szopen
    , @reiner Tor
  177. res 说:
    @Wizard of Oz

    与成功地对计算机进行编程以下国际象棋或围棋一致,显然可以对它们进行编程以进行全套智商测试。但这并不是我建议的那么简单的程序。我们需要的是一个硬件+软件,尝试模拟人脑的过程,然后释放一系列智商测试。

    这篇热门文章讨论了最近一篇讨论人工智能“智商”的论文(并提供了链接): https://boingboing.net/2017/10/10/study-measuring-iq-of-various.html
    研究测量各种人工智能的智商,谷歌的智商为 47.28

    如果有人看这篇论文,看起来他们在表 1 中调换了 2 级和 3 级,还是我读错了?

    他们目前将谷歌的人工智能评为4级,5级的定义特征超过4级是非零C,代表知识和信息的创新和创造。 6级本质上是智力奇点; )

    该论文的第一个参考文献提供了有关 2014 年版本测试的更多详细信息: http://www.sciencedirect.com/science/article/pii/S1877050914005389

    相关说明中有一部关于 AlphaGo 的新纪录片: http://www.businessinsider.com/alphago-documentary-provides-a-rare-look-inside-deepmind-2017-10
    它今天和明天在伦敦上映。

    • 回复: @Wizard of Oz
  178. szopen 说:
    @John Jeremiah Smith

    出于好奇,谁干的?

    在评论 57 中,林恩教授(汤普森引用)回答了有关卡马拉塔和伍德科克研究的问题,但没有说这个差异(由卡马拉塔和伍德科克发现)是否有意义。 Lynn 发现了 4 个 IQ 点(几乎是 1/3 SD,所以差异相当大),这就是我们正在讨论的差异。

  179. @szopen

    打算在随后针对林恩论文的一些批评的帖子中讨论这一点(卡马拉塔和伍德科克)。效应大小并没有那么大,更像是 0.15,但由于男性样本不足而变得复杂。

  180. @szopen

    Lynn 发现了 4 个 IQ 点(几乎是 1/3 SD,所以差异相当大),这就是我们正在讨论的差异。

    嗯。去过那里,看到过。我可以保证,不用担心任何矛盾,你——没有人——可以与两个智商相差 4 分的人一起工作数月或数年,并注意到“智商”所定义的心理能力的任何差异。

    这是唯一真正的观点(我确实很欣赏汤普森先生收入的一部分来自于争论本身的延续,无论结论与否)。如果对某些定义的“智力”(定义是 绝对 对智商科学至关重要)始终显示男性和女性之间存在 0.07 分的差异,或者就此而言,4 分,这有什么区别?

    文明受到怎样的影响?将实施哪些变革?谁将做出这些改变,他们将如何实现这些改变?你真的相信国会会正式声明“小鸡都是愚蠢的”吗?因此,女性将被禁止在整个行业和政府中担任所有监督和管理职务?

    • 回复: @szopen
    , @utu
    , @AP
  181. szopen 说:
    @John Jeremiah Smith

    如果对某些定义的“智力”(定义对智商科学绝对重要)进行的实质性综合“测试”始终显示男性和女性之间存在 0.07 分的差异,或者就此而言,4 分,那么这有什么区别呢?

    正如我已经在同一个线程中写过两次(两次都回复了你),
    0.07 没什么。 4 IQ 点意味着,如果用 IQ 来衡量智力,那么男性在顶层的比例往往会过高,因此试图对男性和女性实行严格的 50/50 比例是错误的。尾部的差异是惊人的:根据假设,我们可以猜测智商超过 2 的人中有近 3/130 是男性。

    你真的相信国会会正式声明“小鸡都是愚蠢的”吗?因此,女性将被禁止在整个行业和政府中担任所有监督和管理职务?

    现在我认为你要么是恶意行事,要么有严重的阅读理解问题。别再造稻草人了。尝试与真实的对手讨论并攻击真实的论点而不是想象的论点。

    祝你生活愉快。我不会再回复你了。

    • 同意: res, Daniel Chieh
    • 回复: @John Jeremiah Smith
  182. utu 说:
    @John Jeremiah Smith

    继续努力吧。不要后退。与高智商的共青团积极分子打交道时,你需要加倍努力。

    • 回复: @John Jeremiah Smith
  183. AP 说:
    @John Jeremiah Smith

    我可以保证,不用担心任何矛盾,你——没有人——可以与两个智商相差 4 分的人一起工作数月或数年,并注意到“智商”所定义的心理能力的任何差异。

    正确的。但这并不是这种差异(如果它确实存在的话)如此重要的原因(我并不完全相信)。

    文明受到怎样的影响?将实施哪些变革?谁将做出这些改变,他们将如何实现这些改变?

    正如其他人指出的那样,平均 4 个百分点的差异会导致较高端的显着差异,例如,智商水平为 130 2/3 的人是男性,但只有 1/3 是女性。因此,如果存在4分的平均差异,那么某些需要足够高智力才能获得这样分数的职业,由于自然原因,男性会多于女性,而不是某种歧视。了解这一点并考虑到这一点可能会影响政策。

  184. utu 说:
    @AP

    较高端的差异,例如,智商水平为 130 时,2/3 的人是男性,但只有 1/3 是女性

    是的,你可以进行这样的计算,但它们的有效性如何?男性和女性人口标准差的不确定性是多少?它们与正态分布的近似程度如何?估计标准差需要比估计总体平均值大得多的样本。

  185. @szopen

    现在我认为你要么是恶意行事,要么有严重的阅读理解问题。别再造稻草人了。尝试与真实的对手讨论并攻击真实的论点而不是想象的论点。

    祝你生活愉快。我不会再回复你了。

    稻草人?哈哈。那不是稻草人。说真的,有了所有这些“布罗德是愚蠢的”科学,嗯,研究结果,你打算用它做什么?什么?任何事物?这是纯粹的“天使在针头上跳舞”的东西。

    你不回复我就很舒服了。您似乎对其他话题确实有一定的了解,所以我会偶尔回复。忽略它即可。

  186. @utu

    继续努力吧。不要后退。与高智商的共青团积极分子打交道时,你需要加倍努力。

    双倍下注?呐……这就像与女子长曲棍球队踢橄榄球一样。

    我承认,当我看到那些“小鸡们傻了”的人时,我会笑得很厉害,以至于我几乎无法打字。

  187. reiner Tor 说:
    @AP

    事实上,对于足够大的样本,即使是 4 个点的差异本身也可能是显着的。假设差异显着(即不是测量误差),我们可以预期 102 IQ 组(由智商恰好为 102 的人组成)比 98 IQ 组(由智商恰好为 98 分的人组成)表现更好。差异不会很大,但如果样本足够大,就会显现出来。如果只有两个人,一个 98,另一个 102,你将无法注意到这一点(通常智商 98 的人实际上在工作中表现得更好),但随机变化会在足够大的样本中相互抵消,并且智商102的组会更好。请注意,差异不会很大,收入相差几个百分点(甚至只是十分之几个百分点),升职的机会更高一点,上大学的机会更高一点(对于两个组来说都非常低,但对于 102 组来说稍微高一些),或者类似的东西。

    但是,是的,真正的意义在于尾部。

    • 回复: @utu
  188. @AP

    因此,如果存在4分的平均差异,那么某些需要足够高智力才能获得这样分数的职业,由于自然原因,男性会多于女性,而不是某种歧视。了解这一点并考虑到这一点可能会影响政策。

    假设男女智商差异理论成立,并且有证据表明在现实生活中存在微小的差异,那么人类也许确实能够在 50,000 年间表现出不可预测的社会和文化特征。多年来,我对这种可怕的差异一无所知。并且幸存下来,更令人惊奇。

    所以…… 可以 影响政策?您的意思是“应该”吗?鉴于最近发生的事件,请问您认为任何人、任何地方、任何权力或责任职位实施此类“政策”的可能性有多大?

    • 回复: @reiner Tor
    , @AP
  189. reiner Tor 说:
    @utu

    众所周知,由于遗传原因,男性的变异一定更高。

    • 回复: @utu
  190. @utu

    估计标准差需要比估计总体平均值大得多的样本。

    好吧,如果做得好,是的。当然,人们可以依靠小样本的统计数据,并以此为基础进行即兴发挥。这种方法的一个问题是,这种男性/女性的大惊小怪只是对厌恶女性主义的一种认可——真正的努力仍然集中在“黑鬼是愚蠢的”、“墨西哥是愚蠢的”等,同时放弃了有问题的“亚洲人不是”哑的”。

    当亚洲女性的智商测试显示她们的平均智商比亚洲男性高 4 分,即比白人男性高 8 分时,会发生什么?或者,哎呀,他们比亚洲男性低 4 分,因此比白人男性高 4 分?

    哇哇。

    • 回复: @reiner Tor
    , @utu
  191. utu 说:
    @reiner Tor

    你的意思是方差而不是变异。但我不明白为什么我们应该知道男性的方差应该更高。如果有的话,女性有 XX 而不是 XY,因此她们比男性有更多的遗传物质来进行改变。

    • 回复: @reiner Tor
    , @res
  192. reiner Tor 说:
    @John Jeremiah Smith

    然而,五万年来,人类一直以不可预测的社会和文化特征行事,但对这种可怕的差异一无所知

    这是非常错误的。在女权主义出现之前,大多数人类——无论男性还是女性——都清楚地知道,具有杰出能力或有能力担任领导职位等的男性远远多于女性。事实上,如果有的话,人们可能低估了历史上女性的能力。

  193. reiner Tor 说:
    @John Jeremiah Smith

    或者,哎呀,他们比亚洲男性低 4 分,因此比白人男性高 4 分?

    东亚人可能比白人高 5 个百分点。因此,如果亚洲女性比亚洲男性低 4 个百分点,就像白人男性和女性之间的差异一样,那么亚洲女性平均只比白人男性高 1 个百分点。

  194. szopen 说:
    @utu

    这是一个有效的观点;我不知道。我所知道的是,似乎男性在许多不同的特征上确实存在更大的差异,包括身高等琐碎的事情,这意味着男性和女性在智商上的标准偏差也可能不同,事实上,我读过一些研究已经发现了这一点。我们还对非常大的样本进行了研究(例如,苏格兰有 80.000 名儿童,这意味着几乎所有儿童都出生在该特定群体中)——这些样本显示平均智商没有差异(但请记住——测量值为 11,林恩提出男孩发育较慢)因此,假设的男性优势在十一点时就已经不明显了),但方差有显着差异(也与正态分布有一些偏差)。

    您还可以在我们东道主的博客上看到罗马尼亚的相当大的样本(15.000 人):

    https://www.unz.com/jthompson/no-sex-differences-in-romania/

    事实上,我更倾向于相信“男性的变异性更大”,而不是“男性的平均值高 4 个百分点”。在我看来,对前者的支持比对后者的支持要强烈得多。正如一位研究人员指出的那样,年龄较大的男性看似更高的优势也可能是自然减员的结果(即更多分数较低的男性停止接受测试)。

    • 回复: @utu
  195. utu 说:
    @reiner Tor

    通常智商98的人

    我经常在这些页面上看到“x IQ person”之类的短语。在我看来,这往好了说是思想的草率,往坏了说是滥用语言。被称为“x IQ 人”的动物没有本体现实。你必须假设它。但你甚至无法测量它。智商测试的不确定性太高,无法判断这个人的智商是 x 或 y。您充其量可以说某个人在 17 年 1989 月 0.9 日进行的此类测试中达到了以下 x IQ 分数。指定日期很重要,因为一年后进行的测试将有 Δx 的不同,其中 Δx 可以是相当大。实际上,从 6 Δx 的重测相关性来看,标准差大约有 XNUMXIQ 点。

    像“x IQ person”这样的短语的使用是具体化和超越合法本体框架边界的例子。它暴露了那些要么被操纵而相信智商崇拜的人,要么那些正在操纵他人的人。

  196. utu 说:
    @John Jeremiah Smith

    在某些方面我同意你,但我不喜欢你。太傲慢了。都是帽子不牛。如果你能降低你真实的或假装的自我满足和对自己的崇拜,也许你可以腾出一些精力来提出有效的论点。

    • 回复: @John Jeremiah Smith
  197. reiner Tor 说:
    @utu

    是的,方差。然而,您对 XX 染色体的想法是错误的。

  198. reiner Tor 说:
    @utu

    我已经明确说过我假设没有测量误差。当然,你不能如此确定地测量一个人的智商,至少这是相当困难的。

    然而,对于大样本,您可以非常准确地测量,因为随机测量误差应该相互抵消。正如我所说,这具有一定的意义 即使没有差异.

  199. utu 说:

    然而,您对 XX 染色体的想法是错误的。

    谢谢你承认我在想,但很抱歉,我无法回报。

  200. reiner Tor 说:
    @utu

    被称为“x IQ 人”的动物没有本体现实。

    假设智商测试确实测量了某些东西,我们通常称之为智商,那么它确实可以测量。也许我们无法足够准确地测量它。

  201. @utu

    在某些方面我同意你,但我不喜欢你。太傲慢了。都是帽子不牛。如果你能降低你真实的或假装的自我满足和对自己的崇拜,也许你可以腾出一些精力来提出有效的论点。

    “喜欢”?这对我来说没什么问题。从过去的经验来看,我也不喜欢你。喜欢预科学校。不,谢谢。

    至于有效论据和智商神秘主义……什么有效论据?你见过智商爱好者承认有效的论点吗?我没有。

    事情的简单事实是:智商测试可能有价值。如果他们有优点,那么如何处理结果呢?这可以做到吗?即随后规定的这种“合乎逻辑的”且可能是法律的行动是否可以实施?

    出色地?他们可以吗?一定不行。在任何可以想象的未来情况下,你能否想象一个女性、黑人和拉丁人被排除在权力和责任职位之外的世界,因为一些可疑的测试表明她们本质上是低等的?他妈的不!

    跨种族、跨性别的智商比较研究还能继续吗?当然。你想付钱吗?我不。也许是一个非营利中心?好想法。哎呀,它的价值至少和那些被视为古代艺术品的五彩块状岩石收藏一样值钱。人们会为任何该死的东西付出代价。但不是我,也不是我的税。

  202. utu 说:
    @szopen

    包括诸如身高之类的琐碎事情

    男性的平均身高更高,因此标准发展结果也更高。当你想到它时,预计会更大。增长过程更类似于缩放,是乘法而不是加法。但如果男性的相对标准差/平均值仍然较大,那么解释就会更加复杂。

    苏格兰有 80.000 名儿童 or 罗马尼亚(15.000 人)

    似乎有足够大的样本来制作像样的直方图来确定男性和女性的分布有何不同。完成了吗? std.dev 的估计。对于小样本来说,对少数“异常值”非常敏感,因此在没有看到可靠的直方图的情况下,关于男女标准差之间存在巨大差异的故事并不十分可信。

    一位爱因斯坦在这里声称,男性的 SD 大于女性的事实(如果这是事实)可以简单地用 XY 与 XY 来解释。检查一下他。

    测量时为 11 岁,林恩提出男孩发育较慢

    不同年龄段的智商问题值得关注,因为它揭示了智商崇拜中的一些隐藏假设或者更确切地说是恶作剧。不同年龄段的智商是相同的 前定义。这是一个假设,而不是经验事实。例如,相同的 Raven 矩阵可以应用于不同的年龄组,但它们对于每个年龄组的缩放不同。缩放比例的差异从何而来?从假设得出平均智商和标准差。每个年龄段都一样。因此,当您听到(正如我们经常听到的那样)智商稳定且不会随年龄变化而变化时,这就是原因之一。然后,当同样的人对遗传力进行估计时,估计从儿童时期的 20% 变为成年后的 80%,他们没有意识到他们与智商稳定的说法相矛盾。双胞胎的智商必须从童年到成年趋同,以解释 20% 到 80% 的变化。这些人会声称童年时期的智商是成年期结果的一个很好的预测指标,却忘记了这会因遗传力的不同而带来不便的结果。人们可以解开许多这样的主张以及它们提出或公开的原因。你提倡先行计划,然后就会通过遗传性研究来打击你。你想要倡导一些更加平等的社会,他们会用智商的稳定性来打击你。总有政治出身的不可告人的动机。那么,当汤普森在这里普及林恩的出版时,人们应该如何看待它呢?首先询问政治权宜之计。这些人的正直是橡胶做的,但他们的偏见是铁铸的。有人被谷歌解雇了,对吧?因此,阿尔斯特研究所的工作人员前来救援。

  203. Santoculto 说:
    @utu

    智商是人们一生中[认知上]保持不变的事物的总体总结。

    例如,如果您一直擅长语言/词汇,那么语言智商测试可能会显示您的语言灵活性。

    不要忘记子测试。

    这解释了为什么许多语言能力较低的人往往有一些单词总是以“不正确的方式”[基于常规形式的不正确]书写。

    我们甚至可以将智商描述为“认知数量恒定性”。

  204. 哦,亲爱的,审查制度。啧啧,汤普森先生,啧啧。

  205. res 说:
    @utu

    如果有的话,女性有 XX 而不是 XY,因此她们比男性有更多的遗传物质来进行改变。

    你这么自信地说出这样的话(还有评论208,哎呀)。您可以考虑查看此页面: https://en.wikipedia.org/wiki/X-inactivation
    基本思想是每个雌性细胞中都有一条 X 染色体被沉默。最终结果是一种平均。

    还值得考虑色盲的遗传学。

    • 回复: @res
  206. res 说:
    @res

    如果有的话,女性有 XX 而不是 XY,因此她们比男性有更多的遗传物质来进行改变。

    由于 utu 对这个想法的投入超过了一倍,让我们进一步研究一下。

    http://languagelog.ldc.upenn.edu/myl/llog/Turner1996.pdf

    因此,具有单条 X 染色体的男性可能更容易受到 X 染色体上有利基因或有害突变事件的影响,这可能解释了两性之间智商分布的差异。

    我希望很明显这会增加男性的变异(以及方差,即 SD)。

    https://www.psychologytoday.com/articles/201109/the-incredible-expanding-adventures-the-x-chromosome

    事实上,男性只有一个 X,且独特地源自母亲,这一事实对智力差异具有进一步的影响。这样看:如果你是一位高智商母亲的儿子,并且 X 染色体确实对智商有重要贡献,那么你将表达你唯一的一条母亲 X 染色体,而不会像女性那样被第二条 X 染色体稀释。继承。这种影响是双向的:如果你是一名男性,你的 X 染色体上有一个与智商相关的基因受损,你将比女性更明显地受到影响,女性可以从她的第二条 X 染色体上表达同等的、未受损的基因。这本身就可以解释为什么智商非常高和非常低的男性比女性多:男性的单条 X 染色体会增加智商的差异,仅仅是因为没有第二条补偿性 X 染色体。

    https://www.ncbi.nlm.nih.gov/pubmed/26161735

    摘要:有大量证据表明,男性的一般智力比女性的变化更大。近年来,研究人员认为,尽管一般智力的平均性别差异很小(如果有的话),但男性在总体智力分布的两端往往都过多。部分解释可能是 X 染色体上存在一些基因,这些基因与涉及精神发育迟滞的综合征性疾病以及正常发生的一般智力的群体变异有关。 X 染色体上的基因在已知与智力低下有关的基因中似乎过多,这与我们开发的一般智力人口分布模型是两种正态分布的混合模型一致的。使用这个模型,我们探索了分布中不同点的男性与女性的预期比率,并估计了 X 染色体上的基因可能导致的一般智力差异的比例。这些估计提供了线索,说明基于生物学的性别差异在环境中可以在多大程度上表现为智力能力的性别差异。我们在特定认知能力的性别差异和性选择进化理论的背景下讨论这些观察结果。

    他们进行了深入的讨论,任何对此感兴趣的人可能都值得一看。

    注意红绿色盲的计算。男性的方差是女性的 10 倍,患病率是女性的 20 倍。

    以下是观察到的智力方差差异的一些数字(比率为 1.13 和 1.19)。

    斯特兰德等人。 (2006) 使用认知能力测试(CAT;Lohman 等人,2001)对样本进行评估。该测试测量言语、定量和非言语推理领域,因此大致平行于通常被认为是最基本的特定认知能力领域(例如,Johnson & Bouchard, 2005;Snow, Corno, & Jackson, 1996;Snow & Lohman, 1989;Vernon,1964;但非语言测试特别避免了空间心理旋转问题)。每个领域都通过三个不同的测试进行评估。计算每个领域和总体的平均分数。与苏格兰医学调查一样,男性的 CAT 总体变异性大于女性;男女方差比为 1.13,与 SMS32 中的结果相同。 SMS47 中的方差比稍大:1.19。分布的顶部和底部 5% 和 10% 中的男女尾部比例也与苏格兰医学调查中的相似。表 2 显示了这种比较。在 CAT 和 SMS47 数据中,一般智力分布低端的尾部比例比高端的尾部比例更大。相比之下,SMS32 中的尾部比例实际上在一般智力分布的高端比在低端更大。

    表 2 给出了五个测试的顶部/底部 5/10% 的尾部比率。

    • 回复: @Daniel Chieh
    , @utu
    , @utu
  207. AP 说:
    @John Jeremiah Smith

    那么……可能会影响政策吗?您的意思是“应该”吗?鉴于最近发生的事件,请问您认为任何人、任何地方、任何权力或责任职位实施此类“政策”的可能性有多大?

    好吧,如果很清楚智商在2以上的人中有3/130是男性,1/3是女性,那么,比如说,2/3的博士级物理学家是男性的情况就无法理解了因此,旨在克服偏见或歧视的政策将不会得到实施,因为不再有理由认为偏见或歧视是造成差异的原因。

    当然,发生这种情况的可能性并不高,但也不是零。无论如何,了解正在发生的事情是件好事,你不觉得吗?

    当然,这似乎还没有得到明确证明。

  208. 当然,发生这种情况的可能性并不高,但也不是零。无论如何,了解正在发生的事情是件好事,你不觉得吗?

    这是毫无意义且最终令人沮丧的练习。知道什么是真实的而不得不忍受虚假的东西只会带来痛苦。

    我重申:在任何可以想象的未来情况下,你能否想象一个女性、黑人和拉丁人被排除在权力和责任职位之外的世界,因为一些可疑的测试表明她们本质上是低等的?

    当然不是。那艘船的船体已经倒在地平线上。

    • 回复: @Daniel Chieh
    , @AP
  209. @res

    感谢您的评论和链接。正如我打算对 JT 所说的(在某种程度上)回答他关于阅读障碍的联系,也许会成功,我想知道,通过发现某些人在一些高度高兴的测试中的高分或平均分是耦合的,可以对大脑了解什么在其他格莱登测试中得分非常低,完全不同于正常人或标准模型。知道大脑图像是否可以指出一些高智商或只是非常有能力的聪明人(例如工作记忆差或阅读障碍)的解决方法将是很有趣的。

  210. @res

    老实说,对于乌图来说,这是一座糟糕的山丘,不能选择死在上面。

    • 同意: res
  211. @John Jeremiah Smith

    嗯,一方面,中国在政府中没有很多女性,尤其是担任较高职位的女性,如果发现有理由不增加更多女性,那么她们改变这一现状的动力可能会大大降低。

    世界的尽头不是美国海岸,yanno。

  212. AP 说:
    @John Jeremiah Smith

    在任何可以想象的未来情况下,你能否想象一个女性、黑人和拉丁人被排除在权力和责任职位之外的世界,因为一些可疑的测试表明她们本质上是低等的?

    没有人声称会根据此类研究的结果制定某种配额。这是不道德的,并且反映出对结果含义的愚蠢缺乏理解。但是,如果事实证明歧视并不是导致该领域女性人数减少的原因,那么此类研究可能会阻止或限制旨在提高某些职位女性比例以补偿歧视的“平权行动”政策。如果显示 2/3 智商为 130 或以上的人是男性,这并不意味着需要这种智商的工作就会有法定配额,要求该领域的工人 2/3 为男性。这只是意味着,如果在没有任何干预的情况下,2/3 恰好是男性,则不应该采取任何措施来改变这种情况,因为我们知道这反映了每种性别的自然能力。如果 9/10 恰好是男性,政策干预的目标可能是使其达到 2/3,而不是 50/50。

    (再次重申,我并不是对林恩的 4 分差距主张是否正确采取任何立场,而是描述他是否正确的含义)。

    • 回复: @James Thompson
  213. 嗯,一方面,中国在政府中没有很多女性,尤其是担任较高职位的女性,如果发现有理由不增加更多女性,那么她们改变这一现状的动力可能会大大降低。

    是的,他们确实这么做了。相对而言,较高级别的数量与美国一样多,而较低级别的比例则更多。

    这不是重点。你对“智商研究人员”关于剥夺女性在政治中平等发言权的报告有何看法?

    不过没关系。我明白。当知识殿堂的官方言论最终证实女性是愚蠢的时候,所有这一切都将改变。一个勇敢的启蒙新时代即将来临。

    • 回复: @reiner Tor
  214. utu 说:
    @res

    感谢您提供链接和信息。这些都是推测性的讨论。基本上是挥手。这是一个典型的例子,人们手上有很多时间,但具体知识却很少,却可以编出长篇故事和假设性解释。只有按照许正在做的事情构建一些智商的多基因评分(线性或非线性)预测函数,其中存在男性中缺失的术语,这些术语可能会产生负面或正面影响,但又非常聪明,才能解决这个问题它们会导致女性 SD 缩小。如果你尝试想象这样一个函数,你就会意识到这不是一件小事。如果它只是一个加性函数,那么它的形式将是 H=M+(WM),其中 H、M、W 分别代表人类、男人、女人。那么,只有当 M 和 (WM) 之间的协方差为负且大于 WM 方差的 1/2 时,男性的 SD 才会大于女性。我认为从数学上来说这并非不可能,但我很难接受它。人们也可以尝试写出一个非线性函数,但这只是太笼统、太不受约束的问题。这正如我所愿。但除此之外只是挥手。请记住,如果人们在期刊上发表的论文的名字听起来非常重要和严肃,而这些论文是由其他与他们一样的人编辑和审阅的,这些人与他们有着相同的信仰,有相同的兴趣,如社会、财务和地位。不排除 BS 出现在这些期刊中的可能性。相反,它确保了 BS 会出现在这些期刊上,因为这是一个裸体主义的皇家殖民地,许多国王赤身裸体行走,并互相保证他们有漂亮的衣服。

  215. utu 说:
    @res

    我看了温迪·约翰逊等人。我不得不承认,这不仅仅是我在之前的评论中假设的挥手。这也意味着 Hsu 的 IQ 预测函数对于男性和女性必须具有不同的形式,因此如果将其包装到一个函数中,它将是非线性的。不管怎样,谢谢你的链接。

    • 回复: @res
  216. Santoculto 说:

    首先,不幸的是,统治我们的人是自我选择或自我提升的。权力,特别是这种类型的权力会吸引某些类型的人的选择。即使在民主国家,那些自认为是优秀政治家或想要权力的人也不能代表具有最佳特征的最佳人民来统治我们。政治是一个神奇的地方,人们并不是根据其真正的执政能力来选择的,这与智慧密切相关。是的,男性政治家已被证明不太擅长管理社会,但这并不是先验的,因为他们是男性,对女性政治家也可以得出同样的结论。

    在理想的情况下,白人,即西欧白人,并不比其他人群更好地治理。但他们的平均水平仍然更好。这不是在政治上比较好与坏、白人与非白人,而是比较好白人与不好的非白人。严格来说,政治并不是根据质量来选择的。解释第三世界国家低发展的原因之一是其精英/政治阶层的智力和道德素质。当然,我并不是说没有优秀的非白人政客,而是说他们往往比白人更稀有,而且已经更稀有了。政治需要社交技能,例如魅力和/或统治力。这是一个马戏团,政治家是根据他们的名人技能来对待和选择的。

  217. reiner Tor 说:
    @utu

    顺便说一句,智商并不是唯一可能随时间变化的因素。例如,身高是一个非常稳定的数字,但多年来也会发生变化。大多数人在 15 岁到 20 岁之间(通常在 18 岁之前)停止生长,但有些人的身高(例如你的真实身高 - 我的身高在 1 岁到 24 岁之间增加了大约 32 厘米)可能会在以后略有增加(就我而言,可能是由于进行运动并获得更好的姿势)。通常(但不是以统一的方式)随着年龄的增长,它也会减少,至少对许多人来说是这样。它还根据一天中的不同时间而变化(就像大多数人在深夜比清晨更短),因此您无法以完全精确的方式测量它(或者至少这是相当困难的),但尽管如此在欧洲我们对“身高174厘米的人”和“身高175厘米的人”这样的概念感到满意,尽管有些人早上测量的身高是174,晚上测量的身高是175。我们还了解到,20 岁时较高的人到 65 岁时会继续较高(尽管可能存在一些个体差异,特别是在数字接近的情况下,有些人身高下降,而另一些人则没有,或者幅度不大) )。也很容易理解,可能存在一些小的测量误差(1厘米当然不是不可能的)。但仅仅因为可能存在一些模棱两可的情况就抛弃身高的概念(或“174厘米高的人”)是不合理的。

    • 回复: @utu
    , @James Thompson
  218. reiner Tor 说:
    @John Jeremiah Smith

    这位评论者似乎并不理解“知识本身”的概念,也不理解在大多数情况下知识是有用的,并且它确实总是影响决策这一事实。推行许多人都知道愚蠢的政策总是比只有少数人知道愚蠢的政策更困难。同样,如果知识是明确且无可争议的,那么反对意见会比只是基于轶事证据的模糊感觉更强烈。

  219. @AP

    是的,这就是重点。这取决于表明性别之间的标准差和平均差异的数据是正确的,尽管第一个已经相当确定,但第二个可能不太确定,但至少有可以争论的材料。

    • 回复: @res
  220. utu 说:
    @reiner Tor

    谁早上测174,晚上测175 – 我一直以为我们晚上会变短。

    身高和智商之间存在很大差异。如果你在 1000 年代在英国找了 1920 个人,让他们并排站着,那么你的右边是一个不矮的人,左边的一个人不高,然后决定第 500 个人(中位数)行的高度等于 100 个高度点 (HP) 前定义 然后在接下来的 60 年里继续这样做,始终假设英国中间人的身高等于 100 HP,直到某个名叫弗林的人指出这些方案掩盖了某些东西,也许只有那时我们才能使用身高类比。

    但是,是的,我理解你的观点。

    • 回复: @reiner Tor
  221. @reiner Tor

    在怀特岛人口研究中,一两个月内儿童身高的重测低于韦克斯勒儿童智力测试的六个月重测。

    • 回复: @dearieme
  222. res 说:
    @James Thompson

    我认为元点是,群体之间在能力或偏好上可能存在差异,这对于工作的代表性等很重要,这一点也很重要。以达莫尔事件为例,我认为观察到的能力(最显着的是空间技能)和偏好(最显着的是人与事物)方面的性别差异足以解释大部分(全部?)技术工作。此外,偏好既是相对的又是绝对的。一个非常有能力的女性很可能会选择成为一名医生并更多地与人打交道,而不是成为一名程序员并在电脑前度过一天——即使后者非常适合后者。

    因此,自动得出不同的代表性意味着正在发生不当歧视的结论是不合理的。尽管这应该被视为一个可能的解释假设。

    但大多数人似乎都不想考虑这种可能性。因此,甚至不可能对此进行合理的对话(该线程中有大量证据,更不用说 l'affaire Damore)。

    PS:我知道您知道所有这些,但我认为值得在谈话中强调这一点。

  223. res 说:
    @utu

    感谢您足够开放的态度来审视并重新考虑。你可以尝试连续阅读你的评论223和224,并思考像223这样的评论对深思熟虑的讨论有什么样的影响。我试图强调最后一篇论文(Johnson、Carothers、Deary 2009)是最好的(我按照找到它们的顺序给出了链接),但我想我的努力还不够。在找到那篇论文后,我停止寻找进一步的证据,这并非巧合。

    您对预测函数提出了很好的观点。 Hsu(等人)的预测器假设加性遗传学,所以它就是这样。以下是一些在加性遗传学背景下讨论 X 染色体的论文:

    估计 X 染色体的加性遗传效应: https://www.ncbi.nlm.nih.gov/pmc/articles/PMC3617965/

    表 1 有更多男性/女性均值和方差估计。

    常染色体和 X 连锁加性遗传变异对寿命和衰老的影响:果蝇性别内部和性别之间的比较: http://www.g3journal.org/content/6/12/3903

    这是摘要:

    理论对由于男性 X 半合性导致的 X 染色体和常染色体之间遗传变异的差异做出了一些预测。 X染色体应该:(i)通常表现出比常染色体相对较少的遗传变异,(ii)由于剂量补偿,与女性相比,男性表现出更多的变异,以及(iii)可能富含性别特异性遗传变异。在这里,我们对果蝇的寿命和衰老进行了每一项预测。为了实现 X 和常染色体加性遗传方差的无偏估计,我们使用 80 个染色体替换系; 40 个用于 X 染色体,40 个结合了两个主要常染色体,我们对其进行性别特异性和跨性别遗传(共)变异分析。我们发现两种性别的两种性状均存在显着的 X 和常染色体加性遗传变异(保留女性中 X 连锁的衰老变异),但没有确凿的证据表明 X 连锁变异的消耗(通过女性测量)。男性比女性表现出更多的 X 连锁寿命变异,但尚不清楚这是否是由于剂量补偿所致,因为男性的常染色体变异也更大。最后,我们的结果表明,X 染色体在生命周期中因性别特异性遗传变异而丰富,但对于整体衰老的结果却不太确定。总的来说,这些结果表明,X 染色体因长期遗传变异而降低了对寿命上的性一致选择的反应能力,而其对性对抗选择的反应能力可能会增强。

    有趣的是,他们还发现了更多男性常染色体变异。

    -

    我不记得 Hsu 论文(或我见过的任何预测论文)中对 X 染色体有任何特别提及。我刚刚错过了吗?或者是否存在某种假设的标准做法?

    我刚刚在最近的身高CS论文中搜索“x chr”并没有看到任何内容。

    PS 值得注意的是,这种效应预计会减少 Hsu 预测器解释的方差百分比。

    • 回复: @res
    , @utu
  224. dearieme 说:
    @James Thompson

    那里有一个研究课题,博士。几十年来我的身高都是 6 英尺 2.5 英寸。一年前,医院测量的身高为 6 英尺 3.5 英寸。我原以为我现在已经开始萎缩了。这可能是什么类型的测量误差?如果我继续这样加速,很快就会达到两米大关。

    PS 这也与我的体重无关;当我与恶魔碳水化合物搏斗时,它正在缓慢下降。

    • 回复: @James Thompson
    , @CanSpeccy
  225. res 说:
    @res

    这篇 2017 年关于男性型秃发的论文分别研究了 X 染色体和常染色体: http://journals.plos.org/plosgenetics/article?id=10.1371/journal.pgen.1006594
    它使用英国生物银行数据。论文作者在以下位置提供了 293MB(压缩!)的(摘要?!)数据: http://www.ccace.ed.ac.uk/node/335
    该网站还提供了 1.1 年关于认知功能和教育程度的 GWAS 论文的数据(2016GB,已压缩!!): http://www.nature.com/mp/journal/v21/n6/abs/mp201645a.html

    摘要:

    男性型秃发可能会产生重大的社会心理影响,并且在表型上与前列腺癌和心血管疾病等不良健康结果有关。我们利用英国生物银行 52,000 多名年龄在 40-69 岁的男性参与者的数据,探索了该性状的遗传结构。我们鉴定了超过 250 个与严重脱发相关的独立遗传位点 (P<5×10-8)。通过将队列分为 40,000 个发现样本和 12,000 个目标样本,我们开发了一种完全基于常见遗传变异的预测算法(AUC = 0.78,灵敏度 = 0.74,特异性 = 0.69,PPV = 59%,NPV = 82) %) 没有脱发的人与严重脱发的人。这项研究的结果可能有助于确定脱发风险最大的人群,以及潜在的干预遗传目标。

    预测器的有效性不是很好,但我喜欢他们使用单独的测试集并呈现多个性能指标的方式。

    我发现一些有趣的摘录:

    在双胞胎研究中估计,早发性脱发和晚发性脱发的男性型秃发中可归因于遗传因素的总变异比例约为 80% [11, 12]。较新的分子遗传学方法估计,基于单核苷酸多态性 (SNP) 的秃头常见变异遗传率约为 50% [13]。分子方法还表明,与秃头相关的遗传变异和与身高、腰臀比、男性声音下降年龄、女性初潮年龄和一字眉等表型相关的遗传变异之间存在一定程度的重叠[14]。

    对来自英国生物银行的 52,874 名英国白人男性进行的四类自我报告秃头测量的全基因组关联研究,从估算数据中得出了 13,029 个常染色体命中 (P<5×10-8),此外还有 117 个命中(来自X 染色体上有 14,350 个基因型 SNP(图 1)。

    使用次要等位基因频率至少为 1% 的常见遗传变异,GCTA-GREML 分析发现,47.3% (SE 1.3%) 的秃发变异可以通过常见常染色体遗传变异来解释,而 4.6% (SE 0.3%)可以用常见的 X 染色体变异来解释。

    请注意 X 染色体的巨大贡献。

    使用 LD 评分回归分析男性型秃发与 24 种认知、健康和人体测量特征之间的遗传相关性。没有发现显着的关联;所有估计值均接近于零(S6 表)。

    表中存在许多有趣的名义上显着的结果(参见上面的摘录),但没有一个能够通过 FDR 修正。

    • 回复: @RaceRealist88
  226. @dearieme

    在怀特岛的研究中,大部分差异可能是由于孩子们不总是站直造成的,但在许多情况下,健康访客错误地读取了体重秤。有错误,也有错误。

  227. @res

    不是众所周知,秃头是在你妈妈那边吗?对我来说是个好兆头。祖父56岁时去世,头发浓密。

    今晚晚些时候我将在另一个线程中回到我们的其他对话。我最近很忙。

    • 回复: @res
  228. utu 说:
    @res

    我不记得 Hsu 论文中特别提到过 X 染色体

    我也不。但说实话,我不知道如何将许的多基因评分与不同显性的孟德尔遗传学规则相协调。这不仅仅适用于第 23 号染色体。如果 Hsu A 矩阵 (y=Ax) 的一行中有一系列 SNP 值,则一对染色体中的 SNP 必须在多基因函数中视为成对,而不是独立的 SNP,并且多基因得分不会与关于一对中的两个 SNP。我只是还没有关于这个问题的良好思维导图。我需要读一些书,但我不知道从哪里开始。

    • 回复: @res
  229. res 说:
    @RaceRealist88

    不是众所周知,秃头是在你妈妈那边吗?对我来说是个好兆头。祖父56岁时去世,头发浓密。

    这是传统智慧(或者老太太的故事?)。事实是,这正是基于 X 染色体的遗传所看到的模式(例如,参见色盲)。因此,该论文支持这一说法有一定道理。例如:“正如预期的那样,最重要的基因命中是 X 染色体上的雄激素受体 (P = 2.0×10-269)。” (喜欢那个 p 值;)

    但实际上事情要复杂得多。首先,遗传给儿子的母亲 X 染色体将是来自外祖父母的 X 染色体的混合体。其次,总的来说,常染色体更为重要。

    PS 这篇关于 X 染色体遗传模式的文章可能有助于思考这个问题: http://www.genie1.com.au/blog/63-x-dna

  230. res 说:
    @utu

    但说实话,我不知道如何将许的多基因评分与不同显性的孟德尔遗传学规则相协调。

    线性度通常与现实情况非常匹配。即使不是这样,记住当 MAF 为 0.3 时,只有 9% 的人是隐性纯合的(对于 0.1 的 MAF,只有 1% 是纯合隐性的),这是有帮助的。这意味着,就方差解释而言,那些纯合隐性基因并不那么重要(无论如何,假设没有重大后果)。

    这是不将群体预测变量视为良好的个体预测变量的关键原因。或者更糟糕的是,作为基因工程理想目标的预测。想象一下像镰状细胞性状这样的东西,加性模型可能会得出镰状细胞纯合子是一件好事的结论。哎呀!

    我想知道是否可以通过标记个体具有显着 SNP 的罕见病例来改进个体预测?鉴于每个 SNP 的影响很小(我们认为,尽管考虑到一些阿什肯纳兹等位基因赋予杂合子优势,但纯合子可能患病)可能不会对预测产生太大影响。

    • 回复: @utu
  231. reiner Tor 说:
    @utu

    这是一个公平的观点,但一个年龄段内的相对身高可能也相当稳定。如果你和我同龄,而且我们 20 岁时你比我高,那么很可能你仍然比我高。即使差异很小,比如只有一英寸。就我而言,有一件奇怪的事情,我不知何故长了一厘米,但据我所知,这种情况很少见,所以即使是一厘米的差异也可能在我们的一生中保持稳定。

    我也确信,对于打篮球来说,一厘米并没有多大区别。然而,如果我们随机组成数百、数千甚至更多的由身高 173 厘米的人组成的球队,以及大量由身高 174 厘米的人组成的球队,并让他们互相比赛数百场比赛,我确信在样本足够大的情况下,我们可以发现,174 厘米的球队比 173 厘米的球队略胜一筹。可能效果会相当小(这只是篮球能力的一个很小的组成部分),并且需要大量的比赛,以便随机变化(由这里或那里非常有天赋的 173 球员等造成)可以相互抵消,但我确信它会存在。

    [更多]

    关于测量误差,这是我个人的例子。

    在18-24岁之间,我被多次测量身高为173厘米,这是我认为的最终身高,而在32或33岁时,我被测量为174厘米。(测量是在下午进行的,连续三四天睡眠不足,而24岁时的测量,是军队做的,是在一大早进行的,据我记得是在睡了好久之后才进行的。 )当我抗议我只有173时,那家伙又看了一遍,他说我有点低于174,可能是1736或1737毫米,但绝对接近174。我不确定,也许我超过173 18岁时已经是厘米了。但后来我38岁时再次测量,他们告诉我我是175。我大声抗议(并告诉他们我通常是173,但一旦测量我是174,绝对远不及175),所以他们又非常仔细地测量了一次,他们说他们错了,我有点接近174,但我比175更接近173。所以现在我肯定长高了至少一厘米。

    现在我写下这篇文章,我记得有一次在174岁时去看医生,测量了19。我觉得很奇怪,但很高兴长了一厘米,并没有推动这个问题。然而不久之后另一位医生给我测量的结果只有173,尽管我告诉他们我之前测量过174,但医生说这肯定是一个粗心的情况。 174岁入伍的时候,我还告诉他们,有一次有一个奇怪的测量结果,是174,所以他们又看了一遍,告诉我,我肯定只有173。所以我以为这只是一个错误的测量结果,几乎完全忘记了它。也许19岁的时候并没有错,只是我平时的姿势太糟糕,让我看起来矮了一厘米?在 20 多岁时开始运动后,我的姿势改善到足以回到 19 岁时的水平?不可能说清楚。无论如何,不​​仅我20岁时的相对身高有点模糊,而且我的绝对身高也有点模糊。

  232. utu 说:
    @res

    线性度通常与现实情况非常匹配。 – 但总是有限制的。看来,为了解释 IQ 分布的尾部,需要使用非线性模型。

    没有将总体预测变量视为良好的个体预测变量 – 人们可能会想到唯象热力学和统计热力学作为一个例子。最终,这取决于单个分子的行为。 Hsu 或 GWAS 所关注的是个体预测因子。我真的很想看到如何在数学上将孟德尔优势纳入许的方案的解释。

    • 回复: @res
  233. Passer by 说:
    @Anonymous

    在衡量语言能力的各种子测试中,语言差异很小——任何性别的智商分都不超过 2-3 分。 平均而言,成年男性和女性的语言能力相当。 (根据林恩提到的海德的荟萃分析,再加上我的经验,我看到了数百项智商研究,男性在言语成分上是相同或稍好一些)。

    现在,对于 WAIS——我不知道,可能有一些语言分测试可能对男性有利,而不是对女性有利。 但这种差异与由于缺乏心理旋转子测试而造成的差异相形见绌,其中存在大量的 1 SD dif。 有利于男性。

    换句话说,假设您添加了一项对智商为 3 分的男性有利的言语测试,并删除了一项心理旋转子测试,其中智商的差异为 15 分,对男性有利。 显然,即使言语分测试偏向男性,这也会导致对女性的重大偏见,仅仅是因为心理旋转的差异非常大,所以无论你如何进行言语分测试,你都无法轻易弥补这一点,因为差异很小。

    我想说的是,无论你在言语分测试中玩什么游戏,你都不会为任何性别获得很大的优势,但如果你消除心理旋转,将为女性带来很大的优势。

    WAIS 的言语数据与我在许多其他智商研究中看到的数据大致一致,即男性在言语上略有优势。

    有很多智商测试。 我不知道他们如何形成他们的言语成分/子测试。 但我可以告诉你,绝大多数智商测试(WAIS、WJ 3、AFQT/ASVAB、Wonderlick 等)都没有心理旋转子测试(你会发现认知能力上最大的性别差异,使任何其他测试相形见绌)差异),这不可能是巧合。 事实上,大多数测试都没有心理旋转子测试,这让我明白了这一点——这是故意的,而且偏见主要有利于女性。

    顺便说一句,我最近也发现了这个——

    “根据梅隆(1944-1945)的说法,男孩在数块任务上比女孩表现得更出色,
    这项测试被从苏格兰心理调查(1933)中删除,作为对一般情况的衡量
    智力。”

    https://www.cambridge.org/core/journals/amg-acta-geneticae-medicae-et-gemellologiae-twin-research/article/human-intelligence-sex-differences/2F028E657F4920C0DCAFC233C81D0C52

    许多现代研究都使用了苏格兰心理调查数据,这些研究声称智力没有性别差异。

    顺便说一句,我正在与林恩谈论其中一些问题,我给了他一些他尚未发布的数据。 所以我可以告诉你,至少还有 60 项其他研究证实了他的理论。 如果你愿意的话我可以给你一些数据。

    • 回复: @res
  234. Anoymous 说:

    这是给大家的一个问题。 这一直困扰着我好几个星期,我在这里问它是因为我不知道现在有任何其他网站正在讨论男性和女性大脑之间的差异(在某种程度上):

    女性大脑可以做男性大脑可以做的一切,那么上帝为什么要创造男性大脑呢? 为什么不是男性身体中的另一个女性大脑?

    • 回复: @John Jeremiah Smith
  235. Passer by 说:
    @Anonymous

    哦,我还发现了一件事

    “当标准化智商测试在 20 世纪初首次开发时,女孩的得分通常高于男孩,直到 14 岁,此时女孩的曲线下降到低于男孩的水平。 随着测试方法的修订,我们努力实现性别表现的平等。”

    https://infogalactic.com/info/Sex_differences_in_intelligence

  236. res 说:
    @utu

    我真的很想看到如何在数学上将孟德尔优势纳入许的方案的解释。

    你读过非线性压缩感知论文吗? https://gigascience.biomedcentral.com/articles/10.1186/s13742-015-0081-6

    如果给定的 SNP 只有两个相关等位基因,那么您需要建模的只是对应于 AA Aa aa 的三个值。二次方程将完全符合这个要求。当然,渗透率可能存在问题。只要环境占方差的相当大比例,任何预测器都不会是完美的。我不知道如果aa很少的话非线性CS会表现如何。

    具有两个以上等位基因的 SNP 非常罕见,我不确定我们是否需要担心它们。这是一个例子: https://www.snpedia.com/index.php/Rs6467

    如果有人想跟进此问题,请更多讨论多等位基因 SNP: https://www.biostars.org/p/511/

    • 回复: @utu
  237. @Anoymous

    女性大脑可以做男性大脑可以做的一切,那么上帝为什么要创造男性大脑呢? 为什么不是男性身体中的另一个女性大脑?

    嗯,这是不可能的,你明白吗?男性大脑是用幼发拉底河的腐烂泥创造的——上帝是使用最接近的可用材料的人,我们假设上帝当时站在齐脚踝深的河里。问题是,雌性是由肋骨构成的,因此雌性大脑都是肋骨 DNA 的子集。这就是为什么小鸡既笨又怕痒。

    • 回复: @Anonymous
  238. @AP

    平均 4 分的差异导致较高端的显着差异

    只是“高端的差异”似乎并不意味着什么该死的事情。或者,如果确实如此,那么有人需要解释为什么特曼在选择高智商个体进行长期研究高智商成就的人时,设法将仅有的两名未来诺贝尔奖获得者(物理学奖也不例外)排除在候选人之外。他们还需要解释为什么没有其他研究证明智商在识别那些会做比成为高薪会计师、律师或直肠科医生更聪明的事情方面的价值。

    从本质上讲,智商主义就是忽视一个人实际上可以做什么,而是根据一些米老鼠测试的结果建立一个晋升和奖励系统,该测试据称表明一个人在受到干扰和影响的情况下应该能够做什么。碰巧有想象力去做这件事。

    最终的结果是一个教育体系不能有效地教授任何东西,因为你知道什么并不重要,重要的是如果你知道如何做并且愿意去做的话你能做什么。因此,通过 SAT 考试进入顶尖大学。难怪美国文明正以越来越快的速度被冲入下水道。

    • 回复: @James Thompson
    , @res
    , @Santoculto
  239. @dearieme

    几十年来我的身高都是 6 英尺 2.5 英寸。一年前,医院测量的身高为 6 英尺 3.5 英寸。我原以为我现在已经开始萎缩了。这可能是什么类型的测量误差?

    身高和精神敏锐度一样,并不是一个完全固定的量。美国将军史迪威,醋乔,考入大学失败,因此寻求进入西点军校,但他太矮了。因此,在以候选人身份出现之前,他在床上躺了几周,这充分减轻了他的脊柱压力,以达到最低身高要求。

  240. @CanSpeccy

    Re Terman,一个好的预测器不会是一个完美的预测器。特曼在测试中专注于言语,因此错过了一些非常聪明的学生。测试语言、数学和空间以获得最佳结果。

    • 回复: @CanSpeccy
  241. res、utu 和 Thompson 博士,

    肯·理查森 (Ken Richardson) 几个月前发表的一篇论文表明,对于 GWAS 而言,相关性是不可避免的且毫无意义。尽管该论文不是开放获取的,但我找不到它。我想知道汤普森博士或其他人是否可以找到这篇论文并留下一些想法?

    http://onlinelibrary.wiley.com/doi/10.15252/embr.201744140/full

    全基因组关联研究描述了与教育和职业成功相关的遗传变异,被誉为未来个性化学习的基础。这些预兆具有误导性,因为这些研究没有考虑更复杂的社会因素。

    • 回复: @res
    , @James Thompson
  242. res 说:
    @Passer by

    感谢您的有趣评论。 Jarvik (1975) 中的 SMS 观察是一个很好的发现。 也来自该论文:

    还有 WAIS 年龄 16-64 岁的标准化数据。 在这里,我们再次获得了实际分数,并且正在处理可能具有代表性的美国白人群体的一项研究,而不是之前介绍的企业集团(图 1-3)。 那些像我一样熟悉旧的 Wechsler-Bellevue 且没有特别关注 WAIS 变化的人可能会惊讶地发现,男性和女性仅在 2 个子测试中获得了相似的平均分数(图 4) 。 这两个子测试是数字跨度和图片排列。 在其余 2 个子测试中,有 6 个子测试中,男性得分高于女性,最显着的差异是在算术子测试中,女性获得的加权分数约为 9,男性约为 9.2。 女性获得较高平均分的 10.3 项测试是相似性、词汇和数字符号替换,其中女性在数字符号测试中差异最为显着。 (男性获得的值低于图 3 中使用的量表。)除了物体组装之外,性别差异均具有统计显着性。

    也许比所有年龄段的差异加起来所达到的统计显着性更令人印象深刻的是,韦克斯勒将样本分为 7 个年龄组,性别差异表现出惊人的一致性。 在信息和算术方面,每个年龄组中男性的平均得分均高于女性,而在理解图片完成和块设计方面,6 个年龄组中有 7 个出现了男性的差异。 所有 7 个年龄组的词汇和数字符号替换以及 5 个年龄组中的 7 个年龄组的相似性均取得了较高的平均分。

    在最广泛使用的一般智力测试的子测试中,这些明显的性别差异可能会让一些读者感到惊讶。 他们对我做了。 然而,对于仔细阅读过 WAIS 手册的人来说,它们是众所周知的。 韦克斯勒用了整整一章来讨论智力的性别差异。 他本人对这些发现印象深刻,并根据区分子测试构建了男性气质-女性气质(MF)指数。 男性测试包括信息、算术和图片完成; 女性测试包括词汇、相似性和数字符号替换。 通过从男性测试中的加权分数之和减去女性测试中的加权分数之和,获得MF分数。 正的 MF 分数表示男性趋势,负分数表示女性趋势。 我将这个量表的含义留给读者来解释。

    韦克斯勒子测试“MF 指数” 似乎是一个有用的搜索。例如,从 http://www.tandfonline.com/doi/abs/10.1080/00221309.1963.9920552?journalCode=vgen20
    我们看:

    然后,他根据以下公式制定了男性和女性百分位数分数表:男性测试-女性测试。 他指出,正分表示男性气质,负分表示女性气质,此外,“男性的得分为负 3.5 分或更低,女性的得分为正 2 分或更高”(149,第 XNUMX 页)。

    有一些文献研究了 MF 指数以及其他按性别和性取向进行的认知和人格测量。

    本文(DOI 10.1002/1097-4679(197807)34:33.0.co;2-2)研究了许多测量方法及其相关性,发现性别和性取向之间存在显着关系: https://www.ncbi.nlm.nih.gov/pubmed/690201

    在看下面的论文(强调我的)之前,值得注意的是:

    值得注意的是,与单一性别组相比,混合性别组中的每个 MF 指数与性取向的相关性更高。 也就是说,当相关性基于有限的分数范围时, 与单一性别组一样,任何 MF 量表与双相标准测量之间的相关性都被严重低估。 在整个分数范围内,每个 MF 量表与性取向之间的显着相关性为我们的论点提供了额外的支持,即 MF 可以被视为双相维度。 还应该指出的是,混合性别组中相关性的大小表明,当自评性取向是标准指标时,三个双相 MF 指数中的每一个都具有相当大的外部效度。

    这篇论文给出了一个标题(摘要的最后一行)结论“我们得出结论,性取向与特定的认知能力无关。”: https://www.researchgate.net/publication/21097178_Sexual_orientation_and_cognitive_abilities

    但往内看,我们会发现类似的东西(HS = 同性恋,HT = 异性恋):

    鉴于此处回顾的文献,似乎特定的认知能力和性取向之间可能存在关系。 HS 男性在语言和空间能力方面与女性相似,但 HS 女性在空间能力方面与男性不同。

    在上面的前一篇论文摘录的背景下考虑以下内容:

    本研究比较了精心挑选的 HS 和 HT 受试者组在众所周知的 MF 测量和六项认知能力测试中的表现。 在这里,我们正在通过性取向来检验性别内的性别非典型性假设。

    不幸的是,我们的参考比较对于判断认知数据的可靠性几乎没有什么帮助。 我们复制了空间能力方面的性别差异,但没有复制语言或数学能力方面的性别差异。 我们的样本和 WAIS 的上述局限性也可能解释了这一发现。 然而,鉴于我们的数据以及其他人未能发现 MF 与智力功能之间的任何关联,我们认为性取向不太可能预测性别非典型认知能力。 在男性中,三种认知测量的差异表明性取向可能是一个预测因素,但只是以某种未指定的方式。 也许HS男性在语言能力上具有性别非典型性,但在其他认知能力上却没有性别差异。

    我对这篇论文的一个担忧是他们消除了“相对双性恋”的受试者。我不确定这可能会带来什么样的偏见。例如,对于男性:“HS 科目的综合金赛评分为 5-6; HT 受试者的综合 Kinsey 评分为 0-1。”因此,范围的整个中间部分已被排除。

    PS Thompson 博士讨论了 Hyde 的空间结果 https://www.unz.com/jthompson/google-culture-wars/#p_1_23:1-11
    我在这篇评论中进行了一些推测: https://www.unz.com/jthompson/google-culture-wars/#comment-1971075

    您对空间测试的非计时/计时区别有什么想法吗? 我的猜测是,定时更好地反映了真实的空间技能,而不定时则添加了重力加载组件。

  243. @James Thompson

    特曼在测试中专注于言语

    那么他有测量智商吗?

    特曼使用斯坦福-比奈测试, 据说可以测量 “语言和非语言分测试[包括]定量推理、视觉空间处理、工作记忆和流体推理。”对我来说听起来像是智商测试。那么我们是否可以假设存在多个智商,并且一个人可以同时具有高智商和低智商呢?

    特曼实际上证明的是,智商并不能衡量非凡的能力。事实上,高智商可能会排除卓越的能力。

    事实上,智商大概衡量的是综合能力。因此,高智商的人会取得职业上的成功:例如,良好的临床态度、良好的诊断技能、良好的公关、良好的办公室管理技能、良好的簿记技能,因此成为年薪百万美元的医生。

    但天才并非全才。 IQ-125 理查德·费曼 (Richard Feynman) 拼写很差,写得也很差,但对某些问题的思考却让他表现出了天才。爱因斯坦也是如此,作为知识分子,像费曼一样,表现出明显的平庸。有人认为,在量子力学领域,莎士比亚会被证明像一袋锤子一样愚蠢,查尔斯·达尔文只不过是一只痴迷的收藏家甲虫,但凭借他对生物多样性知识的广度和深度,他做了一些事情。

    • 回复: @CanSpeccy
  244. res 说:
    @CanSpeccy

    他们还需要解释为什么没有其他研究证明智商在识别那些会做比成为高薪会计师、律师或直肠科医生更聪明的事情方面的价值。

    我认为 SMPY 和杜克大学 TIP 研究驳斥了这一说法。这是一篇关于 SMPY 的 Nature 文章: http://www.nature.com/news/how-to-raise-a-genius-lessons-from-a-45-year-study-of-super-smart-children-1.20537

    下图显示了高智商个体的明显结果差异:

    这是对此类研究的很好的概述: 从特曼到今天:一个世纪关于智力早熟的发现
    http://journals.sagepub.com/doi/abs/10.3102/0034654316675476
    http://emilkirkegaard.dk/en/wp-content/uploads/From-Terman-to-Today-A-Century-of-Findings-on-Intellectual-Precocity.pdf

    摘要:

    回顾了一百年来(1916-2016)关于智力早熟青年的研究,描绘了非凡的人力资本来源以及促进卓越成就、生活满意度和积极成长所需的各种学习机会。重点是针对一般或特定(数学、空间或语言推理)能力排名前 1% 的个体进行的研究。对天赋现象的早期见解实际上预示了 100 年后科学证明的结果。因此,基于证据的概念迅速从将智力早熟的个体视为软弱且情绪不稳定的个体转变为高效且有弹性的个体。与所有群体一样,智力早熟的学生和成人也各有优势和相对劣势。他们还表现出了对不同追求的热情和实现目标的动力的巨大差异。因为他们不具备多重潜能,所以我们必须以多维的视角来看待他们的个性。完成后,它可以很好地预测长期的教育、职业和创造性成果。

    • 回复: @CanSpeccy
    , @CanSpeccy
  245. res 说:
    @RaceRealist88

    它可以在 libgen 上找到。搜索 DOI 10.15252/embr.201744140

    这些愿望因只发现很小且通常不显着的关联以及未能复制结果而受挫。

    存在显着的关联,并且有一些(大多数?不是全部)重复。

    以下是他的完整结论:

    粗略地阅读有关 GWAS 和 CA/EA 的论文似乎很有说服力:“硬”遗传数据、简洁的分数和简单的相关性。然而,他们忽略了人类社会、历史和心理的复杂性以及认知和教育的本质。 SNP与CA或EA之间的关联是发达社会的阶级结构、移民分布不均以及措施本身所造成的“系统性分层”的必然结果。这种刻意的永久分层和变量的非独立性也使纠正人口结构的尝试陷入困境。
    在 GWAS/PGS 中。

    这至少对 GWAS 认知功能的部分动机提出了质疑:
    培育有针对性的遗传方法来改善人类认知发展和教育。当然,这是一个令人畏惧的前景。一些研究人员对这种愿望持谨慎态度是正确的,哪怕只是考虑到技术可行性。然而,他们的担忧应该消除,因为他们认识到这一愿望不仅在技术上令人畏惧,而且在概念上也不现实。正如本文所述,人类变异、群体差异和测量的动态存在于更加复杂的多层次系统中。

    最后,值得强调的是,这一结论还有许多其他假设
    研究计划,尤其是关于基因本身的性质和人类发展的研究计划。
    其他人警告说,GWAS 背后的简单、百年历史的遗传变异模型
    现在已知这些与遗传现实相去甚远,并导致了 GIGO(垃圾进垃圾出)遗传学的热潮。也许是时候更加密切地关注这些批评了。

    这样的“推理”你真的觉得有说服力吗?

    这句话到底是什么意思呢?你能给我解释一下吗? “其他人警告说,目前已知 GWAS 所依据的简单的、已有百年历史的遗传变异模型与遗传现实相去甚远,并导致了 GIGO(垃圾进垃圾出)遗传学的热潮。”

    他是想说加性遗传学并不是一个完整的解释吗?现在有一个启示。我认为解释 40% 的身高差异是“垃圾”。

    还有一些问题,您认为最近的压缩感知高度结果是否有效(例如不受他的论证)?如果是这样,对 IQ 和/或 EA 做类似的事情是否有效?

    • 回复: @RaceRealist88
  246. Santoculto 说:
    @CanSpeccy

    智商往往会捕捉


    一般认知能力

    心理测量学家承诺进行定性方面的分析和/或研究,但是……至少现在……

    他们倾向于将心理方面/个性与认知分开,他们单方面称之为“智力”。

    对于许多人(如果不是大多数人)来说,智力只是在工作和学校中有用。为了与他人打交道并理解我们,我们需要利用我们的个性。也许是一个温和的稻草人……

    心理测量学家的另一个常见错误是过度夸大数学技能,好像其余的都不重要。

    • 回复: @CanSpeccy
  247. @res

    LOL

    感谢您发布图表,该图表充分反驳了您自己的论点。

    首先,SAT数学测试不是智商测试。

    其次,看看预测能力极低。 SAT 数学考试中排名倒数四分之一的人获得博士学位并在 STEM 期刊等上发表论文的人数几乎是排名前四分之一的人的一半。换句话说,在恶毒、无聊和日益奴性的学术界,成为一个数学傻瓜几乎不会成为进步的障碍。

    那么艺术领域的数学假人又如何呢?哈!我们不知道。哦,也许他们没有像数学极客在物理学中获得那么多历史博士学位,但那又怎样呢?对历史学博士学位的需求并不大,那些可能对这个学科感兴趣的人很少会浪费几年的时间去追求一个资格,而这个资格充其量只能让他们获得一个从教的兼职职位。在那些现在管理着美国曾经伟大但现在很糟糕的大学的傻瓜官僚的监督下,制定了一份关于最低工资的谎言文本(他们绝不能背离它)。

    • 回复: @res
  248. Anonymous • 免责声明 说:
    @John Jeremiah Smith

    很好,但我正在寻找更严肃的答案。如果不是上帝……那么自然或宇宙或多元宇宙或随机机会。基本上,无论是谁或什么东西产生了男性大脑和女性大脑。为什么这个实体会产生男性大脑?

    • 回复: @John Jeremiah Smith
  249. @res

    事实上,你的图表主要显示的是(a)一个人的计算能力越强,他们就越有可能选择研究 STEM 领域而不是艺术,以及(b)更多的人在 STEM 领域获得博士学位、发表论文与那些倾向于寻求艺术学术资格的文化程度较高的人相比,他们在 STEM 期刊上发表论文、获得专利等。当然,它并没有告诉你任何关于智力和成就的信息。

  250. utu 说:
    @res

    如果给定的 SNP 只有两个相关等位基因,那么您需要建模的只是对应于 AA Aa aa 的三个值。二次方程将完全符合这个要求。

    这是一个可以对此进行建模的函数 f(x,y)。两个 SNP x 和 y 的值为 A 或 a。根据某种约定,A 和 a 具有一些数值,例如 0 和 1。

    f(x,y)=D0+D1(xa)(ya)+D2[(xA)(ya)+(xa)(yA)] ,其中 D0,D1,D2 是要从拟合中找到的系数

    f(a,a)=D0
    f(a,A)=D0+D2(aA)(Aa)
    f(A,a)=D0-D2(Aa)(aA)=f(a,A)
    f(A,A)=D0+D1(Aa)^2

    这是二维多项式 f(x,y)+d2+d0*x+d1*y+d2*X*y 有四个系数。偏移 d0 是所有 SNP 拟合中一个全局偏移的一部分。 d1 和 d2 已经是线性拟合的一部分,但是它们可能需要从线性拟合中解耦以模拟 f(x,y) 函数的工作。这可能是一个问题。但如果我们暂时忽略它,剩下的就是添加混合项 xy。我认为这将包含在 Hsu 在他的非线性论文中提出的内容中,尽管二次项不是必需的,只需混合项就足够了。

    BD 模型具有 s 因果基因座,每个基因座对表型具有(随机确定的)线性和二次效应,以及 混合项将一个基因座耦合到另一个基因座。用生物学术语来说,该模型描述了一个系统,其中每个基因座与同一块中的其他基因座(包括其自身)相互作用,但不与块外的基因座相互作用。

    不必尝试所有可能的混合术语,只需混合属于染色体中同一对的部分的 SNP,即可减少术语数量。

    第二个想法:我不确定这个模型是否有两个以上的等位基因就足够了。需要考虑是否x^2*y 和 x*y^2 需要添加到模型 AA、Aa、aa、bb、ba、bA 中。

    • 回复: @utu
    , @res
  251. 为什么这个实体会产生男性大脑?

    查尔斯·达尔文(Charles Darwin)很久以前就给出了答案,至少是大体上的。男性和女性之间存在角色分工,这导致男性和女性在大脑功能等方面面临不同的选择压力。结果?男人女人都不是, 尽管运营谷歌的(高智商)白痴们无知地说了什么,智力上相同。

    • 回复: @CanSpeccy
  252. @Santoculto

    心理测量学家的另一个常见错误是过度夸大数学技能,好像其余的都不重要。

    这是灯柱谬误。给人们做一个数字测验比确定他们是否能够从澳大利亚内陆探险中找到回家的路或者是否会渴死更容易。

    真正的问题是,似乎很少有心理学家了解生物学,因此也了解什么是智力,即促进生存和繁殖成功的行为能力,至少在人类心智很大程度上形成的情况下是这样。

  253. @CanSpeccy

    如果男性和女性在智力上并不相同,那么几乎可以肯定女性在某些事情上比男性更好,就像男性在杀人方面可能比女性好得多(即使在扣除体重和力量上的性别差异之后) )。但这对心理计量学来说是一个复杂的问题。不会考虑,因为这会导致单独的男性和女性智商量表的复杂化。

    • 回复: @res
  254. utu 说:
    @utu

    男性和女性预测函数之间的差异在于,在 23 号染色体上,女性将具有混合项和 2 倍以上的 SNP:

    f_male=sum(Ai*SNPi)
    f_female=sum(Ai*SNPi)+sum(Bj*SNPj)+sum(Aij*SNPi*单核苷酸多态性j)

    这些函数将分别适合男性和女性的数据集。其中 SNPi 为男性和女性共有,SNPj 位于女性独有的 X 染色体上。男性和女性的 Aj 系数是否相同?万一他们会这样,就很难想象 std.dev 了。 f_female 的值将小于 f_male 的值。

  255. utu 说:
    @CanSpeccy

    这是灯柱谬误。 – 但你意识到这些混蛋正在构建现实,其中只有一盏灯,他们可以控制它的发光位置。一旦他们完成了这一切,他们的现实就会永远得到巩固。它将比奴隶制或任何其他曾经使用过的社会种姓污名更有效,因此也更持久。唯一的问题是权贵阶层的孩子,尽管有很多机会并被送去智商增强工厂,但他们的表现却不佳。仍然有乌代和库赛·特朗普或侯赛因。基因工程和抢夺不幸者的聪明孩子将是一个解决方案。

    我喜欢澳大利亚内陆地区一些人的形象。其实我有一个清单。我们是否应该开始终极智商之旅并以折扣价邀请阿尔斯特大学的教师?

    • 回复: @CanSpeccy
  256. res 说:
    @CanSpeccy

    其次,看看预测能力极低。 SAT 数学考试中排名倒数四分之一的人获得博士学位并在 STEM 期刊等上发表论文的人数几乎是排名前四分之一的人的一半。换句话说,在恶毒、无聊和日益奴性的学术界,成为一个数学傻瓜几乎不会成为进步的障碍。

    你知道那些“数学傻瓜”的数学成绩都排在前 1% 吗?我想知道这些成就的平均人口比例是多少。

    • 回复: @CanSpeccy
  257. res 说:
    @utu

    我再说一遍,你读过非线性压缩感知论文吗?如果您要回答这个问题,您应该这样做。

    你为什么让这件事变得如此困难?对于双等位基因情况 (AA Aa aa),只需根据单个变量创建一个方程 - 存在的 As(或 as)数量(0、1 或 2)。拟合一个线性方程就可以得到加性模型。拟合二次方程将给出精确拟合。

    • 回复: @utu
  258. res 说:
    @CanSpeccy

    如果男性和女性在智力上不相同,那么几乎可以肯定女性在某些方面比男性更好
    ...
    但这对心理计量学来说是一个复杂的问题。不会考虑,因为这会导致单独的男性和女性智商量表的复杂化。

    请参阅我的评论 251,其中讨论了女性得分更高的子测试。当你发表的评论没有被之前的评论反驳时,你看起来更聪明。

    • 回复: @CanSpeccy
  259. utu 说:
    @res

    是的,我读了报纸。你最好阅读我的两条评论,你就会明白它是如何运作的。您可以稍后感谢我。你的二次方程想法是无稽之谈。你有两个变量而不是一个!

    • 回复: @res
  260. @res

    Libgen 现在对我来说很愚蠢并且没有加载。愿意为我提供 pdf 吗?谢谢。在他发表这篇论文几天后,我发现了这篇论文,并在 libgen 上搜索了它,但还没有找到。

    这样的“推理”你真的觉得有说服力吗?

    关于基因本身和多级系统(智能系统)的本质?非常如此。智能系统理论非常有趣。理查森也有一本关于这方面的书。

    我们还需要讨论基因的性质。这对于讨论很重要。

    其他人则警告说,目前已知 GWAS 背后的简单的、已有百年历史的遗传变异模型与遗传现实相距甚远,并导致了 GIGO(垃圾进垃圾出)遗传学的热潮。

    我不知道我没有读过这篇论文(libgen 很愚蠢)。他在论文中是否提出了“GWAS 背后的简单、百年历史的遗传变异模型”?我确实相信,我们应该开始以不同的方式看待基因——不是将其视为“因果因素”——而是将其视为发育系统的奴隶。请参阅 Denis Noble 和 Eva Jablonka 的作品。

    这是伊娃·贾布隆卡 (Eva Jablonka) 和马里昂·兰姆 (Marion Lamb) 的书中的一段话 四个维度的进化:生命史中的遗传,表观遗传,行为和符号变异 (我忘记了页码,我在电子邮件中引用了这句话):

    分子研究所强化的事情之一是现代遗传学家已经接受的事情:基因作为简单因果因素的流行概念是无效的。存在冒险精神、心脏病、肥胖、宗教信仰、同性恋、害羞、愚蠢或任何其他身心方面的基因的想法在基因话语平台上没有立足之地。尽管许多精神病学家、生物化学家和其他非遗传学家的科学家(但在遗传问题上能够出色地表达自己的观点)仍然使用基因语言作为简单的因果因素,并向观众承诺快速解决各种问题,但他们并没有不仅仅是那些知识或动机值得怀疑的宣传者。遗传学家现在(大多数时候)根据由数十或数百个基因和基因产物组成的遗传网络来思考和谈论,这些基因和基因产物相互作用并共同影响特定性状的发展。他们认识到,在大多数情况下,某种特征(例如性偏好)的形成并不取决于单个基因的差异。它涉及许多基因、许多蛋白质和其他类型的分子以及个体发育的环境之间的相互作用。

    然而,如果一个基因仅在其所属的复杂系统的背景下才有意义,那么就一个或多个孤立基因的频率变化而言,思考进化的标准方式就需要受到质疑。例如,关注替代相互作用网络的频率变化而不是单个基因的频率可能更合适。

    我认为最后一段非常引人注目。您对整个系统中的基因而不是该系统之外的单个基因有何看法(回到我们之前关于整体论/还原论的讨论)?

    还有一些问题,您认为最近的压缩感知高度结果是否有效(例如不受他的论证)?如果是这样,对 IQ 和/或 EA 做类似的事情是否有效?

    你有参考吗?

    顺便说一句,这是我刚刚引用的书的完整第一章,如果你愿意阅读的话(我相信这句话是在第一章或第二章中):

    https://mitpress.mit.edu/sites/default/files/titles/content/9780262600699_sch_0001.pdf

  261. @Anonymous

    很好,但我正在寻找更严肃的答案。如果不是上帝……那么自然或宇宙或多元宇宙或随机机会。基本上,无论是谁或什么东西产生了男性大脑和女性大脑。

    我相信人类大脑会根据宿主的物理生物化学而变化,与性别相关的生物化学是该组的成员。

    有一群特别着迷的男性,他们专注于指出女性的各种缺点。该子集的一些成员很高兴地接受了智商的角度。哪些数字,你知道。有些人只是要做他们必须做的事。向他们解释任何不同的东西是没有意义的。

    这是一种对机会均等的痴迷。对于男性的智力和性格弱点,女性也会这样做。

    除了指出男人或女人的这种行为之外,几乎任何罪过都可以被原谅。

  262. @CanSpeccy

    真正的问题是,似乎很少有心理学家了解生物学,因此也了解什么是智力,即促进生存和繁殖成功的行为能力,至少在人类心智很大程度上形成的情况下是这样。

    普扎克利。这种研究是对杰曼这个白痴大力推行的“行为遗传学”的合法追求。

  263. res 说:

    Libgen 现在对我来说很愚蠢并且没有加载。愿意为我提供 pdf 吗?

    Libgen 早些时候也给我带来了麻烦。请继续努力。我没有好的方法来匿名提供文件,也不喜欢直接链接到 libgen。

    然而,如果一个基因仅在其所属的复杂系统的背景下才有意义,那么就一个或多个孤立基因的频率变化而言,思考进化的标准方式就需要受到质疑。例如,关注替代相互作用网络的频率变化而不是单个基因的频率可能更合适。

    我认为最后一段非常引人注目。您对整个系统中的基因而不是该系统之外的单个基因有何看法(回到我们之前关于整体论/还原论的讨论)?

    我认为该段落在这种情况下并不引人注目。它谈论的是进化,在这种情况下可能是有意义的,但我认为它在研究当前基因型和表型之间的关系方面没有太多可说的。

    我是工程师。工程师最重要的能力之一是进行合理的简化来管理复杂性。告诉我复杂性的存在(我已经知道)并不会让这些简化变得不那么有用。工程学的好处是,在一天结束时,您通常可以观察工作与不工作。恕我直言,创建有用的预测函数的能力是“有效”的明确指标。

    请记住,理查森的论文标题是:“GWAS 和认知能力:为什么相关性是不可避免且毫无意义的。”我想说,一个有用的预测函数驳斥了这个标题。也许你熟悉这句话:“科学的伟大悲剧——丑陋的事实扼杀了美丽的假设。”
    https://www.brainyquote.com/quotes/quotes/t/thomashuxl101763.html

    还有一些问题,您认为最近的压缩感知高度结果是否有效(例如不受他的论证)?如果是这样,对 IQ 和/或 EA 做类似的事情是否有效?

    你有参考吗?

    最近在这个博客中讨论了这个问题,但是既然你问得很好: https://www.unz.com/jthompson/heritability-lost-and-found/

  264. AP 说:
    @CanSpeccy

    真正的问题是,似乎很少有心理学家了解生物学,因此也了解什么是智力,即促进生存和繁殖成功的行为能力,

    所以在你们的世界里,智力真正是什么,是这个人所拥有的东西:

    http://www.dailymail.co.uk/news/article-2146545/Man-fathered-30-kids-11-different-women-says-needs-break--child-support.html

    显然,需要生物学知识才能得出这样的结论。

  265. res 说:
    @utu

    你有两个变量而不是一个!

    有一个限制。给定的 SNP 处恰好有两个等位基因。

    如果你有一个约束 x + y = 2 那么你可以通过替换 y = 2 – x 将事情减少到单个变量

    这是高中代数,utu。

    PS 来自非线性压缩感知论文的数据描述部分(重点是我的):

    我们的大多数模拟都是使用合成基因组进行的,次要等位基因频率 (MAF) 的值限制在 0.05 到 0.5 之间。合成基因组确定如下:为每个基因座生成随机群体水平的 MAF ε (0.05, 0.5),然后用 0,1,2 SNP 值 根据每个位点的 MAF。

    • 回复: @utu
  266. utu 说:
    @res

    如果你有一个约束 x + y = 2 那么你可以通过替换 y = 2 – x 将事情减少到单个变量

    这与问题的相关性如何?您有两个彼此独立的 SNP。一个中的等位基因并不决定另一个中的等位基因,所以你的 基本的,亲爱的华生 例如 x+y=2 没有意义。

    展示对于 AA、Aa、aa 的三个组合可以有三个不同值的函数。它还必须是对称的,即 Aa 和 aA 的值相同。

    您可以自己验证我在注释 #259 中明确构造的函数是可以做到这一点的最低次数的二维多项式。是的,这是初级的。是的,你稍后可以感谢我。

    我将回过头来思考三个等位基因的情况。

    • 回复: @res
  267. Santoculto 说:
    @CanSpeccy

    我认为促进繁殖成功和生存是智力的后果之一,但其原始概念是通过适应达到平衡的能力。当这种平衡被打破并且物种需要重塑自身以重新适应时,进化通常就会发生。

    文明意味着文化的积累,而原住民文化却是同样的改进,一般需要最简单的利用。

    我说过,“系统化”的人往往对非数学技能有偏见,因为他们在 STEM 上占大多数……相反,我们对智力有真正的理解,我们有一个扭曲的概念,偏向于“系统化”的东西。将认知与人格分开,并将第一个命名为智力​​,也是暗示智力士气低落的巧妙方式,即使这不是他们的初衷。

    适应最大化似乎是针对真实智力=在挑战性环境中适应的能力、“真实”或最大化智力的选择。

    文明还创造了一个虚假的现实,即我们并不生活在不同群体之间存在各种紧张关系的地方。

  268. res 说:
    @utu

    对我的0、1、2 PS没有评论吗?我猜你在阅读非线性计算机科学论文时错过了这一部分。这一点是本次讨论的重点。

    如果你真的谈论的是 2 个 SNP(而不是单个 SNP 的 2 个等位基因),那么你谈论的是基因与基因的相互作用,这确实是不同的。

    对于单个 SNP 上的双等位基因情况,我们有以下情况:

    展示对于 AA、Aa、aa 的三个组合可以有三个不同值的函数。它还必须是对称的,即 Aa 和 aA 的值相同。

    假设我们有以下情况:
    等位基因xy
    AA 0 p
    Aa/aA 1 q
    氨基酸2r
    x 是基因型 (0, 1, 2) 的数值表示,而 p、q、r 是表型的数值。

    然后我们将抛物线 y = ax^2 + bx + c 拟合到这三个点:
    p = c
    q = a + b + c
    r = 4a + 2b + c
    由于 p、q 和 r 已给出,我们有三个未知数 (a、b、c) 的三个方程。求解 a、b 和 c 可得出适当的方程。

    因为你似乎不相信我说的一个字,而且似乎也跳过了高中代数,所以这里有一个描述该过程的链接(参见“通用表格”): http://jwilson.coe.uga.edu/emt668/emat6680.f99/jones/instructional%20unit/writingquads.html

    • 回复: @utu
  269. @utu

    但你意识到这些混蛋正在构建一个只有一盏灯的现实,他们可以控制它的发光位置。一旦他们完成了这一切,他们的现实就会永远得到巩固。

    是的。以前也发生过这样的事。它被称为黑暗时代。但现在世界的联系更加紧密,因此,如果没有更有活力的代理人的干预,美帝国是否会陷入无知和压迫是值得怀疑的,中国,也许是复兴的俄罗斯帝国,甚至是摆脱了压迫性统治的欧洲。选民,我的意思是不可或缺的国家。

  270. @res

    你知道那些“数学傻瓜”的数学成绩都排在前 1% 吗?

    你确实明白,第一个百分位数的能力差异比同等规模人口中任何其他部分的能力差异要大得多。

    在这百分之一的学生中,大多数都是聪明的学生,他们坚持并很好地掌握了 12 年级的数学。然后会有一些人真正对数学、火箭、工程等感兴趣,他们掌握了大部分(如果不是全部)大学数学课程,甚至可能读过费尼曼的物理讲座并做过所有例子。

    • 回复: @res
  271. @res

    啊,是的,你在 251 的评论看起来相当长而且复杂。本着及时行乐的精神,我跳过了它,但它证实了我所说的,即男人和女人是不同的,因此,智商衡量“智力”的想法就消失了。所有那些摆弄数字,让男性和女性在不同测试中得分时可以比较他们的智力的说法似乎很荒谬。

    如果存在与女性智力不同的男性智力,那么为什么不能有跨性别智力或同性恋智力,更不用说诗人、政治家、钢琴家和职业扑克玩家的智力明显不同。

    • 回复: @Santoculto
  272. 所以在你们的世界里,智力真正是什么,是这个人所拥有的东西:

    http://www.dailymail.co.uk/news/article-2146545/Man-fathered-30-kids-11-different-women-says-needs-break–child-support.html

    哈哈。 《每日邮报》,智者的科学指南。

    但你为什么认为养育 30 个孩子和总共 11 个[不同!] 的女性并不表现出智慧呢?

    对于大多数男人来说,至少在最初阶段,养育孩子难道不是一种愉快的经历吗?对快乐的追求难道不是大多数男人将他们的智慧投向的目标吗?

    看起来很清楚,并且肯定符合进化论,这家伙所拥有的心态在智商主义消亡后很长一段时间内仍将存在。

    • 回复: @AP
  273. res 说:
    @CanSpeccy

    你确实明白,第一个百分位数的能力差异比同等规模人口中任何其他部分的能力差异要大得多。

    嗯,还有最底层的1%(即你的说法是错误的)。但是,是的,我很清楚尾巴的远端有多少变化。

    观察你对证据的反应是很有趣的。

    • 回复: @CanSpeccy
  274. @CanSpeccy

    特曼实际上证明的是,智商并不能衡量非凡的能力。事实上,高智商可能会排除卓越的能力。

    特曼成功地从筛选的年轻“天才”中排除了仅有的两名获得诺贝尔物理学奖的天才*,这有力地表明,高智商所代表的全面能力确实排除了天才。

    假设天才依赖于狭隘的专注心理发展,这可能需要将认知资源分配给特殊兴趣领域,这是完全有道理的。这种关于天才的观点,是由强迫性思维驱动的不平衡发展的结果,与人们对最高阶天才的认识是一致的,包括牛顿、达尔文、克拉克·麦克斯韦、爱因斯坦、费曼、丘吉尔(措辞)以及可能的大多数天才,如果不是全部,还有其他。

    因此,美国教育工作者对智商的痴迷可能导致许多具有潜在天才的人被排除在最能培养其独特才能的教育机会之外。

    ---
    * 贝尔实验室团队发明晶体管的领导者威廉·肖克利 (William Shockley) 和物理学家路易斯·阿尔瓦雷斯 (Luis Alvarez)。

    • 回复: @res
  275. @res

    观察你对证据的反应是很有趣的。

    意思是我推翻了你的观点,你的狡辩是微不足道的。

    • 哈哈: res
  276. utu 说:
    @res

    显然你没有得到它,包括非线性论文。看论文中的公式(3)。 g 是 SNP 的值,可以是 0、1 或 2。在我们的例子中,只有两个等位基因,因此假设 g 是 0 或 1。变量 g 的这些值来自方程 (1) 中矩阵 A 的行。式(3)中除了g之外没有其他变量。

    你做了什么?您从两个变量 g1 和 g2 构造了新变量 x:

    如果 (g1+g2==0) 则 x=0
    如果 (g1+g2==1) 则 x=1
    如果 (g1+g2==2) 则 x=2

    然后

    y = ax^2 + bx + c

    这是依赖于三个条件语句的函数。显然,在 Hsu 公式 (3) 中您看不到这样的变量,它仅取决于 g 值。虽然您的方法可行,但需要额外的编程代码来创建指向两个特定 SNP 的函数,并且可能需要额外的代码来在优化时处理此问题。

    在上面#259 的评论中,我证明了您以繁琐且缺乏想象力的方式实现的预期效果是通过具有混合项的双线性函数获得的:

    y=a*g1+b*g2+c*g1*g2

    它已经存在于 Hsu 公式(3)的第一个线性项和第三个混合项中。二次中项对此是多余的。

    现在请注意,实际上您的 x=g1+g2 。将其代入三项式中,您将得到具有两个线性项、混合项和两个二次项 g2^1 和 g2^2 的二维多项式。我是否说过我的二维多项式是完成这项工作的最低次数多项式?但是你的多项式有两个额外的项。如果将其插入配合中,您将有两个额外的自由度,这可能会导致问题,特别是在未确定的情况下。

    虽然我并不是说你的笨拙方法行不通,但它肯定不是 Hsu 的评价函数中会使用的方法,而且因为你在这次讨论中多次引用了他的论文,甚至警告我说我还没有读过它,我假设您了解数学问题的本质。也许你有阅读理解问题。我承认你读得很好。你总是有 1 propos 袖子里有纸来支持你的论点。这样的人有时被称为博学之人。有时,一个人可能会读得太好,因为当你阅读时,你不会自己进行创造性的思考。没有时间去思考你可以声称属于自己的原创想法。你的导师没有告诉你要在阅读和思考之间做出正确的平衡吗?和我一起上大学的许多女性都擅长阅读。这似乎是一种女性化的东西。您最近检查过您的 23 号染色体吗?

    • 回复: @res
  277. res 说:
    @CanSpeccy

    我不会太执着于路易斯·阿尔瓦雷斯智商不高的说法。他是安妮·罗伊杰出科学家研究的一部分: https://en.wikipedia.org/wiki/Anne_Roe#Body_of_work
    所以她的论文中有很高的智商测试结果: http://www.amphilsoc.org/collections/view?docId=ead/Mss.B.R621-ead.xml

    她的书第 162-164 页谈论了言语分数。两名实验物理学家在语言测试中的得分低于 40 分(原始)。如果其中之一是阿尔瓦雷斯,那么就有很大的机会解释特曼的结果(我的理解是特曼使用的测试存在言语偏见)。值得注意的是,所有物理学家在数学测试中都达到了(高)上限。安妮·罗伊还进行了空间测试。

    Thompson 博士,您或 ISIR 的其他人有机会获得 Anne Roe 的论文吗?每个科学家都有 10 年后的死亡限制,但大多数都超过了这个限制。我只看到一个还没有超过这个极限的人(他去年去世,享年 101 岁): https://en.wikipedia.org/wiki/Jerome_Bruner

    • 回复: @CanSpeccy
  278. Santoculto 说:
    @CanSpeccy

    智商并不像诗歌或国际象棋游戏的掌握程度那样衡量成就。

    • 回复: @CanSpeccy
  279. AP 说:
    @CanSpeccy

    但你为什么认为养育 30 个孩子和总共 11 个[不同!] 的女性并不表现出智慧呢?

    你认为为什么会这样?

    大多数人的智力目标不就是追求快乐吗?

    让我们跟随你的推理。也许我们可以在多产的男人中添加瘾君子,作为你们世界上最“聪明”的人。

    很明显,这也符合进化论,这家伙所拥有的心态在智商主义消亡后很长一段时间内仍将存在。

    在你的世界里,《白痴统治》是否真的描绘了一个比现在变得更加聪明的世界?

    撒哈拉以南非洲人目前是地球上最聪明的人,而日本人是最不聪明的人吗?

    • 回复: @CanSpeccy
  280. @res

    两名实验物理学家在语言测试中的得分低于 40 分(原始)。如果其中之一是阿尔瓦雷斯,那么就有很大的机会解释特曼的结果(我的理解是特曼使用的测试存在言语偏见)。

    我们又来了。阿尔瓦雷斯的智商很高,但他的语言成绩却很低,这让他很失望。

    看,智商要么是一个东西,要么不是。但特米特人和诺贝尔奖获得者未能晋级的案例表明事实并非如此。

    智力有多个方面,人们或多或少擅长的方面也有所不同。总体而言,女性在某些方面比男性更好,男性在其他方面总体上比女性更好。因此,比较它们是没有意义的。他们有不同的智力。

    即使在物理学教授和低能者之间进行比较也是如此。诚然,教授在任何事情上都可能比低能者更擅长,但也不一定。学者的例子就证明了这一点。像德里克·帕拉维奇尼这样的人在大多数方面都会被归类为低能儿,但他所拥有的音乐天赋可能是任何诺贝尔物理学奖获得者都无法比拟的。

    帕拉维奇尼是一个引人注目的例子,他的心态以高度专业化的方式发展,几乎可以肯定,在这个过程中,他占用了神经资源,而不是另一个人本来可以用于其他用途的资源。那么德里克·帕拉维奇尼和路易斯·阿尔瓦雷斯谁最聪明呢?这是一个荒谬的问题,他们的智力是不可通约的,就像你和我的智力,或者任何其他两个人的智力在不太极端的程度上是不可通约的一样。

    再说一次,“特曼使用的测试存在言语偏见”,这清楚地意味着没有测试可以测量这种称为智商的纯粹事物,因为这种纯粹的东西不存在,它是通过添加不同的测试分数得出的产物。心理能力。

    • 回复: @Santoculto
    , @James Thompson
  281. @Santoculto

    智商并不像诗歌或国际象棋游戏的掌握程度那样衡量成就。

    或者幽默,或者音乐性,或者艺术原创性,或者动觉整合,或者任何真正重要的东西。

    令人惊奇的是,尽管对智商进行了各种大肆宣传,但没有令人信服的证据表明,取得非凡成就的人的智商与众不同,即高于特曼笔下的一位小天才,而后者令人失望地未能表现出天才。人们怀疑这是因为大多数天才除了在他们的特殊成就领域之外实际上没有非凡的天赋。

    难道这个被认为是最高智商的美国公民在现实生活中是一名保镖、一名工人,现在住在一个偏僻的农场里,在那里养了几只山羊。 (我猜最后一个表现出了某种智慧。)

    • 回复: @Santoculto
  282. utu 说:
    @res

    对于三个等位基因,您将执行五次多项式,对吧?我总是忘记你是一名工程师。只要它有效,管道胶带就可以。

    • 回复: @res
  283. Santoculto 说:
    @CanSpeccy

    我认为出色的成就往往与分项测试的相关性大于与测试的相关性,例如,在语言类比方面表现出色,但不一定在语言智商方面表现出色。如果一般“智力”/认知完全或相当程度地导致天才/我的意思是更高的创造力+有利的心理特征,那么所有具有较高一般认知技能/分数的人都将处于或将达到天才水平,但它不会发生。

    我读到“数学早熟人才”的 Res 示例并不是较高/最高智商的人的微不足道的样本,因此最好使用更多高认知/智商聪明人的随机样本,并优先在学术职位上了解这些人是否具有相似的比率的成就。要在学术界工作,人们必须具备一些“有利”的心理特征,例如从众倾向。同样重要的是要知道这些在数学上早熟的人是否取得了天才级别的巨大成就。

    但同样,智商衡量的不是成就潜力,而是(认知)潜力。智商测试远不是一个完美的测量方法,因为有一系列被心理测量学家误解的概念,但至少很难得出智商测试毫无意义或不能测量任何重要东西的结论。

    事实上,智商只是对认知总体趋势的一个相对肤浅的确认(我所说的)。如果我在某些认知任务上总是表现出色,那么智商测试预计会通过更高的分数来反映这一点。如果我从很小的时候起就一直在母语中表现出丰富的词汇知识,那么预计至少在某些语言分测试中我会表现出这一点。智商测试的基本功能是在排名中对人群认知能力进行组织或分层。

    在我看来,心理测量学经常让我生气的是他们对创造力和理性的理解有多糟糕。两者都如此重要,不能与他们所谓的“智能”完全分开,因为它们补充了这种马赛克。也许理性在概念上是不同的,因为它表达了一些“智力产品”或目的,而智力本身(连接点/模式识别的原始能力)是达到这一目的的手段。

    大多数聪明人并不那么理性,这个简单的事实已经表明我们正在谈论两类不同的人:我的意思是工人和思想家,而不是伪知识分子。我常说,智商更多地衡量的是普通工人的技能,而不是思考者的技能。

    如果我们在日常选择中始终以道德方式行事,那么当前对道德或良性/或更好的理性同理心的“蔑视”已经表现出对两者的某种程度的无知。许多人将道德与保守主义或宗教混为一谈,但事实并非如此。

    对于“情报研究人员”来说,创造力应该占有特殊的地位,因为我们拥有的一切都是创造力,我听起来很重复,但这只是显而易见的事情。具有高度创造力的人发明了一些东西,无论其固定领域如何,其他人都可能会学习他/她的发明,而心理测量学家所说的“唯一更聪明的人”将是那些能够快速学习这一点的人。创造力在心理测量学中很少占有一席之地,而“智力”/认知才是主要主题。

    • 同意: CanSpeccy
  284. Santoculto 说:
    @CanSpeccy

    如果两种观点都考虑在内,那就太好了。个人和比较/集体。

  285. @AP

    让我们跟随你的推理。也许我们可以在多产的男人中添加瘾君子,作为你们世界上最“聪明”的人。

    你陷入了无可救药的混乱之中,并且决心不以明智的方式思考任何事情。

    以退化和早逝为代价来追求药物引起的短期快乐是不明智的。

    • 回复: @AP
  286. res 说:
    @utu

    对于三个等位基因,您将执行五次多项式,对吧?

    大多数人似乎并不觉得有必要在实践中这样做。有没有人?也许你可以找一个例子。你的评论是优秀工程的对立面。除非必要,否则不应引入复杂性。

    让我们以一些参考资料来结束本次交流,以防其他人想更好地理解 GWAS。

    全基因组关联分析和分析后询问指南 http://onlinelibrary.wiley.com/doi/10.1002/sim.6605/full

    摘要:

    本教程是一个学习资源,概述了基本过程并提供了用于实施完整的全基因组关联分析的特定软件工具。还提出了分析后可视化和询问潜在新发现的方法。使用免费开源 R 统计计算和图形软件环境、用于生物信息学的 Bioconductor 软件和 UCSC 基因组浏览器来说明应用程序。来自 PennCATH 冠状动脉疾病研究的 1401 名个体的 861,473 个分型单核苷酸多态性的完整全基因组关联数据用于说明。所有数据和代码以及其他教学资源均可通过统计基因组学项目中的开放资源公开获取: http://www.stat-gen.org。 © 2015 作者。医学统计由 John Wiley & Sons Ltd 出版。

    请注意第 4.1 节中的以下内容:

    每个 SNP 的假设潜在关联遗传模型(例如显性、隐性或加性)将影响结果的发现;然而,由于单核苷酸多态性 (SNP) 数量众多且与结果的关系通常未表征,因此通常会选择单一的加性模型。在这种情况下,如所提供的代码所示,每个 SNP 表示为相应数量的次等位基因(0、1 或 2)。值得注意的是,基于替代模型(例如显性或隐性)对 SNP 变量进行编码非常简单,并且所描述的关联分析的进行方式相同 [26, 27]。

    他们再次使用0表示。我认为utu在这个帖子中对此的评论(当他认为这只是我的想法时,哈哈!)清楚地表明他对自己在这里谈论的内容有多少了解。

    第 11 章:全基因组关联研究: http://journals.plos.org/ploscompbiol/article?id=10.1371/journal.pcbi.1002822

    摘要:

    全基因组关联研究 (GWAS) 在过去十年中已发展成为研究人类疾病遗传结构的强大工具。在这项工作中,我们回顾了 GWAS 的关键概念,包括常见疾病的结构、常见人类遗传变异的结构、捕获遗传信息的技术、研究设计以及用于数据分析的统计方法。我们也对 GWAS 之外的未来充满期待。

    概述很好,但我特别喜欢这些数字:1. 疾病等位基因效应谱,2. 连锁和连锁不平衡,以及 3. 间接关联。

    虽然图 1 是疾病关联,但我认为它在概念上也与数量性状的 GWAS 相关。特别是值得思考不同人群中的 GWAS,以及 SNP 等位基因的人群频率如何可能意味着相关 SNP 对一个人群中的 GWAS 显得重要,而在另一个人群中则不然(LD 和间接关联当然是这种情况可能发生的其他原因)。这是(这可能无法正确嵌入):
    http://journals.plos.org/ploscompbiol/article/figure/image?size=large&id=info:doi/10.1371/journal.pcbi.1002822.g001

    遗传病例对照研究中的基本统计分析: https://www.ncbi.nlm.nih.gov/pmc/articles/PMC3154648/

    其重点是二元特征而不是定量,但有一些很好的概述材料(例如术语表),并详细介绍了工具的使用。

    摘要:

    该协议描述了如何在基于人群的遗传关联病例对照研究中进行基本统计分析。所描述的步骤包括(i)适当选择疾病模型的关联和相关性测量; (ii) 适当选择关联测试; (iii) 结果的可视化和解释; (iv) 考虑适当的方法来控制多重测试; (v) 复制策略。假设之前没有使用 PLINK、R 或 Haploview 等软件的经验,我们将描述如何使用这些流行的工具处理单核苷酸多态性数据,以便进行关联测试以及可视化和解释结果。该协议假设已经执行了数据质量评估和控制,如之前的协议中所述,以便识别并删除被认为有可能给研究带来偏差的样本和标记。早期方案中也讨论了病例对照研究的研究设计、标志物选择和质量控制。该协议大约需要 1 小时才能完成。

    • 回复: @utu
  287. AP 说:
    @CanSpeccy

    你陷入了无可救药的混乱之中,并且决心不以明智的方式思考任何事情。

    我只是反映了你自己的论点。你说:“大多数人的智力目标不都是为了追求快乐吗?”

    以退化和早逝为代价来追求药物引起的短期快乐是不明智的。

    那么在你的世界里,活得长久的瘾君子就“聪明”吗?那些在《美丽新世界》中食用 Soma 的人怎么样?在你看来,他们非常聪明?与拥有 30 个孩子和 11 个女人的福利者一样聪明,甚至比他更聪明?我试图帮助你完善你个人对智力的看法。

  288. @CanSpeccy

    您的这段话是否公正地概括了您的总体立场?

    智力有多个方面,人们或多或少擅长的方面也有所不同。总体而言,女性在某些方面比男性更好,男性在其他方面总体上比女性更好。因此,比较它们是没有意义的。他们有不同的智力。

    • 回复: @CanSpeccy
  289. 那么在你的世界里,活得长久的瘾君子就“聪明”吗?

    我没有将任何具体行动描述为明智的。我将追求快乐描述为人类智力普遍应用的对象,当然,我认识到,人们从爬山到解方程等一系列非凡的事情中获得快乐,并且有些人在追求快乐时运用了比其他人更多的智力。其他的。

    问题是,你是一个典型的 每日邮件 一名埃塞俄比亚人与 30 名妇女生下了 XNUMX 个孩子,这让读者感到愤怒。你也是一个典型 乌兹网 读者相信,智力在某种程度上是最高的美德,可以用单一的线性尺度来衡量。

    这种情感的混乱和糟糕的科学导致了欧美人为自己创造的、或者更确切地说是允许那些操纵他们的人创造的愚蠢的、死胡同的文明。

    要了解你如此鄙视的非裔美国花花公子的行为,你应该阅读一些有关进化论的有力阐述。对于那些只对大致轮廓感兴趣的人来说,达尔文可能是最好的。我推荐全集。读过达尔文之后,你会更好地理解为什么人们一般都喜欢做繁殖自己同类(基因)的事情。你也会更好地理解你自己对书中所描述的角色的巨大繁殖成功的愤怒。 每日邮件。一方面,你可能正在为此付出代价,或者至少为该阶层个体的繁殖成功付出代价——通过福利制度来付出代价。

    因此,你真正应该生气的是你所在的愚蠢的政府制度,它利用像你这样的人来促进像德斯蒙德·哈克特这样的人的生育成功。

    • 回复: @Santoculto
    , @res
    , @AP
  290. @James Thompson

    詹姆斯,我不确定你的问题引向何处,但你引用的段落现在看起来和我写它时一样有效,尽管正如哲学家所说,有很多东西需要解开。我否认智商是一个东西的原因是我否认智商的重要性,尽管不是存在。 g.

    智力是一个单一事物的观点(即, g),从大脑生理学、解剖学和发育的角度来看,可以线性尺度测量似乎非常难以置信。此外,统计数据显示不同认知功能测试分数之间的平均相关性较低,因此并不能证实认知功能的中心性。 g.

    是的,所有精神活动都必须依赖于大脑和全身生理学的一些特征:能量代谢、膜特性、血流等。因此,如果所有方面之间的相关性较低,人们不应感到惊讶心理能力,情况似乎就是如此。此外,正如乌图煞费苦心地证明的那样,因素分析可能揭示出不止一个影响所有心理能力的共同因素: g1、g2、g2、gx、yz.

    但如果不认识到大脑的复杂性以及经验、教育和环境因素可能改变心理功能的多种方式,人们似乎就不太可能拥有一种关于人类智力的良好理论。重复我在另一条帖子中所说的话,大脑功能取决于八十多种神经递质,每种神经递质都有自己的生物合成途径,可能还有自己的分泌、再摄取和分解代谢机制。大脑的每个部分和每个大脑功能同样依赖于所有这些神经递质的可能性有多大?我想说,不太可能,在这种情况下,有理由假设不同心理设施的速度和力量不会有很好的相关性,紧密相关性的缺乏反映了不同心理过程的生理基础中由基因决定的变化。

    心理活动被划分在大脑区域和结构中,这些区域和结构显示出个体之间质量的比例变化。也出于这个原因,人们不应该期望不同心理能力之间存在密切的相关性。

    心智能力取决于发育变化,包括与学习相关的神经发生,这意味着即使没有其他导致变异的原因,人们也应该预期智力的各个方面会因文化、教育、动机等而发生变化。此外,强烈参与特定的心理活动很可能涉及神经资源的重新分配,这可以解释天才中强迫症的高频率。

    正是因为智力不是一种东西,而是认知能力的集合,其中一种认知能力的发展可能会以牺牲另一种认知能力的发展为代价,所以天才很少出现在全面发展的高智商个体中,而更多地出现在古怪的人身上。痴迷者,甚至像许多学者一样,大脑受到损害。

    • 同意: John Jeremiah Smith
    • 回复: @Santoculto
    , @utu
  291. Santoculto 说:
    @CanSpeccy

    好吧,根据埃塞俄比亚人口趋势和全球人口趋势,我是一名“每日邮报读者”,但从假设阅读本文的人往往是工人阶级开始,所以我可以推断你的论点是古典主义的,因为你正在判断通过社会阶层来分析智力和性格,首先分析为什么这类人往往会对这个消息感到恐惧,特别是如果他们已经对当前的人口趋势和可能的全球影响有足够的了解,例如许多埃塞俄比亚人逃离人口稠密和极度贫穷的国家并在英国的一些工人/中产阶级社区找到新家。

    请记住,智力是一个在物种内部进化的概念,但其所有特征中最永恒的是最大限度地达到平衡的真正能力,并且有许多孩子没有正确的方法来保持他们的能力,这似乎是基于当前现实的不现实。

    • 回复: @CanSpeccy
  292. res 说:
    @CanSpeccy

    因此,你真正应该生气的是你所在的愚蠢的政府制度,它利用像你这样的人来促进像德斯蒙德·哈克特这样的人的生育成功。

    至少你和我(不是你回复的人)在某些事情上达成了共识。但有一些更正。这个名字叫德斯蒙德·哈切特 (Desmond Hatchett),他似乎(当时?)有 24 个孩子,而不是 30 个。此外,他来自田纳西州诺克斯维尔,而不是埃塞俄比亚: http://archive.knoxnews.com/news/local/magistrate-hes-prolific-but-knoxville-dad-doesnt-really-have-30-kids-ep-360705348-356973501.html/

    这是 2016 年关于这个聪明人的文章: http://wate.com/2016/03/15/wanted-suspect-arrested-in-east-knoxville-after-brief-chase/

    还有后续。显然他们现在只剩下二十几个孩子了: http://www.wbir.com/news/crime/prolific-ex-convict-faces-assault-stalking-charges/84244220

    笑点(尽管事实上这并不好笑)是进攻(强调我的):

    袭击了一名女子 怀了他的孩子 据记录,4 月 XNUMX 日在伯特利大道和萨里街附近。指控称,他开着他的道奇复仇者撞了她乘坐的一辆汽车。

    我想他还没有完成。

    • 回复: @CanSpeccy
  293. Santoculto 说:
    @CanSpeccy

    “智能是我们知道是什么但不知道如何描述它的东西”。你正在以非常字面的方式处理它。
    如果智力不是一个东西,那它是什么?
    有一些新的证据表明环境因素对人格的影响很小。嗯……我们可以在他们之间、在我们自己的家庭中看到这一点。事实上,对于有洞察力的人来说,没有必要通过“科学研究”来展示或证明我们自己所看到的东西。在这种情况下,我不需要科学或心理学来了解/感知我和我的兄弟之间的行为差​​异,并推断这种差异不能仅或从根本上用“环境因素”(家庭环境)来解释,如果这种情况从小就发生并且具有相同的环境因素。遗传学更好地解释了同卵双胞胎之间令人难以置信的相似性以及我们通常在异卵双胞胎兄弟姐妹中发现的这种差异。

  294. @Santoculto

    []拥有许多孩子却没有正确的方法来养育他们似乎是不现实的……

    什么是“正确”的养育孩子方式?

    无论是在北美还是在欧洲,福利似乎对那些依赖福利的人来说效果很好。

    事实上,这就是似乎正在发生或即将发生的事情:

    (1)政协以性“教育”为名的胡言乱语对欧洲人生育能力的抑制,以及大规模移民对住房和工作的竞争造成的高住房成本和低工资的经济压迫,而高额税收又加剧了这种影响使替代移民人口成倍增加的福利。

    (2)最终的人口崩溃,可能是精英煽动的,由于核冬天、瘟疫、特大海啸、粮食歉收等等,剩下的人口主要是非洲裔亚洲人,也是未来几代“欧洲人”的基础发展。

    所以,是的,福利可能会很好地服务于欧洲土地的继承者。

    • 回复: @Santoculto
  295. Santoculto 说:

    也许我们是对的,许多心理测量学家在捍卫行为遗传规则时并不是出于好意,但这并不意味着他们的观点是错误的,他们的某些意图可能非常粗鲁、麻木不仁、缺乏哲学,例如指责较低的人该群体犯罪倾向较高的认知技能。

  296. @res

    他来自田纳西州诺克斯维尔,而不是埃塞俄比亚

    是的,但我的意思是 埃塞俄比亚 HL Mencken 所采用的意义上的。

    至于所有的事实都是错误的,人们还能从像《新闻周刊》这样的有信誉的新闻来源那里指望什么呢? 每日邮件.

  297. Santoculto 说:
    @CanSpeccy

    让家人尽可能承担起责任或可以联系到。

    福利的方式就是被利用,你说的,是精英怂恿的。

    这是一种反向殖民,采用了所谓“精英”所采取的相同方法:人民对抗人民。

    建议:不要试图纠正我的语法,这不是这里的主要主题。

    • 回复: @Santoculto
  298. AP 说:
    @CanSpeccy

    我没有将任何具体行动描述为明智的

    您写道:“智力是什么,即促进生存和繁殖成功的行为能力”

    在我发表了一篇关于那个拥有 30 个孩子的男人的有趣文章后,你进一步澄清道:“为什么你认为养育 30 个孩子和总共 11 个[不同!] 的女人并不表现出智慧?

    对于大多数男人来说,至少在最初阶段,养育孩子难道不是一种愉快的经历吗?对快乐的追求难道不是大多数男人将他们的智慧投向的目标吗?

    看起来很清楚,而且肯定符合进化论,这家伙所拥有的心态在智商主义消亡后很长一段时间内仍将存在。”

    所以你至少强烈建议你认为哪些行为是智力的标志。

    问题是,你是一位典型的《每日邮报》读者

    嗯,实际上没有。这是通过谷歌出现的东西。我从来没有读过它。我什至没有阅读整篇文章。

    既然你在这么简单的事情上都错了,我们可以假设你把更复杂的问题搞得更糟。但你在更简单的事情上却错了:

    一些埃塞俄比亚人竟然为 30 名妇女生下了 XNUMX 个孩子,这让他感到愤怒。

    我发表了关于这个人的文章作为你的智力观念的一个例子。我没有写任何关于我对他和他的困境的感受(如果你想知道的话,这是有趣而不是愤怒)。你现在正在编造事情。我想这凸显了你追求真理的方法?还是投影?

    他不是埃塞俄比亚人,而是美国人。这表明了你对待事实的态度。

    你也是一个典型 乌兹网 读者相信,智力在某种程度上是最高的美德,可以用单一的线性尺度来衡量。

    当然,智力与美德无关。

    • 回复: @CanSpeccy
  299. @AP

    当然,智力与美德无关。

    当然?亚里士多德认为,理性引导人走向美德。

    他不是埃塞俄比亚人,而是美国人。这表明了你对待事实的态度。

    正如已经指出的,我在门肯的意义上使用了埃塞俄比亚人,即非裔美国人,后者在门肯时代并不流行。 (门肯很可能在考虑萨姆·约翰逊饰演的阿比西尼亚王子拉塞拉斯时采用了“埃塞俄比亚”一词(“埃塞俄比亚”是阿比西尼亚帝国的另一个名称))。

    至于其余的,我的精力水平下降了。如果你坚持认为男人不会运用他们的智慧来获得快乐,我很高兴你持有这种观点。我只是碰巧认为这是不正确的。

  300. utu 说:
    @res

    res,你已经精神错乱了。你从哪里得到这个: 我认为utu在这个帖子中对此的评论(当他认为这只是我的想法时,哈哈!)清楚地表明他对自己在这里谈论的内容有多少了解。 您向谁发表此评论?你认为除了我之外还有其他人读过这些东西吗?或者这是你在谈话时的习惯,你也会对路人发表关于你的对话者的评论?

    不管怎样,我们用什么数字来编码不同的等位基因是无关紧要的。可以有 0,1,2 或任何其他等距序列,或者我认为,不等距序列仍然会导致等效的解决方案。它会以某种方式在数学上计算出来。当我第一次思考 DNA 与性状契合度的数学化问题时(显然不需要阅读,因为我不需要阅读,而是自己解决问题),我得出的结论是我们可以有 0,1,2,3, 3 或任何其他数字来做到这一点。当时我假设有四种可能性,因为每个 SNP 表达都有 A、C、G 和 T 版本,我是这么想的。然后当我第一次读Hsu的论文时,我很困惑为什么他只有0,1,2种可能性XNUMX、XNUMX、XNUMX。我仍然感到困惑。 你有解释吗?我缺少什么? 我知道基因可以有很多等位基因,比如五个甚至更多,但这是因为它们可能包含几个定义它们的 SNP。但一个 SPN 可以有多少个等位基因呢?三四个?

    每个 SNP 关联的假设基础遗传模型(例如,显性、隐性或加性……通常选择单个加性模型。

    e)

    我们对单一的加性模型不感兴趣,因为它在数学上是微不足道的,但在数值上仍然极具挑战性。让我们再次看一下 Hsu 公式(8)及其线性项、二次项和混合项。问题是这个公式是否可以解决孟德尔的问题?这就是为什么我需要构造执行孟德尔定律的二维多项式。你明白知道吗?我已经证明,两个线性项和一个混合项负责处理具有两个等位基因的一对 SNP,以模拟孟德尔显性/隐性的东西。对于 2 个等位基因,需要 3 个术语。我们已经在 Hsu 公式 (6) 中得到了它们:8 个线性项、2 个混合项和 1 个二次项以及一个偏移量。如果方程 y=Ax 中的向量 y 的均值被删除,那么根据定义,偏移量实际上可以设置为零,这在所有拟合过程中都是常见的做法。

    因此,这里最重要的结论是:Hsu 公式(8)可以解释最多三个等位基因的孟德尔显性/隐性因素。 它不能解释 4 个等位基因(如果可能,请参阅上面上一段中我的问题)。对于 4 个等位基因,需要公式 (8) 中的三次混合项。

    让我们回到您提出的方法。对于两个等位基因,您需要公式 (8) 中已存在的二次项。到目前为止一切顺利,但您需要在矩阵 A 上附加一列由两个 SNP 值之和定义的 x 值。现在的问题是,您是否应该从矩阵 A 中删除包含这两个 SNP 值的两列,因为如果保留它们,则会创建更复杂的效果:两个 SNP 通过 x 作为孟德尔效应和两个 SNP 单独产生的效果小瓶线性和其他术语。那么你会怎么做?也许你可以在你已经读过的某本书中找到它。

    现在,如果您想采用您的方法(我知道这不是您的;您在某处读到它,因为您所知道的东西都不是真正您的)对于三个等位基因,您将必须生成 x 的 5 次多项式并将 x 附加到矩阵A 并将 Hsu 的公式 (8) 更改为包含 5 次方、4 次方和 3 次方。但你似乎并不担心: 大多数人似乎并不觉得有必要在实践中这样做。 这是您的典型工程方法:因为我们从未听说过它,所以我们不担心。

    但最重要的问题是,当寻找预测函数时,我们不知道哪些 SNP 将提前出现在预测函数中。 Lasso 方法应该找到这些 SNP,其中一些 SNP 会单独起作用,而一些 SNP 会串联起作用以产生孟德尔效应。所以我们不知道需要使用哪对 SNP 来提前计算您的 x。是的,计算机可以对所有可能的对执行此操作,但是 n 列的矩阵将膨胀到 n(n-1)/2 列。因此,正如您所看到的,您的方法(很抱歉,不是您的方法,而是来自您以敬畏的态度勤奋阅读的某本书)很糟糕。

    底线是Hsu非线性论文中的公式(8)可以考虑SNP的部分优势,即孟德尔的东西。你现在可以感谢我学习了这一点。

    • 哈哈: res
    • 回复: @utu
  301. utu 说:
    @CanSpeccy

    也不要忘记你的路灯谬论。

    为什么汽车工业或航空航天工业没有通过一些因素分析得出自己的g值?可以为每辆汽车或飞机分配一个号码,每个人的生活都会变得更加简单。我什至知道他们的 g 可以使用什么单位:美元。汽车价格是g吗?自称研究最复杂进化产物的人们对还原论的痴迷从何而来?他们从哪里得到这种肆无忌惮的?

    如果您从多个测试组中获得 g,则该 g 并不是该组测试可以产生的性能、成功、收益的最佳预测因子……g 是各个测试的无限多个可能的线性组合之一。就 PCA 而言,g 是使解释方差(瑞利商)最大化的那个。但是,如果您想使用相同的测试并预测工作成功,则不同的线性测试组合将最大化该预测,并且不会成功。此外,这还取决于您进行预测的受试者样本是什么。如果样本中从事数学工作的人多于写作,那么您将使用不同的线性测试组合,而样本中作家多于数学家或音乐家,而且他们都不会是 g,但是,是的,g 将与他们。

    如果我们要求每个 IT 部门生产人工智能机器并进行教学,然后将其发送到人工智能奥林匹克竞赛,在那里机器将接受各种测试,包括 Raven 矩阵或 Binet 等,我们也会得到一些 g 并通过以下结果决定获胜者:最高g。但如果我们不断地向测试电池中添加新的测试,g就会不断变化,获胜者就会重新洗牌。直到卡林建造的莫斯科大学人工智能机器生气了(这是智能的明显标志)并用激光枪杀死了所有其他机器(重要的进化功能)。当然,在这种情况下我们无法选择 g,但无论如何我们都会有一个胜利者。进化不是由 g 驱动的。

    • 回复: @CanSpeccy
  302. @utu

    自称研究最复杂进化产物的人们对还原论的痴迷从何而来?

    对权力的渴望,以及因此在严格的等级制度中为每个社会成员提供精确数字坐标的动力。

    如果我知道你的 g, 如果你的 g 低于我的 g,或者至少如果我能让你认为也许你的 g 低于我的 g 并且肯定低于 g 我代表的人,那么我就可以坚定地、永远地把你放在你的位置上。

    他们从哪里得到这种肆无忌惮的?

    这不是肆无忌惮,这只是他们被付钱去做的事情。

    现代大学不是一个自治的学者团体,而是一个由政府资助和指导的官僚机构。今天,一个学者,例如艾萨克·牛顿,永远不会被允许与政府打手对抗,例如 精神病机会主义者杰弗里斯法官。教授们做的是教授们受薪做的事情,那就是弄清楚如何让亿万富翁永生,发明药物以增加大型制药公司的利润,并设计更好的方法来控制人们的思想,并排挤或杀死那些不愿做出反应的人。命令。

    今天,詹姆斯·D·沃森或学术界其他任何人只要稍微偏离政治正确性,就会受到对无害的傻瓜和诺贝尔奖获得者蒂姆·亨特爵士的残酷对待, 不仅是伦敦大学学院的人渣教务长迈克尔·阿瑟教授,甚至是英国皇家学会主席保罗·纳斯爵士(Sir Paul Nurse),他与亨特分享了 2001 年诺贝尔生理学和医学奖。保罗爵士没有表现出丝毫犹豫 用膝盖顶住亨特的腹股沟,学会与这个倒霉的胸部“保持距离”,并将他从皇家学会委员会中除名。

    自由社会唯一真正的希望可能确实是拥有强大激光的卡林人工智能机器。

    • 回复: @utu
    , @utu
  303. utu 说:
    @utu

    在丢失资源的同时,您是否查阅过大量的书籍资源来了解为什么是 0,1,2 而不是 0,1,2,3?为什么不考虑 4 个等位基因?显然不仅仅是我有这个问题:

    https://biology.stackexchange.com/questions/37020/why-do-almost-all-snps-have-two-alleles
    但是有 4 种常见的核苷酸,A、T、C 和 G。为什么在某个位置有 3 或 4 个等位基因不常见呢?对我来说,唯一可能的方法是忽略链,因此在任何给定位置“A”和“T”被认为是相同的等位基因,“C”和“G”被认为是相同的等位基因。是这样吗,还是我理解有误。

    这个人问了一个正确的问题,但他的答案偏离了推测,而那些在这个帖子上回复他的人都是常见的混淆者,他们就是不能说“我不知道”。

    对于一个被认为是 SNP 的基因座,首先必须有全世界人口中至少 1% 拥有与大多数人不同的等位基因。如果每个人都有相同的 A、T、C 或 G 值,则该基因座将没有预测能力来解释任何性状的任何差异。矩阵A(y=Hsu的Ax)中没有一列是常数。我怀疑这 4 个等位基因一定是非常非常罕见的,尽管据我所知从技术上讲并不排除发生的可能性。 3 个等位基因可能也不常见,但确实存在。

    将 DNA 数学化为性状预测函数时,对于任何 SNP,A、T、C 或 G 编码为 0,1,2,3、0、1、2 可以是任意的。这意味着对于一个 SNP,A=3、T=2、C=3 和 G=0,而对于另一个 SNP,它可能是 A=1、T=8、C=XNUMX 和 G=XNUMX。多项式Hsu的公式(XNUMX)并不关心。

    无论如何,如果它们遇到具有 4 个等位基因的 SNP,那么 2 阶 2D 多项式 Hsu 的公式 (8) 将不足以解释 SNP 之间的显性/隐性相互作用。

    • 回复: @utu
    , @res
  304. utu 说:
    @utu

    “当失去时”,显然应该是“当 LOLing。 ” 拼写检查做到了这一点。

  305. res 说:
    @utu

    您是否查阅过大量的书籍资源来了解为什么是 0,1,2 而不是 0,1,2,3?为什么不考虑 4 个等位基因?显然不仅仅是我有这个问题:

    我的评论 245 涵盖了。这是对你的回复。你需要加强你的阅读理解能力。

    如果你想处理三等位基因的情况 0, 1, 2, 3 不起作用。对于 3 个等位基因(称为 a、b、c),您有 6 种可能的组合:aa、ab、ac、bb、bc、cc。而这不能用一个数字来表示。在那里你确实需要更多的变量。

    本文发现,在相当小的中国样本中,大约 0.5% 的 SNP 位点是三等位基因: http://onlinelibrary.wiley.com/doi/10.1111/ahg.12114/abstract

    此链接引用 rs2063690 作为四等位基因 SNP: https://macarthurlab.org/2014/11/18/a-guide-to-the-exome-aggregation-consortium-exac-data-set/
    并继续断言“大约 7% 的 ExAC 位点现在是多等位基因的,并且随着我们样本量的增加,这个比例将会增加。”

    当然,要考虑的重要问题是这些额外等位基因的群体频率是多少?它们真的值得花精力将其纳入分析吗?

    你试图取笑我整理研究文献来支持我的论点的能力,这很可爱。这也会让你看起来像是一个非常不认真的人。

  306. utu 说:

    如果你想处理三等位基因的情况 0, 1, 2, 3 不起作用。对于 3 个等位基因(称为 a、b、c),您有 6 种可能的组合:aa、ab、ac、bb、bc、cc。而这不能用一个数字来表示。在那里你确实需要更多的变量。

    0,1,2,3 – 暗示有四个等位基因,但你谈论的是三个。

    不能用一个数字来表示 – 不,可以,根据你的方法和糟糕的五次多项式。或者通过二维多项式,其中项 x, y, xy, x^5,y^2 表示三个等位基因,额外的三次项表示四个等位基因。所以你是对的,2 个等位基因不能用 Hsu 公式(2)中可用的形式表示。这一切都在我最后的两、三条评论中。看来你还是没有明白公式(4)的数学原理。

    感谢您提供的链接以及评论 #254 中关于多等位基因 SNP 问题的链接。所以这和我想的差不多。并且 Hsu 确实仅使用 0,1,2,因为 4 个等位基因案例很少见,如果它们不是宿主公式(8)将必须扩展到立方混合项。

    非常不认真的人。 不,你错了。我是非常认真的。阿谀奉承者常常将不敬误认为是严肃的运气。我只是不敬,一点也不敬畏。

  307. utu 说:
    @CanSpeccy

    OT:IQ 业务有一点我无法破解并暴露其弱点。这就是遗传性问题。双胞胎研究显然是有道理的。我还没有找到对这些研究的致命批评。即使是“离婚据说有 50% 的遗传力”这一可笑的事实或事实也还不够好。参见杰·约瑟夫的书:

    然而最近我意识到,通常情况下的问题被隐藏在每个人都接受并忘记的早期假设中。这就是巧手。假设方差是遗传 G 方差和环境 E 方差之和。大家都做出这样的假设:

    V=V(G)+V(E)

    这一假设允许人们得出双胞胎研究中常用的 Falconer 遗传力公式 H^2=2(r_mz-r_dz)。

    然而,当 G 和 E 在给定性状的“生产”中相互作用时,方差的划分必须不同,并且事实证明它不是唯一的,这是问题的关键。可以这么写

    V=V(G)+V(E&G) 或 V=V(E)+V(G&E)

    其中 V(E&G) 和 V(G&E) 都是取决于 G 和 E 的提醒,它们在删除可归因于基因的最大方差(第一种情况)或删除归因于环境的最大方差(第二种情况)后留下。案件)。这些公式都不允许我们推导福尔科纳公式。

    我们将遗传力定义为 V(G)/V。让我们将“环境”定义为V(E)/V。它们的总和不再是 1,因为 V≠V(G)+V(E),这违背了普遍接受的假设。

    这是什么意思?如果我们使用像Hsu这样的SNP来估计遗传力V(G)/V,并说获得60%的结果,这并不意味着有40%的“环境”,因为遗传力加上“环境”不再等于100%。总和实际上大于 100%。所以60%的结果解释得比一般理解中的“60%”还少,因为需要解释的东西比“100%”还多。如果我们能够像基因一样以数学方式测量环境,并用它通过类似 Hsu 的方法来解释方差,我们可能会得到“环境”,即 55%。按照一般理解,人们会得出这样的结论:遗传力是 45%。这个结论会不会是错误的?遗传力是60%还是45%?基本上,虽然我们将遗传力定义为方差比,但该定义在数据和测量中不可强制执行,因为我们无法真正唯一地定义 V(G)。因为我们可以将其定义为

    V(G)=VV(E&G) 或 V(G)=V -V(E)=V(G&E)

    两个公式中的 V(G) 不相等。

    从数学上讲,它可以表述如下。假设您在 2D XxY 域中有 f(x,y) 函数。找到两个函数 g(x) 和 h(y) 来最小化以下 RMS 指标:

    ||f(x,y)-g(x)||和 ||f(x,y)-h(y)||

    你会发现对于大多数函数方差 V(f)≠V(g)+V(h)。

    当 f 对称时,即 f(x,y)=f(y,x) 时,g 和 h 相同,但仍不意味着 V(f)=V(g)+V(h)。

    当f(x,y)为双线性时,则V(f)=V(g)+V(h)。这是遗传性研究中常见但隐含的假设。然而,这种假设是没有根据的。

  308. utu 说:
    @CanSpeccy

    还有一点。当我们听说智商的遗传率为 60%,因此只有 40% 的方差是由环境解释时,这是不正确的。这就是他们的绝招。因为超过 40% 的方差很可能可以用环境来解释。所以,是的,他们定义的遗传力可能是 60%,尽管 Falconer 的公式肯定不准确,但这确实意味着环境只能解释 40%。

    我们无法计算或测量环境可以解释多少方差,因为我们无法进行相同的实验,在这些实验中我们可以保持环境固定且可测量,就像它们对基因的作用一样。我们需要门格勒博士来做这件事。

    所以遗传力是60%,环境可以解释60%的变异这一说法并不矛盾;这实际上可能是真的。

    我希望这能改变看待遗传决定论者的陈述和数字的视角。

    • 回复: @Santoculto
    , @CanSpeccy
  309. Santoculto 说:
    @utu

    智商本身就是一种文化产品,就像蓝眼睛是一种长期(或不太长)的生物文化产品一样。如果你的社会正在选择具有学术/语义技能的人,那么从长远来看或不太长期(取决于选择压力的水平),我们将在人与社会要求之间存在可变的对应关系,以达到最低程度的“成功”/适应。同样,西方人可能无法在日常任务/进化要求方面胜过因纽特人或原住民,这对于这两个群体来说都是如此
    但文明往往意味着文化积累,因为它反映了人类战胜自然环境挑战的能力,因此我们可以得出结论,虽然因纽特人很难在自己的环境中超越英国人,但事实可能恰恰相反。

    许多这种“环境”是人类生物认知发展以及智商延长或延迟所固有的。但是,如果语言(语言技能是人类智力的基础)已经是硬件以及一些非常本能的非语言倾向,那么那些从小就具有较好语言技能的人(至少可以增加词汇量和对单词的理解)将擅长IQ 任务。

    关于英国人和因纽特人。很明显,如果我们将随机的英国人放在因纽特人的土地上,而没有事先或逐渐适应(因纽特人的真实情况),那么大多数人都会灭亡,但一群平均智商在 100 左右并且具有某些先前要求的心理特征的人会死于这种类型的环境中。很可能他们完全有能力像因纽特人那样适应,甚至在表现上超越他们。

    同卵双胞胎之间的许多差异是由于自然差异造成的,如果他们不是克隆人(同一个人),这些差异也会发生在他们身上。

    如果遗传学不能解释或决定人类行为,为什么因纽特人在不同的环境中不能像在原来的环境中那样适应得那么好?都怪小白??
    出色地*

  310. @utu

    你的数学超出了我的想象。不过,根据 本文,这表明在某些情况下,基因对双胞胎智商的贡献几乎为零!

    关于这个主题的重要内容似乎是 Feldman 等人 1997 年发表的论文。在《科学杂志》中。 但是三重 AS 的那些愚蠢的家伙甚至不会让你在不付费的情况下看到摘要。事实上,我已经为此付出了代价,作为该协会的成员已有 40 多年的历史,包括费尔德曼论文的发表日期,但由于《科学杂志》有一个不诚实的编辑,我取消了订阅,现在我被拒绝访问我已经付费的文章:可怜。

    但无论如何,需要记住的是,所有这些研究都不是关心智力,而是关心智商,这就是所谓的心理学界相当愚蠢的专业人士提出的:一个线性尺度,人们认为可以直接将爱因斯坦的智力与莎士比亚的智力进行比较,或者将耶稣的智力与阿道夫·希特勒的智力进行直接比较。

    • 回复: @res
    , @utu
  311. res 说:
    @CanSpeccy

    第一项研究中值得注意的一点是,它的种族平衡与整个美国截然不同:“这对双胞胎被归类为 43% 的白人、54% 的黑人和 3% 的‘其他’。”

    我发现有趣的另一件事是,低遗传力似乎是由 DZ 相关性增加和 MZ 相关性减少引起的。我想知道为什么会这样。以及它的复制效果如何。

    以更传统的方式分析数据也能提供丰富的信息。我们将这些配对分为高于中位 SES 和低于中位 SES 的配对,并使用随机效应方差分析(Guo & Wang,2002)来估计两个 SES 组中同卵双胞胎和异卵双胞胎的组内相关性。在低 SES 组中,异卵双胞胎的组内相关性为 63,同卵双胞胎的组内相关性为 68,与 h2 10 和 c2 58 一致;对于高 SES 组,DZ 双胞胎相关性为 51,MZ 双胞胎相关性为 87,与 h2 72 和 c2 15 一致。

    您的第二个链接的格式不正确的版本位于 https://www.researchgate.net/publication/13818005_Twin_Studies_Heritability_and_Intelligence

    这是一系列质疑双胞胎研究结果有效性的信件以及回应。我认为将其称为“关于该主题的重要文章”是很恰当的。

    当然,如果没有一些诸如“所谓心理学界相当愚蠢的职业”之类的言论,CanSpeccy 的评论就不完整。

    PS 这是一项英国研究,看起来与第一项研究有些相似: http://journals.plos.org/plosone/article?id=10.1371/journal.pone.0030320

    一些相当不同的结论:

    成果

    我们发现低 SES 家庭的智力差异较大,但在 5000 个年龄段中 GxE 相互作用的证据却很少。功效计算表明,需要约 0.25 对双胞胎的样本量才能检测小至 80 的智力遗传成分的调节,功效约为 11%——遗传力差异为 53% 至 2%,在低( −2 标准差, SD) 和高 SES (+XNUMX SD) 系列。目前的研究发现,每个年龄段的样本量都在这个范围内,遗传因素对智力的影响并没有减弱。然而,我们发现低SES家庭的较大差异是由于环境效应(环境与环境的相互作用)的缓和造成的。

    结论

    在英国代表性样本中,低社会经济地位家庭和高社会经济地位家庭的遗传对智力的影响相似。儿童的共同经历似乎解释了社会经济地位较低的人智力差异较大的原因。

    这项研究的一个好处是,他们考察了不同年龄段的智商,而不仅仅是 7 岁。这项研究还包括 10,000 多对双胞胎,而 Turkheimer 研究中的双胞胎有 320 对。

    以下是他们对 9 岁 IQ 的最高/最低 SES A/C/E 成分的比较。有趣的是,A 的所有结果都在 0.4 左右或稍低一些。

    与我们在 0.4 岁时的估计值 ~0.5 相比,这个 ~9 似乎有点低 https://www.ncbi.nlm.nih.gov/pubmed/23919982
    但 Turkheimer 的 0.72 和 0.10 数字都与该链接中显示的 0.4 岁时的 ~7 显着不同。

  312. 感谢 ResearchGate 上 Feldman 论文的链接。事实上,我自己也发现了这一点,但如果我用一个对科学交流造成障碍的服装创建一个帐户,我就该死了。他们为什么不简单地允许开放访问呢?他们是否认为他们最终可以通过这样一个蹩脚的网络获利,或者什么?

    但也感谢您提供其他信息。我将研究这一点。

    至于:

    当然,如果没有一些诸如“所谓心理学界相当愚蠢的职业”之类的言论,CanSpeccy 的评论就不完整。

    我的意思是,只要看看它的历史:

    弗洛伊德的精神分析被恰当地描述为科学时代最奇怪的遗迹之一;

    沃森主义和斯金纳主义的行为主义否认人类心灵的现实,并认为轰炸越南是操作性条件反射的理性政策的一部分。

    今天,智商主义是荒谬的,这种观点认为,从莎士比亚到艾萨克·牛顿,从耶稣基督到阿道夫·希特勒,所有的思想都可以从愚蠢到聪明进行线性评估。

    那些相信这种谎言的人难道不是太愚蠢,至少是太容易上当受骗吗?

    • 回复: @Dieter Kief
  313. @CanSpeccy

    从莎士比亚到艾萨克·牛顿,从耶稣基督到阿道夫·希特勒,所有的思想都可以从愚蠢到聪明进行线性评分

    智商是一种做出预测的工具。因此,一旦我们谈论过去,它就没那么有用了。

    • 回复: @CanSpeccy
  314. @Dieter Kief

    智商是一种做出预测的工具。

    您认为?或者也许会创建一个独裁的、自上而下的等级制度。

    无论如何,它并没有做出很好的预测,正如特曼研究通过从选定的潜在天才中排除仅有的两名将继续获得诺贝尔奖的候选人所证明的那样。

  315. utu 说:
    @CanSpeccy

    让我再试一次。

    虽然我不认为双胞胎研究和遗传性问题可以完全消除,但我发现了一个重大遗漏,可能会改变我们对遗传性和结果的看法。它从常用且看似无可争议的假设开始,即某些性状的方差V可以分解为基因V(G)和环境V(E)的方差之和,即V=V(G)+V(E)。根据这一假设,推导出 Falconer 公式 H^2=2(r_mz-r_dz)。如果没有这个假设,福尔科纳公式必须有一个无法轻易估计的附加项。

    实际上,基因和环境相互作用,因此方差不能以这种方式划分。要提取 V(G),必须首先定义它,并且没有唯一的方法。虽然 V(G)/V 的遗传力定义似乎很合理,但这并不是因为我们真的不知道根据哪个定义使用哪个 V(G)。

    这一发现的结果是,当我们听说遗传力被测量为 60% 时,这意味着环境只能解释 40% 的方差。这不一定是这样,因为如果我们使用不同的方法,在这种情况下环境实际上可以解释超过 40%。例如,遗传力为 60%,环境也解释 60% 的说法并不自相矛盾,因为它实际上可能是正确的。遗传力和“环境力”之和实际上大于100%。

    虽然这种见解不一定会改变事实,但它提供了另一种视角,并大大削弱了那些不断推动基因决定论叙事的人所发表言论的修辞力量。 如果你们声称可以用基因解释 60% 的方差,这并不排除环境也可以解释 60% 的方差的可能性。 基本上,这将是我的见解的主要结果。

    • 回复: @CanSpeccy
  316. @utu

    是的,虽然我会算数,但我不是数学家,我童年时的麻烦也许是被什么问题分散了注意力。 xy 真的是。但无论如何,要理解任何定量的东西,我需要将其具体化,我认为,在这种情况下,这并不那么困难。

    拿一个刚一岁多的孩子,一个男孩,注意他的注意力如何倾向于(在许多情况下,无论如何)集中在机械设备,或带有按钮或开关的电子设备上,这些按钮或开关在推、按或猛拉时会产生效果。由于这种反应,父母或其他照顾者很可能会生产更多的小玩意、玩具等来提供类似的干扰。结果,对因果关系的理解开始显现。

    因此,我们对某种类型的行为具有遗传倾向(根据我的观察,在男孩的妹妹的行为中不存在,或者肯定不太明显),这种行为会改变环境以促进具有这种特定遗传倾向的其他行为。这种行为会改变大脑,从而产生可作为智力的一个方面来衡量的能力。

    这将是基因型通过行为调节改变环境的明显例子,因此大脑(智力)不仅受到 G 和 E 的影响,而且还受到 G 的影响。描述,那么毫无疑问,效果是真实的,并且它可以以多种不同的方式出现。

    事实上,处于社会底层的基因似乎对智商测试结果的影响较小甚至没有影响,这表明 G 与 E 的相互作用至关重要,即,如果没有培养,智力就不会正常发展,无论遗传潜力如何或许。

    • 回复: @CanSpeccy
  317. @CanSpeccy

    智力发展在很大程度上取决于对环境的适当改变,以响应基因驱动的发展(促进进一步发展的环境改变),这一观点可以解释很多。

    比如精英、精英家族的坚持。例如,王子传统上受到智者的指导,这些人(在大多数情况下)的任务是观察智力的发展,并为其提供挑战和机遇,以促进遗传潜力的充分表达。大多数精英家庭也是如此。例如,在英国,一对一的辅导要么在家里(已故的德文郡公爵夫人黛博拉·米特福德(娘家姓)有趣地描述过),要么在著名的“公立”(即私立)学校之一,例如伊顿公学(大多数英国首相都参加过),然后是牛津大学,或者以温斯顿·丘吉尔为例(哈罗公学,在一位天才老师的指导下,他学会了构建一个完整的英语句子)。

    G on E 效应,即对先天驱动发展的培育反应的效应,可能解释了智商测试分数中的大部分(如果不是全部)跨文化和种族差异。每个人都知道犹太母亲和虎妈,但我们很少听到欧洲中产阶级推动孩子们除了棒球、曲棍球等,或者更罕见的音乐之外的任何事情。至于非裔美国家庭,有迹象表明,许多孩子从任何人那里得到的智力培养相当少。

    • 同意: utu
  318. utu 说:

    因此,不仅受到 G 和 E 的影响,而且 G 还受到 E 的影响。因此,如果这是查看您所描述的效果的有效方式,那么毫无疑问,该效果是真实的,并且它可以以多种不同的方式出现方法。

    是的,G和E是纠缠的。它们的综合效应不能减少为单独效应的总和。正因为如此,当我们问这样的问题时,群体中某个性状的方差有多大可以单独用 G 或 E 来解释,这些问题无法明确回答,而且它们不是互补的。所以G和E单独可以同时解释50%以上的方差是可能的,这并不矛盾。遗传力和“环境”加起来并不等于100%。

    在基因决定论者发动的信息战中,他们的必胜主义意在让那些支持另类叙事的人放弃任何希望。所以他们说基因可以解释 60% 的变异,因此基因更重要,因为基因暗示环境只能解释 40%。对此我的见解是让反对者说:不那么快何塞,环境也可能可以解释60%,所以在我们的叙述中环境更重要。

    此外,这一观察结果对法尔科纳公式的有效性提出了质疑,所有基于双胞胎和亲属的遗传性研究都是基于该公式。但是我还不知道如何估计与应用公式相关的误差。

    我主要通过分析他们声称使用的数学方法和假设来清理他们在床底下扫过的东西。所以我们已经知道 g 并不是他们希望其他人相信的东西。我们知道,有问题的范围限制校正方法会提高智商和生活结果之间的相关性。现在我们知道遗传力并不意味着“环境性”=100%-遗传力。

    问题是,智商主义者并没有真正被他们的对手所吸引,他们太蔑视他们,不愿意与智商主义者交谈。因此,智商主义者创建了自己的期刊,在期刊中他们没有受到真正的挑战或审查,因此可以摆脱很多废话。俄罗斯的奥赫拉纳也犯了类似的错误,即无视布尔什维克而不是当场枪杀他们。

    • 回复: @CanSpeccy
    , @reiner Tor
  319. @utu

    俄罗斯的奥赫拉纳也犯了类似的错误,即无视布尔什维克而不是当场枪杀他们。

    你说沙皇政权失败是因为它不够极权,现代之前的所有政权都是如此。但马可尼、贝尔、特斯拉、诺贝尔和其他一些人发明了创建极权国家的手段,这意味着不可能被发明。有了这些工具,国家自然就获得了首先对全体民众无可争议的警察权力,现在又获得了越来越无可争议的精神控制。

    人类独立的时代已经结束。我们现在是博格。独立思考者的时代已经结束。世界已经成为少数在权力争夺中幸运的精神病患者的玩物。他们受到政府、学术界、媒体中数以百万计的无名知识分子的服务,只要国家需要在任何地方发挥自己的作用。智商主义者只是现在占据大学、中小学和其他政府机构的众多可悲生物之一,他们的作用是将所需的模因灌输给越来越容易上当受骗的公众的头脑。

    当然,这只是重申乔治·奥威尔 70 年前的预期。奥威尔似乎只是在细节上犯了错误,他的反乌托邦愿景中的空白大部分由奥尔德斯·赫胥黎填补。

    • 回复: @CanSpeccy
  320. utu 说:

    你说沙皇政权失败是因为它不够极权,现代之前的所有政权都是如此。

    1900 年,Okhrana 只为 1000 亿国家配备了 140 名秘密警察。非常极权主义。

    现代之前的所有政权

    是的。就我个人而言,如果有一种像渴望出生前的时光这样的事情,我会怀念奥匈帝国。我很喜欢这个时期的文学。

    我认为美好的旧世界随着第一次世界大战而结束。两个大国出现了:美国人和布尔什维克是同一枚硬币的两面。 1870年至1914年的时期对欧洲来说似乎是田园诗般的时期。

    • 同意: AP
  321. @CanSpeccy

    是的,现代化已经让世界失去了很多魅力。至少在第一次世界大战之前一百年就注意到了这种影响。科布登将新兴的伦敦市描述为大温(或脓包),大约在同一时间,托马斯·卡莱尔正在哀叹世界的机械化。

    我怀疑,正是俄罗斯伟大小说家所描绘的十九世纪俄罗斯乡村生活的真正迷人形象,在很大程度上解释了西方知识分子中亲俄倾向的高发。然而,这种生活实际上也有很多缺点:例如牙痛,只能在不使用麻醉的情况下拔牙来治疗,流行病无法控制,群众极度贫困和几乎普遍的无知,以及产业工人阶级的可怕就业条件。

    尽管如此,19世纪仍然是自由主义的伟大时代,是一个改革的时代,也是一个永远改革的时代,而今天,我们所能期待的只是腐败的法西斯官僚机构的日益收紧的控制。事实上,我们所理解的人类的终结,如果这里的智商主义者有办法的话,他们将被智商超过 1000 的基因改良人类所取代,或者可能只是具有人工智能的机器。

    • 回复: @Daniel Chieh
  322. @CanSpeccy

    事实上,我们所理解的人类的终结,如果这里的智商主义者有办法的话,他们将被智商超过 1000 的基因改良人类所取代,或者可能只是具有人工智能的机器。

    这是相当可怕的,但越来越有可能的替代方案也是如此:一个由可疑智能算法运行的世界,旨在最大限度地刺激最大数量的参与者/观众的多巴胺刺激,除了盲人之外没有任何特定的“精英”目的追求玛门本身。

    对我来说,想象成为一台有意志的机器上的无名齿轮,唯一更可怕的是成为一台没有目的或意义的机器上的无名齿轮,它只会搅动我们所有人,半吸毒,分心于无休止的争论,而最终剥夺了我们任何真正的代理权或权力,而我们则在狂乱中快乐地前进,并在快乐的竞争中压垮自己,以便为了这台机器的目的更好地贬低自己。

    相比之下,不使用止痛药的牙痛拔除似乎更容易被接受。

    • 同意: AP
    • 回复: @reiner Tor
    , @CanSpeccy
  323. reiner Tor 说:
    @utu

    问题是,智商主义者并没有真正被他们的对手所吸引,他们太蔑视他们,不愿意与智商主义者交谈。因此,智商主义者创建了自己的期刊,在期刊中他们没有受到真正的挑战或审查,因此可以摆脱很多废话。俄罗斯的奥赫拉纳也犯了类似的错误,即无视布尔什维克而不是当场枪杀他们。

    我很高兴知道您希望让定义模糊的“IQist”团体的所有成员当场拍摄。

    • 回复: @utu
  324. reiner Tor 说:
    @Daniel Chieh

    最有可能的结果是人工智能陷入混乱,例如杀死全人类以增加回形针的产量,直到一切崩溃。但也有可能它只会产生更好的虚拟现实,直到所有生物人类都被困在其中,然后在最后一个人类死亡后一切都崩溃了。

    费米悖论一定有一个解释,环顾世界,我倾向于认为悲观的解释更有可能。

    • 同意: AP
  325. 费米悖论一定有一个解释,环顾世界,我倾向于认为悲观的解释更有可能。

    空间是一个很大的地方。到目前为止,我们几乎还没有看过。无论如何, 外星人可能已经在这里:他们可能是我们:外星人可能正在寻求逃离先进文明的技术暴政。现在又要再次发生这种事了。

  326. @Daniel Chieh

    人性化的最大希望可能是建立一个储备体系,让人类生活在基本静态的各种文化体系下。

    这似乎是一个合理的提议。技术文明不需要大量人类,因此从逻辑上讲,世界人口将大幅减少到几千万。然后,对于那些希望这样做的人来说,将会有足够的空间,比如说,在西北欧建立一个欧洲中世纪保护区,以容纳数百万人(伊丽莎白时代英格兰的人口)。非洲人可能会选择回归部落生活,而亚洲可能会再次成为一片几乎空无一人的土地,但会有分散的佛教徒、印度教徒等的定居点。

    穆斯林、犹太人和原教旨主义基督徒可能会对他们与生俱来的帝国主义倾向产生疑问,但技术文明当局应该能够遏制他们——伴随着耶和华、安拉或圣灵声音的奇怪的合成闪电应该保留按顺序排列。

    • 回复: @John Jeremiah Smith
  327. utu 说:
    @reiner Tor

    我只是不太擅长类比和隐喻的层次。我倾向于在双曲线方面犯错误。我想,再教育营就可以了。

    • 哈哈: reiner Tor
    • 回复: @John Jeremiah Smith
  328. Factorize 说:

    尚未有人表明,在特曼研究中,真正的治疗组是那些未纳入研究的患者。研究中的那些人是对照组。这将有助于解释仅有两名诺贝尔奖获得者没有被纳入研究的悖论。如果他们是的话,他们可能会赢得诺贝尔奖。当然,我们需要时间机器或平行宇宙来证明这样的猜想。

    那些被公众认为具有极高智力能力的人并没有继续取得任何杰出成就,这是多么令人惊讶?他们为什么需要这样做?他们赢得了比赛,然后遭受了胜利者的诅咒。赢得比赛意味着没有必要再完成任何事情。他们没有赢吗?

    正因为如此,那些没有遭受诅咒的人才是最终的胜利者。这是断点回归设计的一个简单示例。它还有助于解释为什么大多数最成功的人
    硅谷的学生从未完成他们正在努力争取的最后一个学术证书。如果他们
    那么,现实世界的进一步成功在心理上对他们有什么作用呢?

    • 回复: @CanSpeccy
  329. @Factorize

    那些被公众认为具有极高智力能力的人并没有继续取得任何杰出成就,这是多么令人惊讶?他们为什么需要这样做?他们赢得了比赛,然后遭受了胜利者的诅咒。

    好吧,你已经有了。智商专家关于为什么应该放弃智商业务的结论性论点,以避免损害我们最优秀和最聪明的人的个人前景和社会利益。

    • 回复: @utu
  330. utu 说:
    @CanSpeccy

    然而,驱使我打开基因自我认识的潘多拉魔盒的更强大的力量是知道我是谁。如果不了解密码,人们就无法确信自己的自我认知反映了客观事实。该代码应该使我们能够更好地将人们的基因型与适当的环境相匹配,从而使人们更加快乐、更加高效。

  331. Factorize 说:

    CanSpeccy,我并不希望这是一个结论性的论点,而只是一个可能可检验的假设。我预计有人可能会提出替代建议,而不是简单地接受而不反驳。

    我的建议并非没有先前研究的支持。例如,我的想法可以在外在动机的背景下理解。研究发现,当人们因学习等而获得奖励时,他们就会减少对这些活动的参与。因此,如果你要招收积极性很高的物理系学生,如果你付钱给他们,他们的内在动机就会下降,而他们的外在动机就会增加。到了某个时候,这些学生将不再对物理产生任何兴趣,只有得到奖励才会垂涎欲滴。付钱给他们做他们喜欢做的事情会导致他们不再喜欢它,最终他们会放弃它。

    这与特曼的研究有明显的相似之处。研究中的受试者获得了一系列奖励,因此结果与研究一致。

    就个人而言,我可以证实这个想法在我自己的生活中的力量。在某种程度上,我经历过与那些被排除在特曼研究之外的人类似的拒绝。因此,我试图通过在我注册的在线课程中表现出色来证明我的荣誉。我已经取得了非常极端的成就。互联网提供无限的资源来查找任何问题的详细答案或任何任务的软件。

    例如,我参加了初级德语课程,我的导师对我的表现印象深刻。我买了一个软件,用来无休止地重复德国 OW 声音。

    如果我只是遵循官方规则,那么这一切都没有必要。我本可以宣布胜利,并且永远不需要继续取得一个又一个的巨大成功。最近,一位对我参加的科目的所有期末考试进行评分的评分员告诉我,我在期末考试中表现出的知识水平是有史以来最高的。值得注意的是,我通过这所在线大学学习的课程是我以前认为自己较弱的科目。

    我成功的动力实际上来自于我想要证明自己的强烈愿望,以及对我正在学习的课程的强烈的内在兴趣。奇怪的是,不久前我收到了大学的录取通知书,以奖励我的成就。我认为拒绝这个提议是明智的。

    即使进入心理测量时代一个世纪,我们仍然不知道如何充分解放那些拥有极高能力的人的能力。这不应被视为对心理测量学的严厉批评,而只是反映人类心理学确实并不完全是我们所期望的那样。心理测量学实际上更多的是对一个陌生世界的探索。我们应该成为这个领域的观察者,而不是带着先入为主的观念进入这个领域。

    令我惊讶的是,我们的社会对社区高端人才的管理如此之差。高g是我们应该培育的财富。事实上,这在很大程度上是不正确的,这更多地反映了更广泛的社区中缺乏 g,而不是对那些能力非常高的人的控诉。

  332. Factorize 说:

    CanSpeccy,特曼研究虽然最终被拒绝,但考虑到两位诺贝尔奖获得者的事实可以被理解为心理测量学的辩护。

    特曼的研究包括大约 1500 名非常聪明的年轻人,其中大约一半是女性,其中许多人是家庭主妇。考虑到这个时代,这应该不足为奇。我认为期望其他一些非 g 相关选择会确定另外两位诺贝尔奖获得者是不合理的。

    • 回复: @James Thompson
  333. @Factorize

    即使不是完美的预测器,良好的预测器仍然如此。

    Steve Hsu 有一些关于特曼研究的有趣帖子

    这直接涉及到“你没有识别出最佳表现者”的论点,它是描述一般原则的一种方式,即即使它并不完美,人们也可以拥有一个有用的预测器。这是我认为最有力、最明显的论点,但其他解释可能会有所帮助。

    http://infoproc.blogspot.co.uk/2014/07/success-ability-and-all-that.html

    下一篇着眼于智商与人格维度

    http://infoproc.blogspot.co.uk/2011/04/earnings-effects-of-personality.html

    最后一个是关于声称或据说智商低的诺贝尔奖获得者的。这是典型的詹森:清晰且信息丰富。有趣的是,他让肖克利、克里克和费曼中的一位参加了另一项特曼测试,而其中一位得到了很高的分数。 20年后还能挖掘出来吗?顺便说一句,在接受智商测试之前,肖克利不会与任何人交谈。粗鲁,但有效。

    http://infoproc.blogspot.co.uk/2012/05/jensen-on-g-and-genius.html

    谈论诺贝尔奖获得者:

    我认识的一位物理学家习惯于邀请诺贝尔奖获得者参加他所在院系的研讨会,他发现诺贝尔奖获得者经常怀疑自己是否应得这一荣誉,并担心聚集在一起的物理学家会在他们开始演讲时发现归因错误。开车送他们去科室,他必须尽力安抚他们的紧张情绪。因此,即使在这些经过挑选的人员中,也有优先顺序。幸运的是,作为一位继诺贝尔奖获得者之后的客座心理学家,我没有受到任何如此严厉的评价,并且因为我的外围娱乐价值而被接受。

    • 回复: @CanSpeccy
    , @res
  334. Santoculto 说:

    博学者是极其罕见的,或者说是一个真正的神话,没有这样的人可以在最高水平上掌握一切。事实上,似乎许多(如果不是大多数)天才不仅因他们的最高成就而闻名,而且还因他们犯下的重大错误而闻名,其中一些人因完全缺乏理解而变得有趣。天才是所有正常人的异常版本。

    智商主义者相信大多数高智商的人都是博学者。

    事实上,我们发现具有不对称认知特征的人比具有超对称认知特征的人更常见。并且不要忘记智力的心理部分,哪里也会存在不对称性。

    大多数人(如果不是每个人)都已经有了自己的认知优势和弱点,这意味着他们的遗传和神经生理学景观。

    认知不对称不仅在人类中是一种极其常见的特征,而且在非人类生物中也很常见。

    区别在于,至少在神经典型人群中,大多数人往往具有隐含的不对称认知和心理特征。

    适应本身就推动了专业化。

    只有当他说人类进化的下一阶段将是创造普遍的更聪明的“人”时,因式分解才是正确的。同样,大多数健康的人类天生就有能力快速学习一种语言,在“未来”,大多数健康的人类在第一个童年时期就天生有能力快速学习许多其他人类文化知识,相关知识。

    但我相信人类智力的两个最重要的特征:创造力和理性[不要将其与逻辑或伪精神病、冻结情感同理心和仅使用认知同理心混淆]必须得到普遍化。

    • 回复: @CanSpeccy
    , @CanSpeccy
  335. @James Thompson

    在接受智商测试之前,肖克利不会与任何人交谈。粗鲁,但有效。

    证实他在大多数方面都是雅虎人和白痴!

    • 回复: @James Thompson
    , @utu
  336. res 说:
    @James Thompson

    最后一个是关于声称或据说智商低的诺贝尔奖获得者的。这是典型的詹森:清晰且信息丰富。有趣的是,他让肖克利、克里克和费曼中的一位参加了另一项特曼测试,而其中一位得到了很高的分数。 20年后还能挖掘出来吗?顺便说一句,在接受智商测试之前,肖克利不会与任何人交谈。粗鲁,但有效。

    您有需要进行智商测试的肖克利的推荐人吗?我对更多背景感兴趣。

    关于 Jensen/Shockley/Crick/Feynman,克里克 1971 年的这封信似乎很相关: https://profiles.nlm.nih.gov/ps/retrieve/ResourceMetadata/SCBBNM

    这是肖克利/费曼的联系: http://people.seas.harvard.edu/~jones/shockley/First%20Impression%20of%20Shockley.html
    他们显然有一个共同的导师:菲利普·麦科德·莫尔斯 https://books.google.com/books?id=cRb_qzEwWWAC&pg=PA39&lpg=PA39&dq=shockley+feynman+friends

    这让我觉得詹森正在谈论费曼和肖克利。了解该测试的结果将会很有趣。

    埃米尔 (Emil) 拥有詹森 (Jensen) 采访的完整 PDF 版本: http://emilkirkegaard.dk/en/wp-content/uploads/Discussions%20of%20Genius%20-%20Interview%20with%20Arthur%20Jensen.pdf
    也可以在 https://www.slideshare.net/usavel/genius-intelligents

    詹森提到的文章是 http://psycnet.apa.org/record/1997-08230-023
    PDF在 http://arthurjensen.net/wp-content/uploads/2014/06/Giftedness-and-Genius-Crucial-Differences-1996-by-Arthur-Robert-Jensen.pdf
    詹森在那里以有利的措辞提到了艾森克的精神病/创造力思想。他还简短地提到了克里克和肖克利(但与上述没有任何关系)。

    汤普森博士,您写过这些文件吗?我有兴趣阅读你的想法。

    谈论诺贝尔奖获得者:

    我认识的一位物理学家习惯于邀请诺贝尔奖获得者参加他所在院系的研讨会,他发现诺贝尔奖获得者经常怀疑自己是否应得这一荣誉,并担心聚集在一起的物理学家会在他们开始演讲时发现归因错误。开车送他们去科室,他必须尽力安抚他们的紧张情绪。因此,即使在这些经过挑选的人员中,也有优先顺序。幸运的是,作为一位继诺贝尔奖获得者之后的客座心理学家,我没有受到任何如此严厉的评价,并且因为我的外围娱乐价值而被接受。

    恕我直言,邓宁-克鲁格效应和“冒名顶替综合症”都是真实存在的。

    需要明确的是,我所指的邓宁-克鲁格效应的一个特殊方面是,能力最高的人倾向于低估自己。这并不是能力较低的人高估自己的更常见的方面:

    • 回复: @CanSpeccy
    , @James Thompson
  337. @Santoculto

    同意。

    但智商论者永远不会承认智商无法决定心理活动各个领域的能力。

    他们说,智商是真实存在的 g,该死。

    但事实是 g 仅反映心理能力之间非常有限的相关性,或者可能存在多个共同因素, g 1、2、3、x、y 和 z,是 IQists 不会承认或无法理解的东西。事实上,如果他们真的承认这一点,那么他们的骗局就会破产。他们再也不能声称通过测量智商,他们就可以衡量一个人的水平,以及他未来的关键。

    • 回复: @Santoculto
  338. @Santoculto

    博学者极其罕见或者是一个真正的神话

    那些具有非凡记忆力的人,如果他们努力的话,至少会表现出博学的外表。

    根据奥利弗·萨克斯(Oliver Sachs)的轶事证据,完美记忆的潜力可能几乎是普遍存在的,并且即使在没有特殊能力的人中,由于脑损伤或与年龄相关的精神衰退,也可能被揭露出来,例如过去的经历会以完全清晰和详细的方式重温。

    据推测,这种完全回忆通常没有用处。确实,伟大的国际象棋棋手都拥有近乎完美的记忆,至少对棋局有近乎完美的记忆,但他们却因无法摆脱记忆而发疯。

    我的父亲有着超强的记忆力,他所做的一切几乎都是A,包括二战期间的飞行,但他的心态被线性逻辑所支配,以至于他无法接受哪怕是最简单的假设。有些东西是或不是。对他来说,这两种情况都无法忍受,也不值得考虑。因此,我相信博学有其局限性,一个每个人的智商都达到数百的社会可能会构成一个非常迟钝和缺乏想象力的社区。

    • 回复: @Santoculto
  339. @res

    能力最高的人倾向于低估自己。

    当你获得诺贝尔奖时。哈哈

    虚假的谦虚,我亲爱的孩子。

    • 回复: @Santoculto
  340. Santoculto 说:
    @CanSpeccy

    我还发现有趣的是,他们为什么或如何认为一个答案比其他认知测试(即多项选择)绝对正确……。也许我们正在处理不同的认知风格/偏好,而不是缺乏正确的推理,以及为什么他们不要求进行此测试的人来发展他们的推理路线,以了解他们如何或为什么选择这个而不是那个答案......

  341. Santoculto 说:
    @CanSpeccy

    是的,但博学的人也必须是超级思考者,而不仅仅是超级记忆者。我认为性格会影响认知风格/偏好,但不会影响心理状态,我说的是性格而不是气质。我的意思是,两个具有非常非常相似的认知[先天]构成的人可能由于性格差异而具有完全不同的认知风格/偏好。

    也许太多的收敛智能往往会不利于认知可塑性/又称创造力。收敛基本上是完美主义的语义记忆,能够很好地记住大多数信息。

    似乎更有创造力的人往往会出现记忆力问题,或者记忆力不强。

    • 回复: @CanSpeccy
  342. Factorize 说:

    线程中的人可以帮助我吗?

    当我还是一名高中生时,我所有课程的主要参考资料都是教科书。
    对于大多数课程,我从未咨询过任何其他来源。如果作业中提出的问题与教科书上的材料不太相似,那么我会发现很难找到正确的答案。

    然而,在这个信息技术的新时代,任何作业的问题都可以在网上无休止地研究。互联网资源几乎是无限的。我的在线大学实际上免费提供了 20,000 多种在线期刊数据库。

    我不确定的是,为什么媒体空间(包括本博客)没有对这一发展进行更多讨论?凭借现在可用的资源,我的知识范围得到了极大的扩展。如果一定要我猜的话,我会认为这让我的“智力”提高了 50 点 IQ。我承认我的 g 保持不变,尽管我对各种问题进行研究和制定有意义的回答的能力发生了巨大变化。
    我的“智力”定义为通过使用信息工具更好地理解世界的能力,它已经发生了很大程度的变化,而我的生物认知能力g却保持不变。

    那些离现场较近的人一定也注意到了这个变化。
    今天的高中生可以利用的资源比他们的课本要广泛得多。
    考虑信息和计算机环境的变化如何影响智力的含义应该引起心理测量学家的兴趣。奇怪的是,也许在智力功能的生活体验中实际上可能发生的一个非常大和重要的转变可能会被简单地变得不可见,因为即使所有船只的水位都发生变化,相对位置仍然保持不变。

    • 回复: @CanSpeccy
    , @James Thompson
  343. @Santoculto

    是的,但博学的人也必须是超级思考者,而不仅仅是超级记忆者。

    什么是超级思想家?逻辑规则的数量相当有限,但大多数人和人类以外的物种(例如黑猩猩、海豚、鲸鱼和大象)都可以很好地使用它们。

    解决大多数问题的关键要求无疑是相关信息,这就是为什么出色的记忆力足以胜任广泛的能力——只要个人有广泛的兴趣。然而,如果一个记忆力超强的人把所有的时间都投入到国际象棋或其他一些狭隘的专注活动中,那么他们将不是一个博学者,而是一个偏执狂。

    但天才的关键肯定既不是分析能力,也不是记忆力,而是将想法以新颖的方式组合在一起的能力。这就是智商测试者甚至没有声称能够测量的想象力。爱因斯坦在想象力方面表现出色,尽管他的数学天赋似乎有限,因此他依靠朋友(例如格罗斯曼和哥德尔)帮助他用数学方式表达他的想法。

    • 回复: @Santoculto
    , @utu
  344. @Factorize

    考虑信息和计算机环境的变化如何影响智力的含义应该引起心理测量学家的兴趣。

    一本好书,或者更好的是,一个有好老师指导的教程,可能抵得上一千篇互联网文章和 U 型管视频,甚至可能更多,因为互联网上所说的很多内容实际上并非如此。当然,人们在互联网上收集信息时需要非常谨慎。

    由于我们今天的大部分知识与我们的祖先不同,是通过纸上的墨水或屏幕上的像素来获得的,而不是来自对世界的直接经验,所以每个人都需要的是认识论的强化训练。

    即便如此,从电脑屏幕上推断出来的任何东西都不具有直接观察的可靠性,所以也许我们对现实本质的了解确实不如我们的祖先,他们没有受益于互联网、电视或大众媒体。

    • 回复: @John Jeremiah Smith
  345. Santoculto 说:
    @CanSpeccy

    什么是超级思想家?

    某人非常有能力思考/使用/处理/记忆和新颖的信息,并从中提取连贯性和/或只是珍贵的真理。

    然而,如果一个记忆力超强的人把所有的时间都投入到国际象棋或其他一些狭隘的专注活动中,那么他们将不是一个博学者,而是一个偏执狂。

    好吧,但我认为博学者是很难存在的,这是一个基于肤浅想法的神话,即高智商/高智商/认知智能的人聪明得令人难以忍受,但智商雷达没有追踪到一些东西:互动中的心理弱点与认知。

    我相信现在我们有两种智力弱点:认知弱点和心理弱点。认知弱点是最明显的,这就是我们无法“学习”的能力。例如,我几乎不存在空间技能。但心理弱点可能无处不在,甚至或特别是我们的认知能力。我们的个性改变了我们的看法。

    但天才的关键肯定既不是分析能力,也不是记忆力,而是将想法以新颖的方式组合在一起的能力。

    我认为这是这三个要素的结合,但你所说的“分析能力”似乎对创造力也非常重要,这是另一个神话,即高度创造力的人不具备分析能力,他们往往兼具直觉和分析能力。凭直觉的人可能非常有创造力,但还达不到天才的水平。

    这就是智商测试者甚至没有声称能够测量的想象力。

    因为许多人认为这根本不相关。

    爱因斯坦在想象力方面表现出色,尽管他的数学天赋似乎有限,因此他依靠朋友(例如格罗斯曼和哥德尔)帮助他用数学方式表达他的想法。

    是的,他的数学天赋具有分析能力。与想象力相结合产生了分析活动。我认为你过于注重天才水平创造力的灵感方面,而忘记了蒸腾方面,其中认知技能以及与创造力的结合极其重要。

    我非常“有创造力”,当然不是天才水平,甚至接近,但我知道我非常痴迷于自己狭隘的兴趣,也许我也很偏执,而且我一天中的大部分时间都在非常有趣地思考/关于我的兴趣的自然方式,这也意味着我总是分析和重新分析我的想法和想法,有时我有洞察力[其中大多数不太好]。

    • 回复: @CanSpeccy
  346. utu 说:
    @CanSpeccy

    他依靠朋友(例如格罗斯曼和戈德尔)帮助他用数学方式表达他的想法.

    这是更长但仍不完整的助手列表:

    路德维希·霍夫
    内森·罗森(Nathan Rosen)
    瓦尔特·迈耶
    瓦伦丁·巴格曼
    布鲁里亚·考夫曼
    奥托·斯特恩
    科尼利厄斯·兰佐斯(Cornelius Lanczos)
    雅各布格罗默
    巴内什霍夫曼
    赫尔曼·明茨
    马塞尔·格罗斯曼

  347. Factorize 说:

    CanSpeccy,谢谢您的回复。
    我认为这是一个重要的话题,值得讨论。

    我保留了高中时的作业。
    我最近的大学作业质量发生了巨大的变化,因为我可以访问互联网。到目前为止,我的在线课程的每项作业的每个问题都经过了详尽的研究。

    在高中作业中,如果我被要求讨论癌症。我可能会去学校图书馆,签出可能是唯一一本可能在 5-10 年前或更早出版的关于癌症的书,并且可能找到了一些通用科学期刊,其中可能有最近发表的关于癌症的文章。

    我相信,任何见证过这一转变的老师都会验证通过这种转变,学生作业质量所发生的巨大转变。

    最近,当我被要求在最近的在线生物学课程中讨论癌症时,我提供了对当前研究的高度详尽的描述以及多个当代参考文献。通过我的期刊访问和开放获取的在线资源,我可以在舒适的家中阅读数百万篇有关癌症的文章。我通过挑战教科书中提出的对该主题的肤浅治疗来完成我的答案,并提出了一种更引人注目的方式来概念化癌症,并提出了一个更相关的治疗框架。如果没有当前的计算机资源,这在很多年前是不可能实现的。

    癌症具有很高的相关性。
    如果出现这样的挑战,就如何应对这种挑战提出有意义的连贯意见的能力必须在某种程度上说明适应性管理个人生活的能力(智力)。从个人经验来看,我相信信息资源的变化极大地改变了我应对此类挑战的能力。

    • 回复: @CanSpeccy
  348. @Santoculto

    想象力本身也许不是一种能力,而是对好奇心的反应。有些人只是想了解事物,或者他们想完成一些事情——娱乐、养活饥饿的人、治愈病人、窃取皇冠上的宝石。正是渴望知道什么是未知的,或者做不可能的事情,这肯定会激发想象力,以多种方式配置已知的内容并添加假设元素,直到出现对问题的一些新的、可信的观点。

    智商论者似乎缺乏这种能力。以任何你想要的方式挑战他们,但他们会坚持认为 IQ = 智力。如果某人在智商测试的数字部分得分排名前 1%,而在语言测试部分得分排名后 1%,那么他们会说这个人的智商处于平均水平。指出那个人是一个数学明星和一个语言白痴,他们会说,不,智商是一个单一的东西,正如存在的证明 g.

    指出 g 几乎无法解释认知不同方面测试中分数之间的差异,并且它们可能会改变主题。那 g 可能是由于一些微不足道的因素,例如进行测试的大气中的氧分压,或脑血流量,或膳食碘摄入量,或一些或十几个其他生理或解剖变量,它们将不理你。

    在某种程度上,我认为智商主义者拒绝放弃他们声称能够将人类从 A+ 天才评定为 Gamma- 减白痴的说法,这纯粹是职业自私的问题。但人们不得不怀疑是否也缺乏想象力。

    • 回复: @James Thompson
    , @Santoculto
  349. @Factorize

    分解,

    您热衷于利用互联网来推进学业,值得赞扬。但是,您应该小心,不要被大量的可用数据淹没。毫无疑问,互联网确实为您提供了百万篇关于癌症或您可能希望了解的其他主题的学术期刊文章,但您无法阅读一百万篇文章。最好的情况是,在一周内或任何你必须撰写论文的时间,你可以阅读十几篇左右的文章。这就是互联网的挑战,如何选择你真正需要的东西。这就是为什么一位称职的导师是无价的:引导你找到关键的东西,同时让你远离学术垃圾的泥沼。

    并不是说我会阻止您自行研究文献。但需要时间来找到解决办法,学会区分有意义和无意义,并认识到大多数文献都集中在琐碎的问题上,提出不当,调查不恰当,调查结果写得不好,统计数据具有误导性或完全无效。事实上,许多文献根本不真实。

    关于您对癌症的兴趣,请记住,在过去 50 年中,仅美国政府就在癌症研究上花费了超过 XNUMX 万亿美元,但如果您被诊断患有睾丸癌或乳腺癌(我不确定你是男性还是女性)你最好的选择是手术。换句话说,尽管在癌症研究上花费了数十亿美元,但在治愈大多数形式的疾病方面却没有取得多少成果。

    这就是为什么一个好的导师如此有价值。他们可以指导您定义关键问题、关注关键事实。在这方面,与大多数专上学生一样,与那些在你之前一两代人相比,你可能是不幸的。然后大学教授以教师的身份任职。他们很容易接近,并准备好投入时间进行一对一的指导。今天看来情况并非如此。事实上,顶尖大学的本科生可能与他们就读的院系中最杰出的人物没有任何联系。这是一个可悲的发展,在我看来,求助于互联网上未经消化的大量材料并不能替代适当的指导。

    然而,我认为,除了所谓的学术教学人员之外,学生还可以向他们寻求建议。在网络上,似乎有很多人提供教程服务。我认为这可能是一种新的学习方法的基础,尽管以这种方式寻找方向显然还有很大的机会。不管怎样,我祝你学业一切顺利。

  350. @CanSpeccy

    当然是进攻性的。众所周知,他性格粗鲁,对同事也很严厉。 Jensen刚刚参加了考试,然后就遇见了他。我想知道参加智商测试的是克里克还是费曼。可能是费曼,因为他对心理学感兴趣,尽管对其方法感到失望。

  351. @res

    嘿,这些都是有趣的参考资料。你应该把它写下来。詹森和肖克利智商测试。:不知道我在哪里读到的。他没有告诉我这件事。我们有过一些通信,他总是很乐意提供他的推荐信。他给我寄了一大堆论文,我想在 1990 世纪 XNUMX 年代我可能已经丢失了。从我在索引中看到的情况看,Miele 的对话书中没有,尽管它可能仍然存在。 Jensen认为这本书对他来说很公平。
    是的,聪明人在邓宁-克鲁格和冒名顶替综合症之间摇摆不定。

  352. @Factorize

    是的,对于一个有智力兴趣的人来说,这就是活着的最好时机。我们拥有有史以来最大的图书馆,几乎可以访问每个书架。我们才能真正站在巨人的肩膀上。然而,由于我们自身的局限性,其中很多内容需要花费一生的时间才能理解。就我而言,我认为它增加了我的临时知识,但对我的心智能力的增加却很少,而且很可能没有任何作用。

  353. @CanSpeccy

    如果某人在智商测试的数字部分得分排名前 1%,而在语言测试部分得分排名后 1%,那么他们会说这个人的智商处于平均水平。指出那个人是数学明星和语言白痴,他们会说,不,IQ 是一个统一的东西,g 的存在证明了这一点。

    事实上,当四个韦克斯勒因子中的任何一个相对较低时,许多临床心理学家拒绝计算全量表智商。这些都是一个判断问题,但趋势与你所描述的方向相反。

  354. Santoculto 说:
    @CanSpeccy

    想象力本身可能不是一种能力,而是对好奇心的反应

    当我经常想象时,我并不总是这样做,因为我很好奇。想象力也是为了乐趣。

    做某件事的特殊能力

    也许想象力和联觉一样都是一种预科能力。

    智商主义者,似乎缺乏这种能力

    .

    你说想象力不是一种能力。好吧,我同意智商主义者往往缺乏想象力,至少对其主题缺乏想象力,这是他们的“核心”之一,当人们对某个主题绝对正确时,即使他们不检查事实,这意味着这个东西对于他们。这是他们的生存事业之一。

    以任何你想要的方式挑战他们,但他们会坚持认为 IQ = 智力。如果某人在智商测试的数字部分得分排名前 1%,而在语言测试部分得分排名后 1%,那么他们会说这个人的智商处于平均水平。指出那个人是数学明星和语言白痴,他们会说,不,IQ 是一个统一的东西,g 的存在证明了这一点。

    我可能有一种迂腐的印象,认为智商或认知测试的许多错误恰恰在于它们的使用不足。似乎许多心理学家以及该领域的其他专业人士并不真正对这种心理测量测试的结果感到好奇,或者过于笛卡尔式而无法超出一般规则并尝试更深入地进行这种可比较/可测量的分析,更多的“风险行为” ”。

    是的,我认为智商在先验上是一个统一的东西,正如我所说,他们相信理想的 g 就是真实的 g。也许这就像男人和女人的认知差异一样。它们都具有相同的一般认知特征,但这些特征的分布和表达方式不同。理想的 g 是一般认知特征,并以非常对称的轮廓表达,但我们知道,很明显可以看出,人们通常有十一个轮廓,其中一些轮廓更加不平衡。

    指出 g 几乎不能解释认知不同方面测试中分数之间的差异,他们可能会改变主题。

    我所知道的关于 g 的解释似乎相当“经济”。他们说,当一个“人”在某件事上表现出色时,他/她往往在其他事情上也表现出色。真的**

    我发现它非常模糊并且经过预先验证/证明。我们对其他人和自己的看法总是是我们有优点、平均和缺点。因为某人在许多子测试中得分非常相似,但这并不意味着它在所有或大多数任务中具有非常相似的能力水平。

    • 回复: @Santoculto
  355. Santoculto 说:
    @CanSpeccy

    哈哈哈,确切地说,他们太天真了还是其他什么…… 😉

    自闭症..

  356. Santoculto 说:
    @Santoculto

    G=常识

    IDEAL G = 博学者预科/博学者

    真实 G = 肤浅的 一般知识,平均

  357. @James Thompson

    我们拥有有史以来最大的图书馆,几乎可以访问每个书架。

    一个图书馆,人们不断喊着“买这件 T 恤”、“保护和滋养的天然除臭剂”、“立即订阅,每天只需 37 美分”、“看看这个裸体女孩”、“获取全文 40.00 美元”,与此同时,美国的学术成就标准没有任何改善,生产率增长的步伐有所下降,而在全球范围内, 智商分数正在快速下降.

    • 回复: @RaceRealist88
  358. @CanSpeccy

    人性化的最大希望可能是建立一个储备体系,让人类生活在基本静态的各种文化体系下。

    哦,那会发生的。哦,弗舍尔,那会发生的。

  359. @utu

    我只是不太擅长类比和隐喻的层次。我倾向于在双曲线方面犯错误。我想,再教育营就可以了。

    只要那些智商最高的人就能睡上铺!!

  360. @CanSpeccy

    一个图书馆,人们不断喊着“买这件 T 恤”、“保护和滋养的天然除臭剂”、“立即订阅,每天只需 37 美分”、“看看这个裸体女孩”、“获取全文 40.00 美元”,

    Sci-Hub 的存在是有原因的。

  361. @CanSpeccy

    即便如此,从电脑屏幕上推断出来的任何东西都不具有直接观察的可靠性,所以也许我们对现实本质的了解确实不如我们的祖先,他们没有受益于互联网、电视或大众媒体。

    感谢上帝,YouTube 证明了一切。

    我倾向于不同意你的“好书”立场。一本书只是一个人的表达,解释他的个人观点。康德的 “纯粹理性批判 被认为是一本“好书”,但即使对于那些如此受膏的人来说,这也是沉重的,最后,人们可能会眯起眼睛,相当合理地宣布“我不太确定我是否同意这一点”。

    互联网提供了一种选择性搜索信息的手段。返回的大量信息并不一定有效,也不一定经过验证;然而,范围的扩大提供了视角,从而改进了从大量,呃,有些无定形但相关的数据中提炼出有用的结论。

  362. @James Thompson

    就我而言,我认为它增加了我的临时知识,但对我的心智能力的增加却很少,而且很可能没有任何作用。

    它提高了综合和整体整合。和同步,我认为。作为一个智商主义者,我倾向于相信能力必须通过锻炼、通过应用和使用来发展,否则它们仍然是隐性的。

  363. Factorize 说:

    可以,

    感谢你的回复。我最近的在线学习真是太幸运了。谁知道学习可以如此有趣?当你最终能够摆脱所有其他可能妨碍你的事情时,就有可能取得很高的成就,并为你的成就产生巨大的个人满足感。

    确实,资源的海洋简直就是一片,一个人很可能在试图搜索所有资源时遭遇海难。然而,我已经能够很好地导航到赏金。

    我的杯子溢出来了。到处都是宝藏!

    有了所有可用的在线资源,我就没有必要对任何事情不清楚。通常,教科书会有一些以非常混乱的方式呈现的主题。通常这些正是考试的主题。每当我遇到这样的话题时,我都会上网从其他教科书中找到类似的描述。通常没有两本教材会选择同一个题目来乱码。我会坚持不懈地继续理解一个主题,直到我完全满意它对我来说是清楚的。有时我会访问数十个网站来澄清一些困扰我的问题。通过回溯不清楚的主题,人们可以形成近乎完美的理解
    整本教科书。

    如果我需要一个例子来证明一个观点,那么我可以在 pubmed 上找到一百万个或更多。它是无限的。
    在我的研究中,没有什么值得担心的糟粕。虽然导师没有在我的道路上指导我,但搜索引擎几乎帮助我完成了这个角色。有时他们似乎不可思议地无所不知。

    奇怪的是,这确实有助于强调我关于 1000 IQ 生活优势的观点。显然,有时我确实觉得有一座信息山超出了我的能力范围。
    我根本无法阅读 1 万篇有关癌症的文章。即使我这样做了,我也怀疑读完它之后我是否能比《两个城市的故事》有更多的话要说。现在想想如何
    智商 1000、记忆力完美的人可能会做出回应。对于这样的人来说,不同研究领域之间的联系是非常清楚的。

    {值得注意的是,当约翰·冯·诺依曼患上胰腺癌时,他似乎甚至没有尝试与当时可用的癌症文献进行互动。即使对他来说,开始一项访问各个图书馆并寻找可能有帮助的治疗方法的研究计划也一定是完全没有希望的。如果他在现代互联网时代,我非常怀疑他会做出类似的行为。}

    我发现任何有兴趣的人都可以通过访问当前文献来发展专家水平的知识,这非常令人兴奋。我的猜测是,这正是当今孩子们成长过程中所发生的事情。

    我确信这正是我现在要做的事情。所有这一切都可能发生在地下。并不需要华丽的天才计划。基本上,一个在家接受教育的婴儿,一台可以上网的电脑,每周学习 20 个小时,可以持续 100 年。任何可能反对的人都不会抱怨:“哦,他们的重力不够高”。任何在这样的时间表下努力工作的人都会在任何特定领域发展出非凡的才能。

    我认为这样的未来可能是不可避免的。
    所展示的有意义的现实世界成就将是非凡的。

  364. Factorize 说:

    汤普森医生,

    是的,这绝对是有史以来最好的时光。传递一点信息的时代
    几乎没有边际成本给了我们接近信息饱和的生活。同时,
    允许开放获取越来越多的人类智慧储备的运动意味着,这将不仅仅是空洞信息内容的饱和。

    然而,对于那些有更多社交倾向的人来说,这可能接近有史以来最糟糕的时期。如果知识分子沉浸在虚拟现实的乐趣中,而让其他人去应对现实,那么这个世界肯定会带来一些挑战。

    这一转变的全部影响尚未显现。如上所述,什么
    当孩子们开始拒绝他们预先简化的餐桌/点菜生活并开始越野时会发生什么?让孩子们有机会获得广泛的信息和实验室资源可能会对人类产生非常深远的影响。

    尽管我自认为是一名量化分析师,但量化方面却出现了大规模的失败。尽管我们已经经历了信息巨大繁荣的时期,但经济指标并没有反映出这种转变。我可能每年从这个新的互联网时代获得大约 20,000 美元的享受和启发,但这些财富很少或根本没有反映在硬性经济统计数据中。

    心理测量学并没有更好地量化这种变化。据报道,成绩水平没有变化,甚至可能为负。但与此同时,存在着一种压倒性的、强大的新能力,可以通过利用人类的集体智慧来扩大一个人的心智范围。当然,这必须被视为弗林效应的一种新形式。这之所以在很大程度上被忽视,是因为焦点完全集中在 g 上。 g 可能没有改变,尽管典型学生的智力范围肯定已经改变了。就好像我们将学业成绩标准重新调整了 10 倍,却没有人注意到。

    即使将信息转储到短期记忆中,也能让您有机会建立联系并看到新的可能性。 AHA 时刻有可能发生。这些洞察力的火花始终推动人类前进。在当前的信息技术下,这样的火花应该会更频繁地发生。

    如果有任何教育工作者,或者任何人与这样的人有一定的距离,那么阅读他们对互联网时代转变的见解以及这如何影响学生的表现将具有很大的价值。我很想听听这个。在书面作业中应该最深刻地注意到这种差异。在回顾了我自己的记录后,我可以证明这种差异就像白天和黑夜一样。

    互联网等工具显然可以让我们更深入地了解人类知识的宝库。我发现任何作业风格的问题都可以研究得非常详细
    并且可以产生复杂的答案。这不等于 g,尽管很常见
    理解起来还是会认为它很聪明。也许这可能是摆脱无休止的争论的一种出路。

    • 回复: @AP
    , @James Thompson
  365. AP 说:
    @Factorize

    心理测量学并没有更好地量化这种变化。据报道,成绩水平没有变化,甚至可能为负。但与此同时,存在着一种压倒性的、强大的新能力,可以通过利用人类的集体智慧来扩大一个人的心智范围。

    这两种现象并不相互排斥。研究表明,全球定位系统导致人们自然找到目的地的能力下降。拼写检查导致人们拼写得更糟糕。计算器意味着记住乘法表的人越来越少。年长的医生抱怨说,由于谷歌的便捷性,年轻人记住的东西远远少于他们应有的,这使得他们处理“当下”情况的能力较差。让一个健康的人坐在电动轮椅上一年,看看当他试图离开椅子时,他失去了多少力量(也许还​​有平衡能力)。

    自然能力常常会因人工辅助而萎缩。

    • 回复: @RaceRealist88
  366. @AP

    研究表明,全球定位系统导致人们自然找到目的地的能力下降。拼写检查导致人们拼写得更糟糕

    资源?

    • 回复: @res
    , @AP
  367. @Factorize

    心理测量学只很少涉及一个人所拥有的知识储备。一般知识被视为只要你聪明就可以学到的东西。因此,扩展的图书馆会改变你的知识,但只有当你将这些碎片组合成一种有助于加深理解的新形式时,它才会有价值。我们很少有人能够做到这一点。因此,人类成就的比较研究非常关注创新率。我同意这是一个重要的衡量标准,但不同时代的量化存在相当大的问题。

    • 回复: @CanSpeccy
  368. utu 说:
    @CanSpeccy

    肖克利招妓:只是异性恋。不说话。一言不发。

    • 回复: @CanSpeccy
  369. Factorize 说:

    汤普森医生,

    这些变化一次又一次地发生,但又一次又一次地被忽视。就好像我们的整个星球转移到了一个新的太阳系,而这些信息永远不会被记录下来。
    历史书上有提到。令人惊讶的是,尽管智商分数在几十年来不断增加,但没有人想到寻找这种时间趋势。通过简单地重新规范测试,这些差异一直被掩盖了。

    现在似乎正在发生类似的事情。尽管技术革命极大地增强了人们的适应能力,但智商测试的分数可能会下降。所涉及的变化不是 g,尽管这是测试完成方式的结果。

    当我参加考试时,通常只允许使用单行不可编程计算器。如果放宽规则,以便我可以携带我选择的任何电子设备,那么成绩可能会被视为高度可疑,因为我将拥有一个可供我使用的信息资源世界。我什至可以打电话给任何特定学科领域的世界专家并征求他们的意见。然而,一旦我离开高度人工化的考试环境,我就可以自由地使用我所有的技术设备,以帮助我在世界上更智能地运作。

    心理测量学扭曲了智力的测量,以适应​​一个世纪或更久以前的测试环境。这几乎就像心理测量学家想象的那样,五万年前,除了大脑之外,没有任何复杂工具的原始人类可能会对认知测试做出反应。我们应该假装信息革命从未发生过,这样就不会出现中断。

    我意识到这在尝试比较不同时代时会带来挑战。然而,通过尝试从测试环境中删除技术工具,人们正在创建一种高度人为的环境。当人们被问到声速或如何在太空中导航时,他们的表现似乎比过去更差,背景就被忽略了。与其说它们确实表现出了性能下降,不如说是测试行为的适应性意义发生了变化。

    考虑这个问题的一个好方法是考虑图灵测试。你可以在屏幕上输入问题,有人会回答。人们可以想象,在一种情况下,该人只能访问典型的高中图书馆,而在另一种情况下,该人可以访问互联网的全部内容,而无法联系任何人寻求直接帮助。毫无疑问,答复质量的差异会非常大。

    这并不完全是一个空洞的格丹肯实验。正如 Afrosapiens 所指出的,对于上述在非洲进行图灵测试的人来说,可用的典型资源可能甚至比高中图书馆还少。 g 由于非洲甚至缺乏基本的信息资源,目前存在的差异正在大大扩大。在现代世界,技术迅速趋于均等,然后
    g可以再次被置于中心,作为社区分化的驱动力。

    • 回复: @utu
  370. Factorize 说:

    AP,

    这是一个很好的观察。

    在“有效”智力可能增加的同时,“测量”智力可能会减少,尤其是当手机应用程序、计算器等智商放大器被保留在测试环境中时。在允许技术设备的更自然的现实世界条件下测试人们会很有趣。确实,您不会在不同时间段内将苹果与苹果进行比较,但至少您不会在给定时间内将苹果与柠檬进行比较。

  371. res 说:
    @RaceRealist88

    我还没有看到关于 GPS 的明确研究,但这里有一篇示例文章: https://www.gislounge.com/spatial-orientation-and-the-brain-the-effects-of-map-reading-and-navigation/
    如果您选择查看,Google 会显示更多内容。

    • 回复: @AP
  372. utu 说:
    @Factorize

    考虑这个问题的一个好方法是考虑图灵测试……这个人只能访问典型的高中图书馆,而在另一种情况下,这个人可以访问互联网的全部内容

    图灵测试永远不会涉及数据库中存储的知识。那就太简单了。图灵测试将是关于听起来像人类的。关于他察觉微妙讽刺或讽刺或文字游戏的能力。多种含义。不用大词表达复杂的情感。但当我在这里关注你的科幻幻想评论时,我可以想象这样一种情况:人们最终听起来像机器,然后需要进行反图灵测试,人们将竞争不被人工智能检测为人类机器。我认为你在这次比赛中可能会取得很好的成绩。只要坚持路线就好。不要改变任何东西。

  373. Factorize 说:

    乌图,谢谢你的回复。

    是的,您对图灵测试的描述更符合其意图。

    然而,我很想听听你对我们错过了这一重大转变的看法。
    正如我在图灵测试建议中试图描述的“有效”智能。教育系统在知识展示的基础上测试学生。如果这不重要,那么他们可以简单地允许人们携带手机和平板电脑参加考试。我不知道为什么这在心理测量讨论中没有更加突出。

    您坚持认为图灵测试不应该是知识的展示,这是完全正确的。它更多地是在测试一种影响力的展示。

    能够理解人类堆积如山的知识并从中提取某种意义的计算机程序对人类来说具有极大的价值。这将是人类放大“有效智力”的另一种方式。

  374. @James Thompson

    因此,扩展的图书馆会改变你的知识,但只有当你将这些碎片组合成一种有助于加深理解的新形式时,它才会有价值。

    确切地。这就是为什么专家、研究人员、发现和发明者往往高度关注有限的知识领域。通过在有限的领域拥有百科全书式的知识,他们可以看到所有的角度和所有的陷阱(至少是大多数)。在科学或技术领域,他们知道哪些技术已被证明是有效的,他们知道哪些技术是可疑或无效的,他们知道谁的工作经得起审查,谁的工作经不起审查。

    这些人可以成为有价值的导师。他们可以引导你获取所需的信息,以便掌握某个主题的要点,他们可以指出关键的联系、推论和含义。它们可以帮助您避免浪费时间在网络上随手可得的大量外围、不相关、误导性或错误的材料上。

    • 回复: @Santoculto
  375. @utu

    是的,肖克利显然是一个心理发展不平衡和强迫性人格的怪人,而这是创造力的先决条件。

  376. Anonymous [又名“戈德·马斯登”] 说:

    男孩在青春期早期智商会提高 4 分,这一想法可能是有史以来最违反直觉的结果。

    • 哈哈: res
  377. Santoculto 说:
    @CanSpeccy

    现在你需要和他们一起学习,不要再过度地利用智商测试了。

  378. Anon • 免责声明 说:
    @jacques sheete

    3.8分!如果男性如此聪明,那为什么 80% 的犯罪都是男性犯下的呢?

    • 回复: @Joe Franklin
  379. @Anon

    因为公共法律是由女权主义者和酷儿绝大多数人制定和批准的?

  380. Anonymous [又名“Drhouse4882”] 说:

    男性和女性有不同的优点和缺点。男人的优势使他们能够创造出推动社会进步的东西。女性的优势(语言、表达思想)并不能推动社会进步。
    即使你选择智商相同的男性和女性,男性也会为你打造一些女性无法做到的可靠的东西。

当前评论者
说:

发表评论 -


 记得 我的信息为什么?
 电子邮件回复我的评论
$
提交的评论已被许可给 Unz评论 并可以由后者自行决定在其他地方重新发布
在翻译模式下禁用评论
通过RSS订阅此评论主题 通过RSS订阅所有James Thompson的评论